You are on page 1of 273

F5 Course notes

CHAPTER 1
ACTIVITY BASED COSTING
In F2, we have discussed two traditional costing methods: - absorption costing
and marginal costing. What was the main difference? In absorption costing,
both fixed and variable production overheads are charged to production. In
marginal costing, only variable production overheads are charged to
production.

Absorption Costing

In absorption costing, overheads are allocated to products using a three-stage


procedure: -

Stage 1: - Overheads are allocated or apportioned to cost centres (usually


production and service departments) using suitable basis

Stage 2: - Service centre costs are reapportioned to production centres

Stage 3: - Overheads are absorbed into units of production using an overhead


absorption rate

OAR = Budgeted overheads


Budgeted activity level

The budgeted activity level is usually taken as direct labour hours, machine
hours or number of units.

Illustration 1

Budgeted overheads $70,500

Products X Y Z

Budgeted units 15,000 8,000 2,000

Direct Labour(hours) 1 3 4

2
OAR = Budgeted overheads = $70,500
Total labour hours (15000 x 1) + (8000 x 3) + (2000 x 4)

= $70,500
47,000

= $1.50/labour hour

The overhead cost charged per unit is:-

Products X Y Z

Overhead cost (1.5 x 1) (1.5 x 3) (1.5 x 4)

$1.50 $4.50 $ 6.00

If either or both of the actual overhead cost or activity volume differ from
budget, the use of this rate is likely to lead to what is known as under-
absorption or over-absorption of overheads.

Illustration 2

Budgeted overheads $70,500 OAR = $1.50/labour hr

Budgeted labour hours 47,000

Actual labour hours 50,000

Actual overheads $85,000

Overheads absorbed = 50,000 x $1.50 = 75,000

Overheads incurred = 85,000

Overheads Under absorbed = 10,000

Finding the cost per unit Absorption Costing

Cost Card

Direct Materials 15.00


Direct Labour 18.00
Prime Cost 33.00
Variable Overheads 2.00
Fixed Overheads 3.00
Full Production Cost 38.00

3
Marginal Costing

Marginal costing is the accounting system in which variable costs are charged
to cost units and fixed costs of the period are written off in full against the
aggregate contribution.

Contribution is the difference between sales value and the variable cost of
sales.

Contribution = Selling price ALL variable costs

Total Contribution = contribution per unit x sales volume

Profit = Total contribution Fixed overheads

Find the cost per unit Marginal Costing

Cost Card

Direct Materials 15.00


Direct Labour 18.00
Prime Cost 33.00
Variable Overheads 2.00
Marginal Production Cost 35.00

Absorption vs Marginal Costing Profit

Reported profit figures using marginal costing or absorption costing will differ if
there is any change in the level of inventories in the period. If production is
equal to sales, there will be no difference in calculated profits using the
costing methods.

If inventory levels increase between the beginning and end of a period,


absorption costing will report the higher profit. Some of the fixed production
overhead incurred during the period will be carried forward in closing inventory
(which reduces cost of sales) to be set against sales revenue in the following
period instead of being written off in full against profit in the period concerned.

If inventory levels decrease, absorption costing will report the lower profit
because as well as the fixed overhead incurred, fixed production overhead
which had been carried forward in opening inventory is released and is also
included in cost of sales.

4
Therefore,

If inventory levels increase, absorption costing gives the higher profit


If inventory levels decrease, marginal costing gives the higher profit
If inventory levels are constant, both methods give the same profit

Profits generated using absorption & marginal costing can also be reconciled
as follows:

Difference in the profit = change in inventory in units x FOAR per unit

Illustration 3

The following budgeted information relates to a manufacturing company for


next period:

Units $
Production 14,000 Fixed production costs 63,000
Sales 12,000

The normal level of activity is 14,000 units per period.

Using absorption costing the profit for next period has been calculated as
$36,000.

What would the profit for next period be using marginal costing?

Difference in profit = change in inventory x FOAR/unit


= 2,000 x (63,000/14,000)
= 2,000 x $4.50
= $9,000

Inventories are increasing, hence absorption costing profit is higher.

Absorption costing profit 36,000


Difference in profits 9,000
Marginal costing profits 27,000

5
1.1 ACCA SYLLABUS GUIDE OUTCOME 1:
Identify appropriate cost drivers under Activity Based Costing (ABC)

1.1.1 Introduction to ABC

Absorption costing focuses on the product in the costing process. Costs are
traced to the product because each product item is assumed to consume the
resources.

However, in many modern-manufacturing operations, overheads are not


homogeneous in terms of being primarily influenced by volume. In fact, the
majority of overheads in a modern manufacturing operation are largely
unaffected by changes in production volume.

ABC is an alternative costing method to absorption costing. ABC links


overhead costs to the products or services that cause them by absorbing

overhead costs on the basis of activities that drive costs (cost drivers) rather
than on the basis of production volume.

In ABC, activities are the focus of the costing process. Costs are traced from
activities to products based on the products demands for these activities
during the production process. Activities may include equipment preparation,
order handling, quality control.

'Cost driver' is the term used for an activity which influences the amount of
total expenditure on a particular cost. For some costs, volume will be the cost
driver, but for many other costs, volume will be a very poor indicator.

By grouping costs on the basis of cost drivers, we will be able to both manage
costs better (by managing the activity) and to calculate the cost of production.

Examples of cost drivers would be: -

Ordering costs no. of orders


Set-up costs no. of set-ups
Packing costs no. of packing orders

6
1.2 ACCA SYLLABUS GUIDE OUTCOME 2:
Calculate costs per driver and per unit using ABC

1.2.1 Steps in establishing and applying ABC

There are 5 main steps in establishing and applying ABC: -

1. Identify activities that consume resources and incur overhead costs.


2. Allocate overhead costs to the activities that incur them.
3. Determine the cost driver for each activity or cost pool.
Each group of costs which are influenced by a particular cost driver is
referred to as a 'cost pool'.
4. Collect data about actual activity for the cost driver in each cost pool
5. Calculate the overhead cost of products or services.
This is done by calculating an overhead cost per unit of the cost driver.
Overhead costs are then charged to products or services on the basis
of activities used for each product or service.

identify what trace the cost


calculate the
identify a cost causes/drives into the units
cost per driver
it produced

Extracted from Article Activity-Based Costing by K. Garrett, Student


Accountant February 2010
http://www.accaglobal.com/content/dam/acca/global/pdf/sa_jan10_garrett.pdf

The following example looks at the different activities within a company, their
cost and their cost driver. The cost per driver is found by dividing the total cost
of the activity by the quantity of the cost drivers. Overhead costs are then
charged to products or services on the basis of activities used for each
product or service.

Activity Cost Pool Cost Driver Cost/ Driver

7
$ Volume $
Process set up 37,500 100 set ups 375 / set up
Material 9,000 50 purchase orders 180 / purchase
procurement order
Maintenance 10,000 10 standard 1000/
maintenance plans maintenance
plan
Material handling 22,500 2,000 material 11.25 / material
movements movement
Quality control 20,500 250 inspections 82 / inspection
Order processing 13,000 300 customers 43.33 /
customer
$112,500

1.3 ACCA SYLLABUS GUIDE OUTCOME 3:


Compare ABC and traditional methods of overhead absorption based
on production units, labour hours or machine hours

Traditional absorption costing assumes that overhead expenditure is related


to direct labour hours, machine hours or production units. However, this
assumption is no longer reliable in many companies.

Using ABC to allocate overhead costs to products will lead to very different
values of overheads allocated per unit.

Lecture Example 1

Kira manufactures three products: X, Y, and Z.

Data for the period just ended is as follows:

X Y Z
Production (units) 20,000 25,000 5,000
Sales Price (per unit) $18 $40 $60
Material (kg per unit) 1kg 2kg 3kg
Labour Hours (per unit) 2 hours 1 hour 1 hour

(Material cost is $5 per kg and labour is paid at the rate of $5 per hour)

8
Total overheads for the period were as follows:

$
Set-up costs 100,000
Machining 55,000
Receiving 40,000
Ordering 15,000
210,000

The following data is available:

Product
X Y Z
Machine hours per unit 2 2 2
Number of set-ups 10 13 2
Number of deliveries received 10 8 2
Number of orders done 30 30 20

Required: -

(a) Calculate the cost (and hence profit) per unit, absorbing all the
overheads on the basis of labour hours.

(b) Calculate the cost (and hence the profit) per unit absorbing the
overheads using an ABC approach. All calculations should be to 2
decimal places.

1.3.1 The Advantages of ABC

1. More accurate cost information is obtained. It identifies ways of reducing


overhead costs in the longer-term. This will enable managers to make
better decisions, particularly in respect of pricing and marketing activities.

2. In absorption costing, as the profitability of a product would be overstated,


the company's marketing effort is likely to be directed towards maximising
the sale of this product, with a lesser emphasis on the other products. In
addition, as the resulting selling price will be less than is required to fully
recover overheads and yield a satisfactory profit, the market will perceive
the product to be particularly attractive.

3. It provides much better insights into what drives overhead costs. ABC
recognises that overhead costs are not all related to volume. It also
identifies activities and costs that do not add value.

4. ABC can be applied to all overhead costs, not just production overheads.

9
1.3.2 Disadvantages of ABC

ABC may not be universally beneficial. There are four major issues to be
considered:

1. Cost vs benefit

The need to analyse costs on a radically different basis will require


resources, which will lead to additional costs. Clearly the benefits which
will be obtained must exceed these costs.

In general terms, an organisation which has little competition, a stable


and standardised product range and for which overheads represent a
small proportion of total cost, will not benefit from the introduction of
ABC.

2. Need for informed application

While ABC is likely to provide better information for decision makers, it


must still be applied with care. ABC is not fully understood by many
managers and therefore is not fully accepted as a means of cost
control.

3. Difficulty in identifying cost drivers

In a practical context, there are frequently difficulties in identifying the


appropriate drivers. ABC costs are based on assumptions and
simplifications. The choice of both activities and cost drivers might be
inappropriate.

4. Lack of appropriate accounting records

ABC needs a new set of accounting records, this is often not


immediately available and therefore resistance to change is common.
The setting up of new cost pools is needed which is time-consuming.

Lecture Example 2 (extracted from the article Activity-Based Costing by K.


Garrett, Student Accountant February 2010)
http://www.accaglobal.com/content/dam/acca/global/pdf/sa_jan10_garrett.pdf

A company offers two products: ordinary and deluxe. The company knows
that demand for the deluxe range will be low, but hopes that the price
premium it can charge will still allow it to make a good profit, even on a low
volume item.

10
The following data is available: -

Budget Ordinary Units Deluxe Units

Units produced 20,000 2,000

Costs per unit $ $

Material 10 12
Labour (5 hours x $12/hr) 60 (6 hours x $12/hr) 72
Var overhead (5 hours x $1/hr) 5 (6 hours x$1/hr) 6
Marginal cost 75 90

Budgeted fixed production overheads are $224,000.

An analysis of the fixed overheads of $224,000 shows that they consist of: -

$
Batch set-up costs 90,000
Stores material handling etc 92,000
Other (rent, etc) 42,000
Total 224,000

Ordinary units are produced in long production runs, with each batch
consisting of 2,000 units.

Deluxe units are produced in short production runs, with each batch consisting
of 100 units.

Each ordinary unit consists of 20 components, each deluxe unit of 30


components.

Required: -

(a) Calculate the cost per unit, absorbing the overheads on the basis
of labour hours.

(b) Calculate the cost per unit absorbing the overheads using an
Activity Based Costing approach.

(c) Explain how this company can benefit from using Activity-Based
Costing in dealing with its fixed overheads.

11
1.3.3 Service organisations

Five key characteristics of service organisations are: -

Simultaneity/spontaneity (production and consumption of the service


coinciding);
Perishability (the inability to store the service);
Heterogeneity (variability in the standard of performance of the
provision of the service);
Intangibility (of what is provided to and valued by individual customers).
No transfer of ownership.

ABC can be effectively applied to service organisations. Indeed, the fact that
for most service organisations, indirect costs will represent the major
proportion of total cost means that the technique is of particular relevance to
service organisations.

Further questions

Question 1

Which ONE of the following is an advantage of Activity Based Costing?

A. It provides more accurate product costs


B. It is simple to apply
C. It is a form of marginal costing and so is relevant to decision making
D. It is particularly useful when fixed overheads are very low

Question 2

Which of the following are benets of using activity based costing?

(1) It recognises that overhead costs are not always driven by the volume
of production.
(2) It does not result in under or over absorption of xed overheads.
(3) It avoids all arbitrary cost apportionments.
(4) It is particularly useful in single product businesses.

A. 1 only
B. 1 and 2 only
C. 2 and 3 only
D. 1 and 4 only

12
Question 3

In which circumstance is activity based costing a more useful approach to


product costing: -

A. One product is produced


B. Overheads form a high proportion of total costs
C. Overhead expenditure is driven by the volume of output
D. It is very difficult to identify the relevant cost drivers

13
CHAPTER 2
TARGET COSTING
2.1 ACCA SYLLABUS GUIDE OUTCOME 1:
Derive a target cost in manufacturing and service industries

A target cost is a cost estimate derived by subtracting a desired profit margin


from a competitive market price. This market price is determined based on the
expected price to be paid by the market to achieve a certain market share and
sales volume. The required profit margin is then deducted from the anticipated
selling price to arrive at the target product cost. A product of acceptable
quality is then designed within that cost.

The main focus of target costing is not finding what a new product does cost
but what it should or needs to cost. The firm can then focus on the costs
which can be reduced to achieve the target cost.

Target costing is used by such companies as Sony, Toyota and Swatch.

2.1.1 Steps in target costing 1

1. Target costing begins by specifying a product an organisation wishes to


sell. This will involve extensive customer analysis, considering which
features customers value and which they do not. Ideally only those
features valued by customers will be included in the product design.

2. The price at which the product can be sold at is then considered. This
will take in to account the competitors products and the market
conditions expected at the time that the product will be launched.
Hence a heavy emphasis is placed on external analysis before any
consideration is made of the internal cost of the product.

3. From the above price a desired margin is deducted. This can be a


gross or a net margin.

4. This leaves the cost target. An organisation will need to meet this target
if their desired margin is to be met.

5. Costs for the product are then calculated and compared to the cost
target. If it appears that this cost cannot be achieved then the
difference (shortfall) is called a cost gap. This gap would have to be

1 Examinable June 2012 Qs 2a, Sept/December 2015 Qs 1a

14
closed, by some form of cost reduction (for e.g. value engineering),
while satisfying the needs of customers.

6. Before going ahead with the project, the company may hold
negotiations with customers.

Illustration 1

Targeted selling price $20


Gross profit margin 20%

$
Target cost = Selling Price 20
Less margin (4) (20% of $20)
Target Cost 16

Lecture Example 1

Play plc is considering whether or not to launch a new product. It has targeted
a selling price of $100 per unit.

Play wants to earn a margin on selling price of 20%.

Calculate the target cost.

15
Lecture Example 2

A company, ABC Ltd, could sell 100,000 units per annum of a new product at
a competitive market price of $75 per unit. Capital investment of $10,000,000
would be required to manufacture the product. The company seeks to earn a
return on initial capital employed of 20% per annum.

Required:

What is the target cost per unit of the new product? _________

2.2 ACCA SYLLABUS GUIDE OUTCOME 2:


Explain the difficulties of using target costing in service industries

Target costing was introduced by major Japanese manufacturing companies


for use when:

1. a new product was to be designed to meet the target cost


2. a substantial part of the production cost consisted of bought-in
materials

Four key characteristics of service organisations are: -

Simultaneity/spontaneity (production and consumption of the service


coinciding);
Perishability (the inability to store the service);
Heterogeneity (variability in the standard of performance of the
provision of the service);
Intangibility (of what is provided to and valued by individual customers).
No transfer of ownership.

Hence, although target costing can be used in service industries, it may face a
number of problems: -

1. it is very difficult to determine a market-driven price for services


provided
2. the introduction of new services occurs far less frequently than in a
manufacturing company.
3. the major cost in the service industry is salaries. Bought-in materials
are usually low when compared to salaries. It is very difficult to reduce
the cost of salaries!

16
2.3 ACCA SYLLABUS GUIDE OUTCOME 3:
Suggest how a target cost gap might be closed

Where a gap exists between the current estimated cost levels and the target
cost, it is essential that this gap be closed. Efforts to close a target cost gap
are most likely to be successful at the design stage. It is far easier to design
out cost during the pre-production phase than to control out cost during the
production phase.

2.3.1 Ways to reduce a cost gap

1. Review the products features.

2. Remove features that add to cost but do not significantly add value to
the product when viewed by the customer (non-value-added activities).
This should reduce cost but not the achievable selling price (value
engineering / value analysis).

3. Team approach - cost reduction works best when a team approach is


adopted. The company should bring together members of the
marketing, design, assembly and distribution teams to allow discussion
of methods to reduce costs. Open discussion and brainstorming are
useful approaches here.

4. Review the whole supplier chain - each step in the supply chain should
be reviewed, possibly with the aid of staff questionnaires, to identify
areas of likely cost savings. For example, the questionnaire might ask
are there more than five potential suppliers for this component?
Clearly a yes response to this question will mean that there is the
potential for tendering or price competition.

5. Efficiency improvements should also be possible by reducing waste or


idle time that might exist. Where possible, standardised components
should be used in the design. Productivity gains may be possible by
changing working practices or by de-skilling the process. Automation is
increasingly common in assembly and manufacturing.

Lecture Example 3

Kingo is in the process of introducing a new product and has undertaken


market research to find out more about competitors products. A target selling
price of $60 has been established.

The target profit margin for each unit is 30% of the proposed selling price.

Cost estimates have also been prepared

17
Production costs per unit $

Direct material 13.00


Direct labour 14.00
Direct machinery costs 1.10
Design 7.00
Quality assurance 4.50

Non-production costs per unit

Marketing 3.20
Distribution 3.50
After-sales service and warranty costs 1.10

Required:
a. Calculate the target cost for each unit
b. Identify any cost gap which may have arisen
c. Suggest ways in which Kingo may reduce their unit cost

2.3.2 Difficulties in implementing target costing 2

1. As has already been mentioned, it is often more difficult to use target


costing in service organisations: for target costing to be useful, a
service has to be clearly defined.

2. For one-off jobs, comparative data may not be available. Hence, it


would be difficult to set a target cost.

3. For specialist jobs, it may be difficult to establish a market price. Hence


target costing will be difficult to use.

2.3.3 Benefits of Target Costing 3

1. When should target costing be used? It is useful in competitive


markets. Hence, the company has to accept the price set by the market
for their products. This would help the company focus of the price of
the goods and services offered by competitors.

2. Target costing helps an organization to look into its internal processes


and their costs more closely. It should find ways how to close the cost
gap: focus on reducing costs and retaining customers.

Further Questions

2 Examined Sept/Dec 2015 Qs 1b


3 Examined Sept/Dec 2015 Qs 1b

18
Question 14

S Company is a manufacturer of multiple products and uses target costing. It


has been noted that Product P currently has a target cost gap and the
company wishes to close this gap.

Which of the following may be used to close the target cost gap for product P?

A. Use overtime to complete work ahead of schedule


B. Substitute current raw materials with cheaper versions
C. Raise the selling price of P
D. Negotiate cheaper rent for S Companys premises

Question 25

The selling price of Product X is set at $550 for each unit and sales for the
coming year are expected to be 800 units.

A return of 30% on the investment of $500,000 in Product X will be required in


the coming year.
What is the target cost for each unit of Product X?

A. $385
B. $165
C. $18750
D. $36250

Question 3

The selling price of product K is set at $450 for each unit.

If the company requires a return of 20% in the coming year on product K, the
target cost for each unit for the coming year is:

A. $300
B. $360
C. $400
D. $450

4 Specimen Exam Applicable from December 2014


5 Specimen Exam Applicable from December 2014

19
Question 4

Which of the following BEST describe target costing?

A. Setting a cost by subtracting a desired profit margin from a competitive


market price
B. Setting a price by adding a desired profit margin to a production cost
C. Setting a cost for the use in the calculation of variances
D. Setting a selling price for the company to aim in the long run

Question 5

Which of the following describes target costing?

A. A method of costing that sets a target cost by subtracting a desired


profit margin from a competitive market price.
B. A method of costing that sets a target price by adding a desired profit
margin to actual cost.
C. A method of costing that targets selected business departments and
aims to minimize their costs.
D. A method of costing whose target is to reduce unit cost without
impairing value to the customer.

Question 6

Edward Co assembles and sells many types of radio. It is considering


extending its product range to include digital radios. These radios produce a
better sound quality than traditional radios and have a large number of
potential additional features not possible with the previous technologies
(station scanning, more choice, one touch tuning, station identification text
and song identification text etc).

A radio is produced by assembly workers assembling a variety of


components. Production overheads are currently absorbed into product costs
on an assembly labour hour basis.

Edward Co is considering a target costing approach for its new digital radio
product.

Required:

(a) Briefly describe the target costing process that Edward Co should
undertake.
(3 marks)

20
(b) Explain the benefits to Edward Co of adopting a target costing approach at
such an early stage in the product development process.
(4 marks)

(c) Assuming a cost gap was identified in the process, outline possible steps
Edward Co could take to reduce this gap.
(5 marks)

A selling price of $44 has been set in order to compete with a similar radio on
the market that has comparable features to Edward Cos intended product.
The board have agreed that the acceptable margin (after allowing for all
production costs) should be 20%.

Cost information for the new radio is as follows:

Component 1 (Circuit board) these are bought in and cost $410 each.
They are bought in batches of 4,000 and additional delivery costs are $2,400
per batch.

Component 2 (Wiring) in an ideal situation 25 cm of wiring is needed for


each completed radio. However, there is some waste involved in the process
as wire is occasionally cut to the wrong length or is damaged in the assembly
process. Edward Co estimates that 2% of the purchased wire is lost in the
assembly process. Wire costs $050 per metre to buy.

Other material other materials cost $810 per radio.

Assembly labour these are skilled people who are difficult to recruit and
retain. Edward Co has more staff of this type than needed but is prepared to
carry this extra cost in return for the security it gives the business. It takes 30
minutes to assemble a radio and the assembly workers are paid $1260 per
hour. It is estimated that 10% of hours paid to the assembly workers is for idle
time.

Production Overheads recent historic cost analysis has revealed the


following production overhead data:

Total Production o/heads Total assembly labour hrs


$
Month 1 620,000 19,000
Month 2 700,000 23,000

Fixed production overheads are absorbed on an assembly hour basis based


on normal annual activity levels. In a typical year 240,000 assembly hours will
be worked by Edward Co.

21
Required:

(d) Calculate the expected cost per unit for the radio and identify any cost gap
that might exist.
(13 marks)

(Acca Paper December 2007 Question 1)

22
CHAPTER 3
LIFE-CYCLE COSTING
Life-cycle costing tracks and accumulates the actual costs and revenues
attributable to each product from inception to abandonment. It enables a
products true profitability to be determined at the end of the economic life.

Traditional cost accounting systems do not accumulate costs over a products


entire life but focus instead on (normally) twelve month accounting periods. As
a result the total profitability of a product over its entire life becomes difficult to
determine.

3.1 ACCA SYLLABUS GUIDE OUTCOME 1:


Identify the costs involved at different stages of the life-cycle

As mentioned in Chapter 2, target costing places great emphasis on


controlling any of the costs that relate to any part of the products life.

Every product goes through a life cycle.

1. Development. The product has a research and development stage


where costs are incurred but no revenue is generated. During this
stage, a high level of setup costs will be incurred, including research
and development, product design and building of production facilities.

2. Introduction. The product is introduced to the market. Potential


customers will be unaware of the product or service, and the
organisation may have to spend further on advertising to bring the
product or service to the attention of the market. Therefore, this stage
will involve extensive marketing and promotion costs. High prices may
be changed to recoup these high development costs.

3. Growth. The product gains a bigger market as demand builds up. Sales
revenues increase and the product begins to make a profit. Marketing
and promotion will continue through this stage. Unit costs tend to fall
as fixed costs are recovered over greater volumes. Competition also
increases and the company may need to reduce prices to remain
competitive.

4. Maturity. Eventually, the growth in demand for the product will slow
down and it will enter a period of relative maturity. It will continue to be
profitable. However, price competition and product differentiation will

23
start to erode profitability. The product may be modified or improved, as
a means of sustaining its demand.

5. Decline. At some stage, the market will have bought enough of the
product and it will therefore reach 'saturation point'. Demand will start to
fall and prices will also fall. Eventually it will become a loss maker and
this is the time when the organisation should decide to stop selling the
product or service. During this stage, the costs involved would be
environmental clean-up, disposal and decommissioning. Meanwhile, a
replacement product will need to have been developed, incurring new
levels of research and development and other setup costs.

The level of sales and profits earned over a life cycle can be illustrated
diagrammatically as follows.

24
3.2 ACCA SYLLABUS GUIDE OUTCOME 2:
Derive a life cycle cost in manufacturing and service industries

Lecture Example 1

Quick Ltd is launching a new product on the market. The following costs have
been estimated for the whole life of the product:

Research and development (already incurred) $30,000


Marketing costs $6,000
Production cost per unit $5
Selling and distribution costs $4,000

The expected number of units to be produced and sold is 10,000.

Required

Calculate the life-cycle cost per unit.

Lecture Example 2 (extracted from the article Target Costing and Life-Cycle
Costing by K. Garrett, Student Accountant, March 2010)
http://www.accaglobal.com/content/dam/acca/global/pdf/Feb10_tarcosting_F5.pdf

A company is planning a new product. Market research information suggests


that the product should sell 10,000 units at $21.00/unit. The company seeks to
make a mark-up of 40% product cost. It is estimated that the lifetime costs of
the product will be as follows:

1) Design and development costs $50,000


2) Manufacturing Costs $10/unit
3) End of life costs $20,000

The company estimates that if it were to spend an additional $15,000 on


design, manufacturing costs/unit could be reduced.

Required

a. What is the target cost of the product?


b. What is the original lifecycle cost per unit and is the product
worth making on that basis?
c. If the additional amount were spent on design, what is the
maximum manufacturing cost per unit that could be tolerated if
the company is to earn its required mark-up?

25
3.3 ACCA SYLLABUS GUIDE OUTCOME 3:
Identify the benefits of life cycle costing

The benefits of product life cycle costing are summarised as follows:

1. All costs (production and non production) will be traced to individual


products over their complete life cycles and hence individual product
profitability can be more accurately measured.

2. The product life cycle costing results in earlier actions to generate


revenue or to lower costs than otherwise might be considered.

3. Better decisions should follow from a more accurate and realistic


assessment of revenues and costs, at least within a particular life cycle
stage.

4. Product life cycle thinking can promote long-term rewarding in contrast


to short-term profitability rewarding.

5. It helps management to understand the cost consequences of


developing and making a product and to identify areas in which cost
reduction efforts are likely to be most effective. Very often, 90% of the
products life-cycle costs are determined by decisions made in the
development stage. Therefore, it is important to focus on these costs
before the product enters the market.

6. Identifying the costs incurred during the different stages of a products


life cycle provides an insight into understanding and managing the total
costs incurred throughout its life cycle. Non production costs will
become more visible and the potential for their control is increased.

7. More accurate feedback on the success or failure of new products will


be available.

To maximise a products return over its lifecycle, a number of factors need to


be considered: -

1. Design costs: - since approximately 90% of a products costs are often


incurred at the design and development stages of its life, it is absolutely
important that design teams work as part of a cross-functional team to
minimize costs over the whole life cycle.

26
2. Minimise the time to market: - since competition is harsh, it is vital to
get any new product into the marketplace as quickly as possible. and
make a profit before competition increases.

3. Maximise the length of the life cycle itself: - Generally, the longer
the life cycle, the greater the profit that will be generated. How can the
life cycle be maximised?
a. Get the product to the market as quickly as possible
b. Find other uses or markets for the product
c. Market skimming (introducing the product at a high price) will
prolong life and maximise the revenue over the products life.

4. Minimise break-even time: - The quicker costs are covered, the more
funds the company will have to develop further products.

Lecture Example 3

Birtles plc is a manufacturer of small domestic electrical appliances. Its market


is very competitive in terms of both price and new product innovation. As a
result product life cycles are short.

Birtles plcs managers are concerned about the reliability of its product costing
system. It currently uses an absorption costing system, and absorbs
overheads on the basis of budgeted direct labour hours. On this basis the
estimated cost of its latest product, a talking electric kettle, is as follows:

$ per unit
Direct Materials 4.50
Direct Labour ($12 per hour) 0.50
Production overheads ($120 per hour) 5.00
Production Cost 10.00

The firms management accountant has suggested that more accurate


product costs would be obtained if an activity based costing (ABC) approach
were used. He has collected the following information as a starting point for an
ABC treatment of production overhead cost.

Budgeted factory overhead per annum.

Cost Pools Cost per annum Cost Driver


$000
Stores administration 5,000 Number of different
components

27
Production Line set ups 3,000 Number of set ups
Dispatch 1,000 Number of dispatches
Other overheads 3,000 Direct labour hours
Total production overhead 12,000

Estimated activity per annum

Cost Driver Total Activity per annum


Number of components 2,000 items
Number of set ups 10,000 set ups
Number of dispatches 20,000 dispatches
Direct labour hours 100,000 hours

Each talking kettle uses 10 different components and kettle manufacture will
involve six production line set ups per annum. Five hundred dispatches will be
required per annum. Budgeted production is 10,000 kettles per annum.

Required:

Estimate the cost of a talking kettle using an ABC approach and the cost
drivers suggested by the management accountant.

Birtles plcs Finance Director supports the proposal to introduce activity based
costing but argues that the firm should consider all the costs involved in the
development, production and marketing of the kettle. In addition to the above
ABC costs, $30,000 has already been spent on research and development for
the talking electric kettle and he estimates that a further $5,000 will be spent
on marketing the new product. There are no other costs attributable to the
new product. Total sales over its life will be 10,000 units per annum for the
next two years.

On past experience he knows that the firm will have to reduce the selling price
of the kettle by 40% in its second year of sales in order to remain competitive.

Required:

Calculate the price to be charged per unit for the talking electric kettle in
the first year of sales so that it will earn an OVERALL 20% margin on
sales over its two year life after covering ALL attributable costs outlined
above.
(CAT Paper T7 December 2004 Qs no 3)

28
Further Questions

Question 1

Which of the following costs would be included to find the life-cycle cost of a
product?

(i) Research and development costs


(ii) Production costs
(iii) Distribution costs
(iv) Marketing costs

A. (i), (ii), (iv)


B. (ii), (iii), (iv)
C. (i), (ii), (iii)
D. All of the above

Question 26

The following costs arise in relation to production of a new product:

(i) Research and development costs


(ii) Design costs
(iii) Testing costs
(iv) Advertising costs
(v) Production costs

In calculating the lifetime costs of the product, which of the above items would
be EXCLUDED?

A. (i), (ii), and (iii) only


B. (ii) and (iii) only
C. (iv) and (v) only
D. None of the above

Question 3

The following statements relate to life-cycle costing:

(i) It helps forecast a products profitability over its entire life.


(ii) It takes into account a products total costs over its entire life.
(iii) It focuses on the production of monthly profit statements throughout a
products entire life.

6 Specimen Exam Applicable from December 2014

29
Which of the statements are true?

A. (i) only
B. (i) and (ii) only
C. (i) and (iii) only
D. (i), (ii) and (iii)

Question 4

In calculating the life cycle costs of a product, which of the following items
would be excluded?

i. Planning and concept design costs


ii. Preliminary and detailed design costs
iii. Testing costs
iv. Production costs
v. Distribution and customer service costs

A. (iii)
B. (iv)
C. (v)
D. None of them

30
CHAPTER 4
THROUGHPUT ACCOUNTING
4.1 ACCA SYLLABUS GUIDE OUTCOME 1:
Calculate and interpret a throughput accounting ratio (TPAR)

4.1.1 What is throughput?

Throughput is the rate of converting raw materials and purchased components


into products sold to customers. In money terms, it is the extra money that is
made for an organisation from selling its products.

Throughput = Revenue Raw material cost

4.1.2 What is throughput accounting?

Throughput accounting (TA) is an approach to accounting which is largely in


sympathy with the JIT philosophy. In essence, TA assumes that a manager
has a given set of resources available. These comprise existing buildings,
capital equipment and labour force. Using these resources, purchased
materials and parts must be processed to generate sales revenue. Given this
scenario the most appropriate financial objective to set for doing this is the
maximisation of throughput which is defined as:

sales revenue less direct material cost.

4.1.3 Main concepts in throughput accounting

1. In the short run, most costs in the factory (with the exception of
materials costs) are fixed. These fixed costs include direct labour.
These fixed costs are called Total Factory Costs (TFC) (operating
expenses).

2. In a JIT environment, the ideal inventory level is zero. Products should


not be made unless a customer has ordered them.

Work in progress should be valued at material cost only until the output
is eventually sold, so that no value will be added and no profit earned
until the sale takes place.

3. Profitability is determined by the rate at which sales are made and, in a


JIT environment, this depends on how quickly goods can be produced
to satisfy customer orders. Since the goal of a profit-orientated

31
organisation is to make money, inventory must be sold for that goal to
be achieved.

4.1.4 Traditional Cost Accounting versus Throughput Accounting Ratio

Conventional cost accounting Throughput accounting

1. Inventory is an asset. Inventory is not an asset. It is a result of


unsynchronised manufacturing and is a
barrier to making profit.

2. Costs can be classified either Such classifications are no longer


as direct or indirect. useful.

3. Product profitability can be Profitability is determined by the rate at


determined by deducting a which money is earned.
product cost from selling price.

4. Profit can be increased by Profit is a function of material cost, total


reducing cost elements. factory cost and throughput.

Marginal costing and throughput accounting both determine a contribution by


calculating the difference between sales revenue and variable costs. However
this contribution figure will be higher under throughput accounting since only
material costs are recognised as being variable costs. Under marginal costing,
direct labour costs and certain overhead costs will also be deducted from
sales revenues in order to calculate contribution. Throughput accounting
regards such costs as fixed and this is true insofar as they cannot be avoided
in the immediate sense.

Illustration 1

X Y

Sales revenue 25 30
Material cost 5 8
Labour cost (@ $3/hr) 3 6
Variable overheads 2 2
Fixed overheads 1 4
Max demand 10,000 15,000

Total labour hours available are 30,000 hours.

32
How do we calculate contribution?

X Y

$ $ $ $
Sales revenue 25 30
Less all Variable costs
Material 5 8
Labour 3 6
Variable cost 2 10 2 16
Contribution / unit 15 14

How do we calculate return?

X Y

$ $
Sales Revenue 25 30
Less Material 5 8
Return / unit 20 22

4.2 ACCA SYLLABUS GUIDE OUTCOME 2:


Discuss and apply the theory of constraints

The theory of constraints is applied within an organization by following the


five focusing steps a tool which was developed to help organisations deal
with constraints.

Step 1: Identify the systems bottlenecks


Step 2: Decide how to exploit the systems bottlenecks
This involves making sure that the bottleneck resource is actively
being used as much as possible and is producing as many units as
possible.
Step 3: Subordinate everything else to the decisions made in Step 2
The production capacity of the bottleneck resource should determine
the production schedule for the organization as a whole. Idle time is
unavoidable and needs to be accepted if the theory of constraints is to
be successfully applied.
Step 4: Elevate the systems bottlenecks
This will normally require capital expenditure.
Step 5: If a new constraints is broken in Step 4, go back to Step 1
The likely constraint in the system is likely to be market demand.

33
Lecture Example 1 (extracted from the article Throughput Accounting and
Backflush Accounting by K. Garrett, Student Accountant, March 2010) 7

A not-for-profit hospital performs a medical screening service in three


sequential stages:
1) Take an x-ray
2) Interpret the result
3) Recall patients who need further investigation/tell others that all is fine

Process Time/patient (hours) Total hours


available/week

Take an X-ray 0.25 40

Interpret the result 0.10 20

Recall patients who


need further investigation/
tell others that all is fine 0.20 30

Required: -

a. Find the bottleneck process


b. How can we increase the throughput of the process identified in
part (a) as the bottleneck?

Lecture Example 28

Cat Co makes a product using three machines X, Y and Z. The capacity of


each machine is a s follows:

Machine X Y Z
Capacity per week 800 600 500

The demand for the product is 1,000 units per week. For every additional unit
sold per week, net present value increases by $50,000. Cat Co is considering
the following possible purchases (they are not mutually exclusive):

Purchase 1 Replace machine X with a newer model. This will increase


capacity to 1,100 units per week and costs $6m

Purchase 2 Invest in a second machine Y, increasing capacity by 550 units


per week. The cost of this machine would be $6.8m.

7 http://www.accaglobal.com/content/dam/acca/global/pdf/Feb10_throughput_F5.pdf
8Irons A., Throughput accounting and the theory of constraints, November 2011
http://www.accaglobal.com/content/dam/acca/global/pdf/sa_oct11_throughput.pdf

34
Purchase 3 Upgrade machine Z at a cost of $7.5m, thereby increasing
capacity to 1,050 units.

Required: Which is Cat Cos best course of action?

4.2.1 The Throughput Accounting Ratio (TPAR)

Where there is a bottleneck resource (limiting factor), performance can be


measured in terms of throughput for each unit of bottleneck resource
consumed.

4.2.1.1 Three important ratios

Throughput (return) per factory hour =

Throughput per unit


Products time on the bottleneck resource

Cost per factory hour =


Total Factory Cost
Total time available on bottleneck resource

The cost per factory hour is across the whole factory and therefore only needs
to be calculated once (not for each product).

Throughput accounting ratio =


Return per factory hour
Cost per factory hour

TPAR>1 would suggest that the rate at which the organisation is generating
cash from sales of this product is greater than the rate at which it is incurring
costs, so the product should make a profit. Priority should be given to the
products generating the best ratios.

TPAR<1 would suggest that throughput is insufficient to cover operating


costs, resulting in a loss. Hence, changes need to be made quickly.

35
Illustration 2

From the information given in Illustration 1,

Return / factory hour


X Y
Return / unit 20 22
Labour Hours per unit 1 2
Ret / factory hr 20 11

Which product should be produced first?


X as it has the higher return / factory hour. Any remaining labour hours should
be used on product Y.

Cost / factory hour

Factory costs are assumed to be fixed in the short term. Take all costs
excluding material

Product X (3 + 2 + 1) = 6 x 10,000 = 60,000


Product Y (6 + 2 + 4) = 12 x 15,000 = 180,000
240,000

i.e. Cost / factory hour = $240,000


30,000
= $8 / hr

TPAR X Y

$20 $11
$8 $8
= 2.5 = 1.375

Both products have a TPA ratio greater than 1, i.e. worth producing.

4.2.1.2 Criticisms of TPAR

1. It concentrates on the short-term


2. It is more difficult to apply throughput accounting concepts to the longer
term when all costs are variable
3. In the long run, ABC might be more appropriate for measuring and
controlling performance

36
4.3 ACCA SYLLABUS GUIDE OUTCOME 3:
Suggest how a TPAR could be improved

Management should focus attention upon improving the throughput


accounting ratio. If they can do this then higher levels of profit will be
achieved.

Options to improve the TPAR include: -

1. increase the selling price this will increase throughput per unit
2. reduce material costs per unit this will also increase throughput per
unit
3. reduce total operating expenses this will reduce the total factory costs
4. improve the productivity of the assembly workforce. Therefore, the time
required to make each unit will fall and throughput will increase.

4.4 ACCA SYLLABUS GUIDE OUTCOME 4:


Apply throughput accounting to a multi-product decision-making
problem

Four steps: -

1. calculate the throughput per unit for each product (selling price
material cost)
2. identify the bottleneck constraint
3. calculate the throughput return per hour of bottleneck resource
4. rank the products in order of the priority in which they should be
produced starting with the product that generates the highest return per
hour first
5. calculate the optimum production plan, allocating the bottleneck
resource to each one in order, being sure not to exceed the maximum
demand for any of the products.

37
Lecture Example 3

GMX plc manufactures 2 types of games.

Their cost cards are as follows:

Game 1 Game 2
$ $

Selling price 25 28
Materials 10 18
Labour 4 3
Other Variable Costs 6 3
Fixed Costs 3 23 2 26
Profit $2 $2

Machine hour per unit 2 hrs 1 hr


Maximum demand 20,000 units 10,000 units

The total hours available are 40,000.

Required:

a. Calculate the optimum production plan and the maximum profit


using conventional key factor analysis.

b. Calculate the optimum production plan and the maximum profit, on


the assumption that in the short-term only material costs are
variable (i.e. using a throughput accounting approach).

c. Calculate the Throughput Accounting ratios for both products.

38
Lecture Example 4

Ride Ltd is engaged in the manufacturing and marketing of bicycles. Two


bicycles are produced. These are the Roadster which is designed for use on
roads and the Everest which is a bicycle designed for use in mountainous
areas. The following information relates to the year ending 31 December
2005:

(1) Unit selling price and cost data is as follows:

Roadster Everest
$ $
Selling price 200 280
Material cost 80 100
Variable production conversion costs 20 60

(2) Fixed production overheads attributable to the manufacture of the bicycles


will amount to $4,050,000.

(3) Expected demand is as follows:

Roadster 150,000 units


Everest 70,000 units

(4) Each bicycle is completed in the finishing department. The number of each
type of bicycle that can be completed in one hour in the finishing department
is as follows:

Roadster 6.25
Everest 5.00

There are a total of 30,000 hours available within the finishing department.

(5) Ride Ltd operates a just in time (JIT) manufacturing system with regard to
the manufacture of bicycles and aims to hold very little work-in-progress and
no finished goods stocks whatsoever.

Required:

(a) Using marginal costing principles, calculate the mix (units) of each
type of bicycle which will maximise net profit and state the value of
that profit.

(b) Calculate throughout accounting ratio for each type of bicycle and
briefly discuss when it is worth producing a product where
throughput accounting principles are in operation. Your answer
should assume that the variable overhead cost amounting to

39
$4,800,000 incurred as a result of the chosen product mix in part (a)
is fixed in the short term.

(c) Using throughput accounting principles, advise management of the


quantities of each type of bicycle that should be manufactured which
will maximise net profit and prepare a projection of the net profit that
would be earned by Ride Ltd in the year ending 31 December 2005.

(ACCA Paper 3.3 December 2004 Qs 2)

Further questions

Question 19

A company manufactures a product which requires four hours per unit of


machine time. Machine time is a bottleneck resource as there are only ten
machines which are available for 12 hours per day, five days per week. The
product has a selling price of $130 per unit, direct material costs of $50 per
unit, labour costs of $40 per unit and factory overhead costs of $20 per unit.
These costs are based on weekly production and sales of 150 units.

What is the throughput accounting ratio (to 2 decimal places)?

A. 133
B. 200
C. 075
D. 031

9 Specimen Exam Applicable from December 2014

40
Question 210

An organisation has market demand of 50,000 units for a product that goes
through three processes: cutting, heating and assembly. The total time
required in each process for each product and the total hours available are:

Process Cutting Heating Assembly

Hrs per unit 2 3 4

Total hours available 100,000 120,000 220,000

Which is the bottleneck process?

A. Cutting process
B. Heating process
C. Assembly process

Question 3
Beta Co produces 3 products, E, F and G, details of which are shown below:

Product E F G

$ $ $

Selling price per unit 120 110 130

Direct material cost per unit 60 70 85

Maximum demand (units) 30,000 25,000 40,000

Time required on the bottleneck


resource (hours per unit) 5 4 3

There are 320,000 bottleneck hours available each month.


Required:

Calculate the optimum product mix each month.

1010Throughput Accounting and the Theory of Constraints part 2,


http://www.accaglobal.com/gb/en/student/acca -qual-student-journey/qual-resource/acca-
qualification/f5/technical-articles/throughput-constraints2.html, March 2013

41
CHAPTER 5
ENVIRONMENTAL
ACCOUNTING11
5.1 ACCA SYLLABUS GUIDE OUTCOME 1:
Discuss the issues businesses face in the management of
environmental costs

5.1.1 What is environmental accounting?

Environmental accounting encompasses the provision of environment-related


information both externally and internally. It focuses on reports required for
shareholders and other stakeholders, as well of the provision of management
information.

5.1.2 What is environmental management accounting?

Management accounts give us an analysis of the performance of a business


and are ideally prepared on a timely basis so that we get up-to-date
management information. They break down each of our different business
segments (in a larger business) in a high level of detail. This information is
then used to assess how the business historic performance has been and,
moving forward, how it can be improved in the future.

Environmental management accounting is simply a specialised part of the


management accounts that focuses on things such as the cost of energy and
water and the disposal of waste and effluent. It is a subset of environmental
accounting. It focuses on information required for decision making within the
organisation (internally focused), although much of the information it
generates could also be used for external reporting.

The focus of environmental management accounting is not all on purely


financial costs. It includes consideration of other non-financial matters such as
the costs vs. benefits of buying from suppliers who are more environmentally
aware, or the effect on the public image of the company from failure to comply
with environmental regulations.

11 Prepared using the two articles, Environmental Management Accounting by S. Johnson, Student
Accountant, June 2004
http://www.accaglobal.com/en/student/qualification-resources/acca-qualification/acca-exams/p5-
exams/exams-p54/environmenta-management.html and
Environmental Management Accounting by A. I rons, Student Accountant, Issue 15/2004
http://www.accaglobal.com/content/dam/acca/global/pdf/SA_july2004_F5_EMA.pdf

42
In 1998, the International Federation of Accountants (IFAC) originally defined
environmental management accounting as:

The management of environmental and economic performance through the


development and implementation of appropriate environment-related
accounting systems and practices. While this may include reporting and
auditing in some companies, environmental management accounting typically
involves lifecycle costing, full cost accounting, benefits assessment, and
strategic planning for environmental management.
Environmental management accounting is
Then, in 2001, The United Nations Division for Sustainable Development
(UNDSD) emphasised their belief that environmental management accounting
systems generate information for internal decision making rather than external
decision making.

The UNDSD make what became a widely accepted distinction between two
types of information: physical information and monetary information.
Hence, they broadly defined EMA to be the identification, collection, analysis
and use of two types of information for internal decision making:
physical information on the use, flows and destinies of energy, water
and materials (including wastes)
monetary information on environment-related cost, earnings and
savings.

5.1.3 Defining environmental costs

Many organisations vary in their definition of environmental costs.

A useful cost categorisation is that provided by the US Environmental


Protection Agency in 1998. They stated that the definition of environmental
costs depended on how an organisation intended on using the information.
They made a distinction between four types of costs:

1. conventional costs: raw material and energy costs having


environmental relevance
2. potentially hidden costs: costs captured by accounting systems but
then losing their identity in general overheads
3. contingent costs: costs to be incurred at a future date, e.g. clean up
costs
4. image and relationship costs: costs that, by their nature, are
intangible, for example, the costs of producing environmental reports.

The UNDSD, on the other hand, described environmental costs as comprising


of:

1. costs incurred to protect the environment, e.g. measures taken to


prevent pollution and

43
2. costs of wasted material, capital and labour, i.e. inefficiencies in the
production process.

Neither of these definitions contradict each other; they just look at the costs
from slightly different angles. Hence, definitions of environmental costs vary
greatly, with some being very narrow and some being far wider.

5.1.4 Managing environmental costs its importance

Environmental risks cannot be ignored, they are now as much a part of


running a successful business as product design, marketing, and sound
financial management.

There are three main reasons why the management of environmental costs is
becoming increasingly important in organisations.

1. society as a whole has become more environmentally aware, with


people becoming increasingly aware about the carbon footprint 12 and
recycling taking place now in many countries. Companies are finding
that they can increase their appeal to customers by portraying
themselves as environmentally responsible.

2. environmental costs are becoming huge for some companies,


particularly those operating in highly industrialised sectors such as oil
production. In some cases, these costs can amount to more than 20%
of operating costs. Such significant costs need to be managed.

3. regulation is increasing worldwide at a rapid pace, with penalties for


non-compliance also increasing accordingly. Penalties include fines,
increased liability to environmental taxes, loss in value of land,
destruction of brand values, loss of sales, consumer boycotts, inability
to secure finance, loss of insurance cover, contingent liabilities, law
suits and damage to corporate image.

In the largest ever seizure related to an environmental conviction in the UK, a


plant hire firm, John Craxford Plant Hire Ltd, had to not only pay 85,000 in
costs and fines but also got 1.2m of its assets seized. This was because it
had illegally buried waste and also breached its waste and pollution permits.
And its not just the companies that need to worry. Officers of the company
and even junior employees could find themselves facing criminal prosecution
for knowingly breaching environmental regulations.

In 2010, the failure of a 40-ft-tall blowout preventer has severely damaged


BPs image and wiped out more than $20 billion of the companys stock

12A carbon footprint (as defined by the Carbon Trust) measures the total
greenhouse gas emissions caused directly and indirectly by a person,
organisation, event or product

44
market value. BP claimed it was spending $6 million a day on the salvage
effort. The spill cost BP more than $4.6 billion in containment and clean-up
expenses. The oil spill also affected the Louisiana fishing industr and the
tourism business along the Florida coast13.

By failing to incorporate environmental concerns, organisations are unaware


of the impact on the income statement and statement of financial position of
environment-related activities. Moreover, they miss out on identifying cost
reduction and other improvement opportunities, employ incorrect
product/service pricing, mix and development decisions. This leads to a
failure to enhance customer value, while increasing the risk profile of
investments and other decisions with long-term consequences.

5.1.5 How do organisations control these environmental costs?

It is only after environmental costs have been defined, identified and allocated
that a business can begin the task of trying to control them.
Much of the prepare environmental management accounts
Let us consider an organisation whose main environmental costs are as
follows:
1. waste and effluent disposal
2. water consumption
3. energy
4. transport and travel
5. consumables and raw materials.

1. Waste and effluent disposal

There are lots of environmental costs associated with waste. For example, the
costs of unused raw materials and disposal; taxes for landfill; fines for
compliance failures such as pollution. It is possible to identify how much
material is wasted in production by using the mass balance approach,
whereby the weight of materials bought is compared to the product yield.
From this process, potential cost savings may be identified.

In addition to these monetary costs to the organisation, waste has


environmental costs in terms of lost land resources (because waste has been
buried) and the generation of greenhouse gases in the form of methane.

2. Water

Businesses actually pay for water twice first, to buy it and second, to
dispose of it. If savings are to be made in terms of reduced water bills, it is

13Article, BP Oil Spill Clean-Up to Cost Nearly $5 Billion, Environmental Leader, May 2010
http://www.environmentalleader.com/2010/05/03/bp-oil-spill-clean-up-to-cost-nearly-5-billion/

45
important for organisations to identify where water is used and how
consumption can be decreased.

3. Energy

A recent UK government publicity campaign reports that companies are


spending, on average, an extra 30% on energy through inefficient practices.
With good energy management, we could reduce the environmental impact of
energy production by 30% and slash 30% of the organisations energy
expenditure.

For example, environmental management accounts may help to identify


inefficiencies and wasteful practices and, therefore, opportunities for cost
savings.

4. Transport and travel

Again, environmental management accounting can often help to identify


savings in terms of business travel and transport of goods and materials. At a
simple level, a business can invest in more fuel-efficient vehicles.

5. Consumables and raw materials

These costs are usually easy to identify and discussions with senior managers
may help to identify where savings can be made. For example, toner
cartridges for printers could be refilled rather than replaced. This should
produce a saving both in terms of the financial cost for the organisation and a
waste saving for the environment (toner cartridges are difficult to dispose of
and less waste is created this way).

It is equally important to allocate environmental costs to the processes or


products which give rise to them. Only by doing this can an organisation make
well-informed business decisions.

For example, a pharmaceutical company may be deciding whether to continue


with the production of one of its drugs. In order to incorporate environmental
aspects into its decision, it needs to know exactly how many products are
input into the process compared to its outputs; how much waste is created
during the process; how much labour and fuel is used in making the drug; how
much packaging the drug uses and what percentage of that is recyclable etc.
Only by identifying these costs and allocating them to the product can an
informed decision be made about the environmental effects of continued
production.

46
5.1.6 How should environmental information be reported?

Hansen and Mendoza (1999) stated that environmental costs are incurred
because of poor quality controls. Therefore, they advocate the use of a
periodical environmental cost report that is produced in the format of a cost of
quality report, with each category of cost being expressed as a percentage of
sales revenues or operating costs so that comparisons can be made between
different periods and/or organisations.

The categories of costs would be as follows:

1. Environmental prevention costs: the costs of activities undertaken to


prevent the production of waste.

2. Environmental detection costs: costs incurred to ensure that the


organisation complies with regulations and voluntary standards.

3. Environmental internal failure costs: costs incurred from performing


activities that have produced contaminants and waste that have not
been discharged into the environment.

4. Environmental external failure costs: costs incurred on activities


performed after discharging waste into the environment.

It is clear from the suggested format of this quality type report that Hansen
and Mendozas definition of environmental cost is relatively narrow.

5.1.7 Identifying environmental costs

Much of the information that is needed to prepare environmental management


accounts could actually be found in a business general ledger. A close review
of it should reveal the costs of materials, utilities and waste disposal, at the
least. The main problem is, however, that most of the costs will have to be
found within the category of general overheads if they are to be accurately
identified. Identifying them could be a lengthy process, particularly in a large
organisation.

The management of environmental costs can be a difficult process. This is


because:

1. just as EMA is difficult to define, so too are the actual costs involved.
2. having defined them, some of the costs are difficult to separate out and
identify.
3. the costs can need to be controlled but this can only be done if they
have been correctly identified in the first place.

47
5.2 ACCA SYLLABUS GUIDE OUTCOME 2:
Describe the different methods a business may use to account for its
environmental costs14

Environmental management accounting uses some standard accountancy


techniques to identify, analyse, manage and hopefully reduce environmental
costs in a way that provides mutual benefit to the company and the
environment, although sometimes it is only possible to provide benefit to one
of these parties.

In 2003, the UNDSD identified four management accounting techniques for


the identification and allocation of environmental costs: input/ outflow analysis,
flow cost accounting, activity based costing and lifecycle costing.

1. Input/outflow analysis

This technique records material inflows and balances this with outflows on the
basis that, what comes in, must go out.

Input/output analysis according to Envirowise

The purchased input is regarded as 100% and balanced against the outputs
which are produced, sold and stored goods and the residual (regarded as
waste). Materials are measured in physical unit and include energy and
water.

For example, if 100kg of materials have been bought and only 80kg of
materials have been produced, then the 20kg difference must be accounted
for in some way. It may be that 10% of it has been sold as scrap and 90% of it
is waste. By accounting for outputs in this way, both in terms of physical

14 Examined December 2013 Qs 1c

48
quantities and, at the end of the process, in monetary terms too, businesses
are forced to focus on environmental costs.

2. Flow cost accounting

This technique uses not only material flows but also the organisational
structure. It makes material flows transparent by looking at the physical
quantities involved, their costs and their value. It divides the material flows into
three categories: material, system, and delivery and disposal.

i. The material values and costs apply to the materials which are involved
in the various processes.
ii. The system values and costs are the in-house handling costs which are
incurred to maintain and support material throughput e.g. personnel
costs and depreciation.
iii. The delivery and disposal values and costs refer to the costs of flows
leaving the company, e.g. transport costs or costs of disposing waste.

The values and costs of each of these three flows are then calculated. The
aim of flow cost accounting is to reduce the quantity of materials which, as
well as having a positive effect on the environment, should have a positive
effect on a business total costs in the long run.

3. Activity-based costing

ABC allocates internal costs to cost centres and cost drivers on the basis of
the activities that give rise to the costs. In an environmental accounting
context, it distinguishes between environment-related costs, which can be
attributed to joint cost centres (e.g. incinerators and sewage plants), and

49
environment-driven costs, which tend to be hidden on general overheads, e.g.
increased depreciation or higher cost of staff.

For example, activity-based costing may be used to ascertain more accurately


the costs of washing towels at a gym. The energy used to power the washing
machine is an environmental cost; the cost driver is washing.

Once the costs have been identified and information accumulated on how
many customers are using the gym, it may actually be established that some
customers are using more than one towel on a single visit to the gym. The
gym could drive forward change by informing customers that they need to pay
for a second towel if they need one. Given that this approach will be seen as
environmentally-friendly, most customers would not argue with its
introduction. Nor would most of them want to pay for the cost of a second
towel. The costs to be saved by the company from this new policy would
include both the energy savings from having to run fewer washing machines
all the time and the staff costs of those people collecting the towels and
operating the machines. Presumably, since the towels are being washed less
frequently, they will need to be replaced by new ones less often as well.

In addition to these savings to the company, however, are the all-important


savings to the environment since less power and cotton
(or whatever materials the towels are made from) is now being used, and the
scarce resources of our planet are therefore being conserved. Lastly, the gym
is also seen as an environmentally friendly organisation and this, in turn, may
attract more customers and increase revenues.

4. Lifecycle costing

Within the context of environmental accounting, lifecycle costing (full costing)


is a technique which requires the full environmental consequences, and,
therefore, costs, arising from production of a product to be taken account
across its whole lifecycle, literally from cradle to grave.

One example of the potential gains from using lifecycle costing can be seen in
the case of Xerox Limited.

Xerox Limited, a subsidiary of Xerox Corporation, introduced the concept of


lifecycle costing for its logistic chain. The core business of Xerox Limited is
manufacturing photocopiers, which are leased rather than sold. This means
the machines are returned to Xerox Limited at the end of their lease.
Previously, machines were shipped in a range of different types of packaging,
which could rarely be re-used by customers to return the old copiers. The
customer had to dispose of the original packaging and to provide new
packaging to return the machine at the end of its lease, which in turn could not
be used to re-ship other machines. This meant Xerox lost the original costs
and had to bear the costs of disposal of the packaging.

50
A new system was invented which used a standard pack (tote). Two types of
totes were introduced to suit the entire range of products sold by Xerox. Totes
can be used for both new machines delivery and return carcasses. The whole-
chain cost analysis showed the considerably lower cost of the tote system,
compared to the previously existing system and the supply chain became
more visible. The tote system resulted not only in cost savings but also in
reduced de-pack times and improved customer relations (Bennett and
James, 1998b).

The abovementioned accounting techniques are useful for environmental


management accounting to identify and allocate environmental costs. In
addition, there are alternative techniques to estimate environmental costs
such as the environmental cost decision tree. Also, the undertaking of
environmental audits on a regular basis provides the platform for a successful
programme of total quality management (TQM).

Lecture Example 1

British Petroleums Annual Review describes a number of activities aimed at


reducing the environmental impact of the companys operations.

a. What are the advantages, for a company like BP, of reducing


environmental costs?
b. What are the disadvantages encountered by these companies in
trying to reduce these environmental costs?

Lecture Example 215

The following are types of management accounting techniques:

(i) Flow cost accounting


(ii) Input/output analysis
(iii) Life-cycle costing
(iv) Activity based costing

Which of the above techniques could be used by a company to account for its
environmental costs?

A. (i) only
B. (i) and (ii) only
C. (i), (ii) and (iii) only
D. All of the above

15 Specimen Exam Applicable from December 2014

51
CHAPTER 6
Planning with Limiting Factors
A limiting factor is the factor (aspect of business/resource) that limits an
organisations activities. For many businesses, this may frequently be the
level of sales that can be achieved but at other times a business may be
limited by a shortage of a resource which prevents the business from
achieving its sales potential. Other examples of limiting factors would include:
- supply of skilled labour, supply of materials, factory space, finance, plant
capacity and market demand.

6.1 ACCA SYLLABUS GUIDE OUTCOME 1:


Identify limiting factors in a scarce resource situation and select an
appropriate technique in a scarce resource situation
Determine the optimal production plan where an organisation is
restricted by a single limiting factor, including within the context of
make or buy decisions

A business may face a single constraint situation; however, others may face a
multi constraint scenario.

Planning with one limiting factor

When there is only one scarce resource, key factor analysis can be used to
solve the problem. Options must be ranked using contribution earned per unit
of the scarce resource.

6.1.1 Three steps in key factor analysis

Step 1: - First determine the limiting factor (bottleneck resource)

Step 2: - Rank the options using the contribution earned per unit of the scarce
resource

Step 3: - Allocate resources

52
Illustration 1

Product A Product B

SP / unit 100 120


VC / unit 80 75
FC / unit 10 12
Skilled Labour / unit 0.5 hr 0.75 hr
Demand (units) 5000 4000

Only 4,000 labour hours are available.

How many labour hours are required?

Required: Product A (0.5 x 5000) 2500


Product B (0.75 x 4000) 3000
5500
Available 4000
Shortfall 1500

Labour hours are a limiting factor

Product A Product B

SP / unit 100 120


VC / unit 80 75
Cont / unit 20 45
Lab hrs / unit 0.5 0.75
Cont / lab hr 40 60
Ranking 2 1

Prod Plan

Product Units Lab hrs / unit Total lab hrs

Available = 4000
Product B 4000 0.75 (3000)
1000
Product A 1000 = 2000 0.5 (1000)
0.5 0__

Produce 2000 units Product A


4000 units Product B

53
6.1.2 Assumptions

1. A single quantifiable objective. In reality, there may be multiple


objectives.
2. Each product always uses the same quantity of the scarce resource
per unit.
3. The contribution per unit is constant. However, the selling price may
have to be lowered to sell more; discounts may be available as the
quantity of materials needed increases.
4. Products are independent. It may not be possible to prioritise product A
at the expense of product B.
5. We focus on the short term, therefore ignoring fixed costs.

Lecture Example 1

Dave Ltd manufactures 3 products using the same machinery. Only 8,000
machine hours are available each month.

The following information is available: -

Product A Product B Product C

SP / unit ($) 40 45 60
VC / unit ($) 15 18 25
Machining mins / unit 60 40 30
Monthly demand 6,000 9,000 3,000

Required:

a. Calculate the monthly machine hours required to meet all demand


for products A, B and C. Hence, calculate any surplus or shortfall in
machine hours.

b. Calculate the number of units of each product which Dave Ltd


should produce in order to maximize its profit.

c. Calculate the monthly contribution that this production mix should


yield.

6.2 ACCA SYLLABUS GUIDE OUTCOME 2:


Formulate and solve a multiple scare resource problem both
graphically and using simultaneous equations as appropriate

When there are two or more resources in short supply, linear programming is
required to find the solution.

54
Linear programming is used to: -

1. maximise contribution and/or


2. minimise costs

6.2.1 Steps involved in linear programming

1. Define the variables


2. Define and formulate the objective
3. Formulate the constraints
4. Draw a graph identifying the feasible region. The constraints are
represented as straight lines on the graph. The feasible region shows
those combinations of variables which are possible given the resource
constraints.
5. Solve for the optimal production plan. An iso-contribution line (an
objective function for a particular value) must be drawn. All points on
this line represent an equal contribution. This line must move to and
from the origin in parallel. The objective is to get the highest
contribution or the minimum cost within the binding constraints.

A linear programming situation can be solved using simultaneous equations.


However, this technique should be used after one has determined graphically
the constraints and the feasible region.

6.2.2 Linear programming assumptions

1. a single quantifiable objective


2. each product always uses the same quantity of the scarce resource per
unit.
3. the contribution (or cost) per unit is constant for each product,
regardless of the level of activity. Therefore, the objective function is a
straight line.
4. products are independent
5. the focus is short-term
6. all costs either vary with a single volume-related cost driver or they are
fixed for the period under consideration.

Illustration 2

A company manufactures two types of boxes, Box A and Box B.

Contribution / box A = $20 Contribution / box B = $45

Two materials, X and Y are used in the manufacturing of each box. Each
material is in short supply.

55
Material X = 3,000 kg available Material Y = 2,700 kg available

Each box A uses 20 kg of Material X and 15 kg of Material Y


Each box B uses 30 kg of Material X and 50 kg of Material Y
The objective of this company is to maximize profits.

Variables

Let A be the number of boxes of A produced and sold


Let B be the number of boxes of B produced and sold

Objective function

Max C = 20A + 45B

Constraints

Mat X = 20A + 30B 3,000


Mat Y = 15A + 50B 2,700

Non-negativity = A,B 0

Material X

20A + 30B = 3,000


When A is 0, B = 100
When B is 0, A = 150

Material Y

15A + 50B = 2700


When A = 0, B = 54
When B = 0, A = 180

Product A

200 OPQR = feasible region


180
150 P B
Q
100
A
45 Material X
Iso-cont. Line
0 Material Y
O 25 50 R 75 100 Product B

56
How can we draw the iso-contribution line?

20A + 45B = 900 take a number which is a multiple of both the 20 and
the 45

when A is 0, B = 20
when B is 0, A = 45

Which is the optimal point?

1. Shift out the objective function.


2. We need to maximize profits. Hence the optimal point is the last point to
reach in the feasible region, OPQR

Therefore the optimal point in the above example is B.

How can I find the optimal point using simultaneous equations?

1. Find the contribution at each point on the feasible region.

At point O, contribution is 0

At point P, contribution is 20A + 45B


20(150) + 45(0) = 3,000

At point R, contribution is 20A + 45B


20(0) + 45(54) = 2430

At point Q point of intersection of material X, material Y.

Material X = 20A + 30B = 3,000 x5


Material Y = 15A + 50B = 2,700 x3

100A + 150B = 15,000


45A + 150B = 8,100
55A = 6,900
A = 125.45

100(125.45) + 150B = 15,000


B = 16.36

Contribution = 20A + 45B


= 20(125.45) + 45(16.36)
= 2509 + 736.2
= 3245.2

Therefore point Q is the optimal point where contribution is $3245.20

57
Lecture Example 2

(Article by Mr Geoff Cordwell Linear Programming)16

A profit-seeking firm has two constraints: labour, limited to 16,000 hours, and
materials, limited to 15,000kg. The firm manufactures and sells two products,
X and Y. To make X, the firm uses 3kg of material and 4 hours of labour,
whereas to make Y, the firm uses 5kg of material and 4 hours of labour. The
contributions made by each product are $30 for X and $40 for Y. The cost of
materials is normally $8 per kg, and the labour rate is $10 per hour.

Required: -

a. Write down the objective function and the constraints for this firm.
b. Draw a graph to illustrate all the constraints, shading the feasible
region.
c. Find the optimal solution if the company aims to maximize
contribution. Calculate the contribution gained at this optimal
point.
d. Find the optimal point using simultaneous equations. How much
is contribution at each point on the feasible region?

6.3 ACCA SYLLABUS GUIDE OUTCOME 3:


Explain and calculate shadow prices (dual prices) and discuss their
implications on decision-making and performance management

Any scarce resource that is fully utilised in the optimal solution will have a
shadow price. It would be worth paying more than the normal price to obtain
more of the scarce resource because of the contribution foregone by not
being able to satisfy the sales demand. Therefore, if more critical (scarce)
resource becomes available, then the feasible region would tend to expand
and this means that the optimal point would tend to move outward away from
the origin, thus earning more contribution.

Hence the shadow price of a binding constraint is the amount by which the
total contribution would increase if one more unit of the scarce resource
became available.

Management can use the shadow price as a measure of how much they
would be willing to pay to gain more of a scarce resource over and above the
normal price subject to any non-financial issues that may be present.

If the availability of a non-critical scarce resource increased then the feasible


region would not tend to expand and therefore no more contribution could be

16 http://www.accaglobal.com/content/dam/acca/global/pdf/sa_mar08_cordwell.pdf

58
earned. In this case, extra non-critical scarce resource has no value and a nil
shadow price.

6.3.1 Calculating shadow prices

1. add one unit to the constraint concerned while leaving the other critical
constraint unchanged
2. solve the revised simultaneous equations to derive a new optimal
solution
3. calculate the revised optimal contribution and compare to the old
contribution. The increase in contribution is the shadow price for the
constraint under consideration.

Illustration 3

Following from illustration 2, find the shadow price of Material X.

20A + 30B = 3001 (add 1 kg to Material X)


15A + 50B = 2700

20A + 30B = 3001 .. x5


15A + 50B = 2700 .. x3

100A + 150B = 15005


45A + 150B = 8100
55A / = 6905
A = 125.55

100 A + 150B = 15005


100(125.55) + 150B = 15005
B = 16.33

Therefore New Contribution = 20 (125.55) + 45 (16.33)


= 2511+ 734.85
= 3245.85
Old Contribution = 3245.20
Shadow price = 0.65

Extra contribution / kg of Material X

59
6.4 ACCA SYLLABUS GUIDE OUTCOME 4:
Calculate slack and explain the implications of the existence of slack
for decision-making and performance management (excluding simplex
and sensitivity to changes in objective functions)

Slack occurs when the maximum availability of a resource is not used.


Therefore, slack is the amount by which a resource is under-utilised. It will
occur when the optimum point does not fall on the given resource line.

If, at the optimal solution, the resource used equals the resource available, the
constraint is binding and there is no slack. Hence, a shadow price has to be
calculated.

Lecture Example 3

Using the information from lecture example 2, calculate the shadow price of
both the materials and the labour.

Lecture Example 4

Using the information from lecture examples 2 and 3, calculate the amount of
extra material which should be bought in.

What happens if the objective is to minimize costs?

1. Same steps as above

2. Check the constraints :- are they less than or greater than?

3. If they are greater than, the region which you should consider is above
the constraint.

4. The optimal point will be the first point you reach on the feasible region
when you shift out the iso-cost function.

Lecture Example 5

A linear programming model has been formulated for two products, X and Y.
The objective function is depicted by the formula C = 5X+6Y, where C =
contribution, X = the number of product X to be produced and Y= the number
of product Y to be produced.

Each unit of X uses 2 kg of material Z and each unit of Y uses 3 kg of material


Z. The standard cost of material Z is $2 per kg. The shadow price for material
Z has been worked out and found to be $2.80 per kg.

60
If an extra 20 kg of material Z becomes available at $2 per kg, what will the
maximum increase in contribution be?

A. Increase of $96
B. Increase of $56
C. Increase of $16
D. No change

Further questions

Question 1

Pegs plc manufactures 2 products, Pega and Pegi.

The cost cards are as follows.


Pega Pegi

Selling price 25 28

Materials 8 20
Labour 5 2
Other variable costs 7 2
Fixed costs 3 2
23 26

Profit $2 $2
Machine hours per unit 2 hrs 1 hr
20,000 10,000
Maximum demand units units

The total hours available are 48,000.

Calculate the optimum production plan and the maximum profit using
conventional key factor analysis.

Question 2

A company manufactures and sells two products (X and Y) both of which


utilize the same skilled labour. For the coming period, the supply of skilled
labour is limited to 2,000 hours. Data relating to each product are as follows:

61
Product X Y

Selling price per unit $20 $40


Variable cost per unit $12 $30
Skilled labour hours per unit 2 4
Maximum demand (units) per
period 800 400

In order to maximize profit in the coming period, how many units of each
product should the company manufacture and sell?

A. 200 units of X and 400 units of Y


B. 400 units of X and 300 units of Y
C. 600 units of X and 200 units of Y
D. 800 units of X and 100 units of Y

Question 3

A company uses limiting factor analysis to calculate an optimal production


plan given a scarce resource.

The following applies to the three products of the company:

Product i ii iii
$ $ $
Direct materials (@ $6/kg) 36 24 15
Direct labour (@ $10/hour) 40 25 10
Variable overheads (@ $2/hour) 8 5 2

84 54 27

Maximum demand (units) 2,000 4,000 4,000


Optimal production plan 2,000 1,500 4,000

How many kg of material were available for use in production?

A. 15,750 kg
B. 28,000 kg
C. 30,000 kg
D. 38,000 kg

62
Question 4

Which of the following is correct?

A. When considering limiting factors the products should always be


ranked according to contribution per unit sold.
B. If there is only one scarce resource linear programming should be used
C. In linear programming the point furthest from the origin will always be
the point of profit maximization
D. The slope of the objective function in profit maximization depends on
the contributions of the products.

Question 5

A company manufactures and sells two products (X and Y) which have a


contribution per unit of $8 and $20 respectively.

The company aims to maximize profit. Two materials (G and H) are used in
the manufacture of each product. Each material is in short supply 1,000 kg
of G and 1,800 kg of H are available next period. The company holds no
inventories and it can sell all the units produced.

The management accountant has drawn the following graph accurately


showing the constraints for materials G and H.

Product Y
(units)

100 Material G
90

Material H

0 Product X
125 150 (units)

a. What is the amount (in kg) of material G and material H used in each unit of
Product Y?

63
Material G Material H
A. 10 20
B. 10 10
C. 20 20
D. 20 10

b. What is the optimal mix of production (in units) for the next period?

Product X Product Y
A. 0 90
B. 50 60
C. 60 50
D. 125 0

Question 617

Highfly Co manufactures two products, X and Y, and any quantities produced


can be sold for $60 per unit and $25 per unit respectively.

Variable costs per unit of the two products are as follows:

Product X Product Y
$ $
Materials (at $5 per kg) 15 5
Labour (at $6 per hour) 24 3
Other variable costs 6 5

Total 45 13

Next month, only 4,200 kg of material and 3,000 labour hours will be
available. The company aims to maximise its profits each month.

The company wants to use the linear programming model to establish an


optimum production plan. The model considers x to be number of units of
product X and y to be the number of units of product Y.

Which of the following objective functions and constraint statements (relating


to material and labour respectively) is correct?

Objective function Material constraint Labour constraint


A. 60x + 25y 3x + y 4,200 4x + 05y 3,000
B. 15x + 12y 3x + y 4,200 4x + 05y 3,000
C. 15x + 12y 3x + y 4,200 4x + 05y 3,000
D. 60x + 25y 3x + y 4,200 4x + 05y 3,000

17 Specimen Exam Applicable from December 2014

64
CHAPTER 7
Relevant Costing, Make-or-buy
and other short-term decisions
7.1 ACCA SYLLABUS GUIDE OUTCOME 1:
Explain the concept of relevant costing

Companies and government bodies have increasingly tended to concentrate


on their core competences what they are really good at and turn other
functions over to specialist contractors. This is known as outsourcing or sub-
contracting.

Any short term decisions should be approached using relevant costing


principles.

Relevant costs and revenues are future cash flows arising as a direct
consequence of a decision.

1. Relevant costs are revenues are future costs and revenues


2. Relevant costs and revenues are cash flows
3. Relevant costs and revenues are incremental costs and revenues.

Decision making should be based on relevant costs and revenues.

1. Relevant costs are future costs. A decision is about the future and it
cannot alter what has been done already. Costs that have been
incurred in the past are totally irrelevant to any decision that is being
made 'now'. Such costs are called past costs or sunk costs and are
irrelevant.

2. Relevant costs are cash flows. Only cash flow information is required.
This means that costs or charges which do not reflect additional cash
spending (such as depreciation and notional costs) should be ignored
for the purpose of decision making.

3. Relevant costs are incremental costs and it is the increase in costs and
revenues that occurs as a direct result of a decision taken that is
relevant. Common costs can be ignored for the purpose of decision
making.

4. Avoidable costs are costs which would not be incurred if the activity to
which they relate did not exist. Therefore, they are relevant to a
decision.

65
5. Committed costs are future costs that cannot be avoided because of
decisions that have already been made. These are non-relevant costs.

7.2 ACCA SYLLABUS GUIDE OUTCOME 2:


Explain and apply the concept of opportunity costs

Where the choice of one course of action requires that an alternative course
of action is given up, the financial benefits that are forgone or sacrificed are
known as opportunity costs. Opportunity costs represent the lost contribution
to profits arising from the best use of the alternative forgone. Opportunity
costs only arise when resources are scarce and have alternative uses.

Lecture Example 1

RCA which manufactures and sells one single product is currently operating
at 85% of full capacity, producing 102,000 units per month. The current total
monthly costs of production amount to $330,000, of which $75,000 are fixed
and are expected to remain unchanged for all levels of activity up to full
capacity.

A new potential customer has expressed interest in taking regular monthly


delivery of 12,000 units at a price of $2.80 per unit.

All existing production is sold each month at a price of $3.25 per unit. If the
new business is accepted, existing sales are expected to fall by 2 units for
every 15 units sold to the new customer.

Required: -

What is the overall increase in monthly profit which would result from
accepting the new business?

Relevant Costs for Materials

The relevant cost of raw materials is generally their current replacement cost,
unless the materials have already been purchased and would not be replaced
once used. In this case the relevant cost of using them is the higher of: -

Their current resale value


The value they would obtain if they were put to an alternative use

If the materials have no resale value and no other possible use, then the
relevant cost of using them for the opportunity under consideration would be
nil.

66
Not
Not in
in stock
stock In
In stock
stock

Used
Used No
No other
other Scarce
Scarce
regularly
regularly use
use
Needs
Needstotobe
be Wont
Wont Cant
Cant
Buy
Buyit!
it! replaced
replaced be replaced
be replaced replace
replace

Current
Current Current
Current Current
Current Opportunity
Opportunity
replacement
replacement replacement
replacement Resale
Resale Cost
Cost
cost
cost cost
cost value
value

Illustration 1

Qty needed Qty currently Original Cost Current Current


Material for contract in inventory of qty in inv. Purch price Resale price

A 400 kg 200 kg $10 / kg $15 / kg $12 / kg


B 200 kg 100 kg $20 / kg $22 / kg $15 / kg

Material A is used regularly in the business.


Material B is no longer used and has no alternative use in the business.

The relevant cost of material is

Material A regularly used replace


400kg x $15 = $6000

Material B 100kgs in stock could have been sold if not used in the contract
opportunity cost = 100kg x $15 = $1500

The other 100kg have to be purchased at $22


100kg x $22 = $2200

Therefore $1500+$2200 = $3700

Please note that the original cost is a sunk cost, therefore irrelevant.
Relevant Costs for Labour

67
The key question here is: Is there spare capacity?

If there is spare capacity within a department, additional work can be


undertaken at no extra cost. Therefore, the relevant cost of labour is nil.

If a department is working in full capacity, additional work can be undertaken


only in two circumstances: -

a. Hire more employees the relevant cost of labour will be the


current rate of pay given to these employees
b. Shift work from another department in this case, the relevant cost
of labour will be the lost contribution from not producing the
alternative product/not working on the other work. This can be
calculated by taking the sales value of the units of the product which
will now be forgone and deduct the cost of producing them but
excluding the labour cost. Workers will still have to be paid even if
they are working on another product.

Another method is: - calculate the lost contribution (selling price less
all variable costs) plus the wages paid to the workers working on
the new product/work.

Spare
Spare Capacity
Capacity Full
Full Capacity
Capacity

Additional
Additionalwork
workcannot
cannot
Additional
Additionalwork
workcan
can
be be undertaken
undertaken at
be undertaken atno
noextra
extracost
cost be undertaken

Hire
Hiremore
moreemployees
employees Shift
Shiftwork
workfrom
fromanother
anotherdept
dept

Current Lost
Lost
Nil Current Contribution
Nil rate of
rate ofpay
paygiven
given Contribution++
Variable
Variablecost
cost

Illustration 2

68
A contract requires 500 hours of labour. There are 400 hours of spare labour
capacity. The remaining hours can be worked as overtime at time and a half.
Labour rate is $12/hr.

500 hours

400 hrs 100 hrs

Rel. Cost = 0 Rel. Cost = 100 x $12 x 1.5


= $1800

Lecture Example 218

To make a special order, labour will have to be transferred from the


production of Product X, which earns a contribution of $24 per unit made.

Each unit of Product X requires 0.5 hours of labour, which is paid at $24 per
hour. The special order will require 100 hours of labour and 500 hours of
machine time.

The variable cost of running the machine is $30 per hour.

What are the total relevant costs for labour and machine time that should be
included in the cost of the special order?

A. $22,200
B. $10,200
C. $19,800
D. $17,400

Lecture Example 319

B Co. operates a production process which generates a contribution of $4 per


hour. Wages are paid at $7 per hour and labour is fully utilised. During busy
periods workers are offered the chance to work overtime, which is paid at $10
per hour. However, workers are currently refusing to work overtime because
of an industrial dispute.

B Co has just received an additional order which must be fulfilled immediately


which will require 10 hours of labour to fulfil.

18 F5 December 2015 Examiners Report


19 F5 September 2015 Examiners Report

69
What is the total relevant cost of labour for the additional order?

A. $11
B. $40
C. $100
D. $110

7.3 ACCA SYLLABUS GUIDE OUTCOME 3:


Explain the issues surrounding make vs. buy and outsourcing decisions

7.3.1 Make versus Buy

When assessing the differences in costs between making a product in-house


or outsourcing, a key consideration is whether spare capacity does or would
exist.

If there is spare production capacity available the following issues arise:


1. Production resources may be idle if the component is purchased from
outside.
2. The fixed costs of those resources are irrelevant to the decision in the
short term as they will be incurred whether the component is made or
purchased.
3. Purchase would be recommended only if the buying price was less
than the variable costs of internal manufacture.
4. In the long term, however, the business may dispense with or transfer
some of its resources and may purchase from outside if it thereby
saves more than the extra cost of purchasing.

If there is not spare capacity available, the following issues arise:


1. A decision to make components in-house might displace the
manufacture of other existing products. This could give rise to
opportunity costs of lost contribution or additional costs of buying in
those products (if cheaper).
2. In the longer term, management may look to other alternatives, such as
capital expenditure of machinery.

7.3.2 Outsourcing

Matters to be considered before deciding to outsource: -

1. Quality The quality of the outside producer must be acceptable.


2. Continuity of supply must be guaranteed.
3. If a component is no longer produced by the company, the
management should investigate whether the capacity freed up can be
used to generate additional profits from a different product.

70
4. Management should consider whether labour morale will be adversely
affected.
5. Stability in pricing is also very important.

7.4 ACCA SYLLABUS GUIDE OUTCOME 4:


Calculate and compare make costs with buy-in costs
Compare in-house costs and outsource costs of completing tasks and
consider other issues surrounding this decision

Further questions

Lecture Example 420

A business makes two components which it uses to produce one of its


products. Details are:

Component A Component B
Per unit information: $ $
Buy in price 14 17

Material 2 5
Labour 4 6
Variable overheads 6 7
General fixed overheads 4 3

Total absorption cost 16 21

The business wishes to maximise contribution and is considering whether to


continue making the components internally or buy in from outside.

Which components should the company buy in from outside in order to


maximise its contribution?

A. A only
B. B only
C. Both A and B
D. Neither A nor B

7.5 ACCA SYLLABUS GUIDE OUTCOME 5:


Apply relevant costing principles in situations involving shut down, one-
off contracts and the further processing of joint products

20 Specimen Exam Applicable from December 2014

71
7.5.1 Shut down decisions

In evaluating whether to close part of a business, the cost accountant must


consider:
1. loss of contribution from the segment
2. savings in specific fixed costs from closure
3. penalties resulting from the closure, e.g. redundancy, compensation to
customers
4. alternative use for resources released
5. knock-on impact, e.g. supermarkets often stock some goods which
they sell at a loss. This is to get customers through the door, who they
then hope will purchase other products which have higher profit
margins for them.

7.5.2 One-off contracts

A business should identify the incremental cash flows associated with a new
one-off contract/project.

Lecture Example 5
(Article, Relevant Costs for Decision-Making, by B.Jay, Student Accountant, 2004)
http://www.accaglobal.com/student_accountant/archive/2004/47/1163453

The managing director of Parser Limited, a small business, is considering


undertaking a one-off contract. She has asked her inexperienced accountant
to advise on what costs are likely to be incurred so that she can price at a
profit. The following schedule has been prepared:

Costs for special order Notes $


Direct wages 1 28,500
Supervisor costs 2 11,500
General overheads 3 4,000
Machine depreciation 4 2,300
Machine overheads 5 18,000
Materials 6 34,000
98,300

Notes

1. Direct wages comprise the wages of two employees, particularly skilled


in the labour process for this job. They could be transferred from

72
another department to undertake the work on the special order. They
are fully occupied in their usual department and sub-contracting staff
would have to be brought in to undertake the work left behind.
Sub-contracting costs would be $32,000 for the period of the work.
Other sub-contractors who are skilled in the special order techniques
are also available to work on the special order. The costs associated
with this would amount to $31,300.

2. A supervisor would have to work on the special order. The cost of


$11,500 is made up of $8,000 normal payments plus a $3,500
additional bonus for working on the special order. Normal payments
refer to the fixed salary of the supervisor. In addition, the supervisor
would lose incentive payments in his normal work amounting to $2,500.
It is not anticipated that any replacement costs relating to the
supervisors' work on other jobs would arise.

3. General overheads comprise an apportionment of $3,000 plus an


estimate of $1,000 incremental overheads.

4. Machine depreciation represents the normal period cost, based on the


duration of the contract. It is anticipated that $500 will be incurred in
additional machine maintenance costs.

5. Machine overheads (for running costs such as electricity) are charged


at $3 per hour. It is estimated that 6,000 hours will be needed for the
special order. The machine has 4,000 hours available capacity. The
further 2,000 hours required will mean an existing job is taken off the
machine resulting in a lost contribution of $2 per hour (before
overheads are charged).

6. Materials represent the purchase costs of 7,500kg bought some time


ago. The materials are no longer used and are unlikely to be wanted in
the future except for the special order. The complete stock of materials
(amounting to 10,000kg), or part thereof, could be sold for $4.20 per
kg. The replacement cost of material used would be $33,375.

Because the business does not have adequate funds to finance the special
order, a bank overdraft of $20,000 would be required for the project duration
of three months. The overdraft would be repaid at the end of the period. The
company uses a cost of capital of 20% to appraise projects. The bank's
overdraft rate is 18%.

The managing director has heard that for special orders such as this, relevant
costing should be used that also incorporates opportunity costs. She has
approached you to create a revised costing schedule based on relevant
costing principles.

73
Required:
Produce a revised costing schedule for the special project based on
relevant costing principles. Fully explain and justify each of the costs
included in the costing schedule.

7.5.3 Joint product further processing decisions 21

Joint products are two or more products which are output from the same
processing operation, but which are indistinguishable from each other up to
their point of separation.

Joint products have a substantial sales value. Often they require further
processing before they are ready for sale. Joint products arise, for example, in
the oil refining industry where diesel fuel, petrol, paraffin and lubricants are all
produced from the same process.

Inputs Joint Products

Direct Material -------- Product A


Direct Labour Process ------ Product B
Overheads:- Fixed & Variable -------- Product C

By-product Scrap Waste

7.5.3.1 What is the split off point?

The specific point at which individual products become identifiable is known


as the split off point.

21 Examined December 2013 Qs 1

74
Costs incurred before the split off point (joint or pre-separation costs) must be
shared between joint products produced (e.g. for inventory valuation
purposes).

After separation products may be sold immediately or may be processed


further. Any post-separation costs (further processing costs) are allocated
directly to the product on which they are incurred.

7.5.3.2 Should we process a product further?

Incremental profit is the correct basis for justifying further processing.

We should take into consideration: -


1. Any difference in revenue.
2. Any extra costs both further processing costs and any differences in
selling costs.

Pre-separation costs are sunk at this stage and thus not relevant to the
decision.

Lecture Example 6

Fragrance Co jointly processes a specialty chemical that yields two perfumes:


50 litres of Love and 150 litres of Passion. The sales values per litre at split-off
are $6 for Love and $4 for Passion. The joint costs incurred up to the split-off
point are $880. The manager has the option of further processing 150 litres of
Passion to yield 100 litres of Romance. The total additional costs of
converting Passion into Romance would be $160, and the selling price per
litre of Romance would be $8.

Should Fragrance Co further process Passion into Romance?

Further Questions

Question 1

Krol plc uses a standard costing system to control its costs. In the most recent
month its cost accountant has reported a large adverse direct material usage
variance. An initial investigation has shown that the variance is caused by a
faulty machine.

The production manager is trying to decide whether to close down the


production line for one day to allow engineers to perform emergency
maintenance work that could rectify the problem. Past experience of

75
investigating raw material usage variances suggests that there is a 70%
chance of correcting the fault. If the emergency maintenance work is not
carried out now it is estimated that extra material costs of $60,000 per month
for the next six months will be incurred. After this time the problem will
definitely be corrected by scheduled maintenance work during the companys
annual shut down.

Two maintenance engineers would be required to carry out the emergency


maintenance work. Maintenance engineers are paid $25,000 per annum and
each engineer works for 250 days each year. There is currently surplus
capacity in the maintenance department. The emergency maintenance would
use parts costing $10,000. These parts would have to be replaced again
during the scheduled annual maintenance. Emergency maintenance would
involve stopping production for a day resulting in lost production with an
estimated sales value of $160,000, direct material cost of $45,000 and direct
labour cost of $90,000. Direct labour would continue to be paid during the
one-day stoppage.

In this time the otherwise idle labour would be used to repaint the factory,
saving $7,000 in outside painting contractor costs. Krol carries no finished
goods stocks and is currently unable to satisfy demand for its product.

Required:

Using relevant cost principles, calculate whether the emergency


maintenance should be performed.

(CAT T7 December 2006 Qs 4 Part)

Answer

Expected benefit of investigation


Saved material costs $60,000 x 6 months x 70% = 252,000

Costs of the investigation


Engineers salaries (irrelevant as there is surplus capacity) 0
Parts (10,000)
Lost sales (160,000)
Saved direct material cost 45,000
Saved painting costs 7,000

76
Net benefit $134,000

The investigation shows a net benefit of $134,000 and it should be


carried out.

Question 2

Hard Times Ltd is evaluating a proposed expenditure on an item of


equipment that would cost $200,000. A technical feasibility study has
been carried out by consultants, at a cost of $25,000. The equipment
would have a life of four years, and annual profits, after deducting annual
depreciation of $30,000 would be $8,000.

Identify the cash flows that should be evaluated for this project.

Question 3

A machine was purchased for $20,000 and has an estimated life of five years.
It is not currently being used in the business.

A special order has now been received from a customer which would require
the use of the machine for five months. The current net realizable value of the
machine is $10,000. If it is used for the job, its value is expected to fall to
$7,000. The machine has a carrying value of $6,000.

Due to this special order, costs will increase by $50/month.

Determine the relevant cost of using the machine for the special order.

Question 4

A company purchased a machine several years ago for $50,000. its written
down value is now $10,000. The machine is no longer used on normal
production work and it could be sold now for $8,000. A project is being
considered which would make use of this machine for six months. After this
time the machine would be sold for $5,000.

What is the relevant cost of the machine to the project?

A. $2,000
B. $3,000

77
C. $5,000
D. $10,000

Question 5

The following statements relate to relevant cost concepts in decision-making:

i. Materials can never have an opportunity cost whereas labour can


ii. The annual depreciation charge is not a relevant cost
iii. Fixed costs would have a relevant cost element if a decision causes a
change in their total expenditure

Which statements are correct?

A. (i) and (ii) only


B. (i) and (iii) only
C. (ii) and (iii) only
D. (i), (ii) and (iii)

Question 6

A contract is under consideration which requires 600 labour hours to


complete. There are 350 hours of spare labour capacity for which the workers
are still being paid the normal rate of pay. The remaining hours for the
contract can be found either by weekend overtime working paid at double the
normal rate of pay or by diverting labour from other production. This other
production makes a contribution, net of labour cost, of $5 per hour. The
normal rate of pay is $9 per hour.

What is the total relevant cost of labour for the contract?

A. $1,250
B. $3,500
C. $4,500
D. $4,900

Question 7

A company requires 600 kg of raw material Z for a contract it is evaluating. It


has 400 kg of material Z in stock which were purchased last month. Since
then the purchase price of material Z has risen by 8% to $27 per kg. Raw
material Z is used regularly by the company in normal production.

78
What is the total relevant cost of raw material Z to the contract

A. $15,336
B. $15,400
C. $16,200
D. $17,496

Question 8

A company is evaluating a project that requires 4,000 kg of a material that is


used regularly in normal production. 2,500 kg of the material, purchased last
month at a total cost of $20,000 are in stock. Since last month the price of the
material has increased by 2%.

What is the total relevant cost of the material for the project?

A. $12,300
B. $20,500
C. $32,300
D. $32,800

Question 922

A company has received a special order for which it is considering the use of
material B which it has held in its inventory for some time. This inventory of
945 kg was bought at $450 per kg. The special order requires 1,500 kg of
material B. If the inventory is not used for this order, it would be sold for $275
per kg. The current price of material B is $425 per kg.

What is the total relevant cost of material B for the special order?

A. $4,95750
B. $6,375
C. $4,125
D. $6,61125

Question 10

A machine is no longer used by a company. It could be sold now for net


proceeds of $300. Its only other use is on a short-term contract which is under
consideration. The variable running costs of the machine during the period of
the contract would be $400. On completion of the contract the machine would
have no realizable value and would cost $150 to dismantle and remove.

22 Specimen Exam Applicable from December 2014

79
What is the total relevant cost of using the machine on the contract?

A. $450
B. $550
C. $700
D. $850

Question 11

The Western is a local government organisation responsible for waste


collection from domestic households. The new management accountant of
The Western has decided to introduce some new forecasting techniques to
improve the accuracy of the budgeting. The next budget to be produced is for
the year ended 31 December 2010.

Waste is collected by the tonne (T). The number of tonnes collected each year
has been rising and by using time series analysis the new management
accountant has produced the following relationship between the tonnes
collected (T) and the time period in question Q (where Q is a quarter number.
So Q = 1 represents quarter 1 in 2009 and Q = 2 represents quarter 2 in 2009
and so on)

T = 2,000 + 25Q

Each quarter is subject to some seasonal variation with more waste being
collected in the middle quarters of each year. The adjustments required to the
underlying trend prediction are:

Quarter Tonnes
1 -200
2 +250
3 +150
4 -100
Once T is predicted the new management accountant hopes to use the values
to predict the variable operating costs and fixed operating costs that The
Western will be subjected to in 2010. To this end he has provided the
following operating cost data for 2009.

Volume of Waste Total operating cost in 2009


(fixed + variable)
Tonnes $000s
2,100 950
2,500 1,010
2,400 1,010
2,300 990

80
Inflation on the operating cost is expected to be 5% between 2009 and 2010.

The regression formula is shown on the formula sheet.

Required:

(a) Calculate the tonnes of waste to be expected in the calendar year 2010.
(4 marks)

(b) Calculate the variable operating cost and fixed operating cost to be
expected in 2010 using regression analysis on the 2009 data and allowing for
inflation as appropriate.
(10 marks)

Many local government organisations operate incremental budgeting as one


of their main budgeting techniques. They take a previous periods actual
spend, adjust for any known changes to operations and then add a % for
expected inflation in order to set the next periods budget.

(c) Describe two advantages and two disadvantages of a local government


organisation funded by taxpayers money using incremental budgeting as its
main budgeting technique.
(6 marks)

(Acca Paper December 2009 Question 3)

81
CHAPTER 8
Pricing Decisions
Different Types of Market Structures

The price that a business can charge for its products or services will be
determined by the type market in which it operates.

In a perfectly competitive market, the firm is a price taker, i.e. it takes its
price from the industry. No market participant influences the price of the
product it buys or sells.

Perfectly competitive markets exhibit the following characteristics:

1) There are no barriers to entry into or exit out of the market.


2) There is perfect knowledge, with no information failure or time lags.
Knowledge is freely available to all participants, which means that risk-
taking is minimal and the role of the entrepreneur is limited.
3) Firms produce homogeneous, identical, units of output that are not
branded.
4) Each unit of input, such as units of labour, are homogenous.
5) No single firm can influence the market price, or market conditions. The
single firm is said to be a price taker, taking its price from the whole
industry.
6) There are a very large numbers of firms in the market.
7) There is no need for government regulation, except to make markets
more competitive.
8) There are assumed to be no externalities, that is no external costs or
benefits.
9) Firms can only make normal profits in the long run, but they can make
abnormal profits in the short run.

Imperfect competition refers to the market structure that does not meet the
conditions of a perfect competition. Its forms include:

Monopoly: there is only one provider of a good or service. The


monopolist sells a product for which there are no close substitutes. It
controls the market and has great market power. It can set the price of
products sold in the market.

Oligopoly: a few large companies dominate the market and are inter-
dependent. They offer the same product and compete for market
dominance eg. Telecommunication costs.

82
Monopolistic competition: products are similar, but not identical.
Each firm sells a branded product, hence it is a monopolist for its
brand. There is freedom of entry or exit into the industry.

8.1 ACCA SYLLABUS GUIDE OUTCOME 1:


Explain the factors that influence the pricing of a product or service

It is a fundamental rule of business that selling price must be greater than


cost. The most basic point about pricing is that in the long run, the revenue
earned from selling a product or service must exceed the costs incurred.

What influences the price of a product or a service?

1. Cost: - The most straightforward approach to arrive at the selling price is to


calculate cost and add on a profit margin/mark up. Using this cost plus pricing
approach, cost is calculated on the basis of either marginal (this may also be
termed variable) cost or total cost.

2. Price perception: - Customer-based pricing reflects customers


perceptions of the benefits they will enjoy if they obtain that product or service.
Quality is an important aspect of price perception.

3. Competition: - Competition-based pricing means setting a price based


upon the prices of competing products, i.e. taking into consideration both
substitutes and complimentary products.

However, it is important to remember that in many instances, the price will be


dictated by the market. If the product or service is provided by a number of
competitors, and it is not possible to differentiate between different providers,
organisations will have little influence on prices.

4. Inflation: - In periods of inflation, a company may need to increase the


price of its products and services to offset the increases in the prices of
materials, labour and overheads.

5. Innovation: - A company may set a high price for an innovative product


price skimming.

6. Ethics: - When setting a price, the company will take into account various
ethical considerations: if the product is scarce, should it rise its price to exploit
these short-term shortages?

83
8.2 ACCA SYLLABUS GUIDE OUTCOME 2:
Explain the price elasticity of demand

Generally, it is expected that there will be an inverse relationship between


selling price and sales demand. If the selling price is increased, sales demand
would be expected to fall. If the selling price is reduced, sales demand would
be expected to rise. The key question is to what extent is demand likely to
respond to a change in price?

The price elasticity of demand (i.e. the degree of sensitivity of demand for a
product to changes in the price of that product) can be measured as:

% change in sales demand


% change in selling price

If the % change in demand >the % change in price then price elasticity > 1.
Demand is elastic, i.e. very responsive. Total revenue increases when price
is reduced, and decreases when price is increased.

If the % change in demand < the % change in price, then price elasticity < 1.
Demand is inelastic, i.e. not very responsive. Total revenue decreases when
price is reduced, and increases when price is increased.

Price elasticity of 1 will mean that the % change in demand offsets the %
change in price, leaving total sales revenue unchanged. An increase in selling
price will be offset by a decrease in sales demand: a decrease in selling price
will be offset by an increase in sales demand.

Illustration 1

At a price of $1.50, annual demand is 100,000.

If price is increased to $1.75, annual demand is 80,000

Price elasticity of demand is: -

% change in demand = 20,000/100,000 x 100 = -20%

% change in price = 0.25/1.50 x 100 = 16.67%

Price elasticity of demand is 20/16.67 = 1.2 (ignore the minus sign)

Lecture Example 1

ABC Ltd currently charges a price of $10 per unit and earns a 20% profit
margin. It is planning to earn a mark-up of 40%.

84
What will be the new selling price? What is the percentage change in
price?

8.3 ACCA SYLLABUS GUIDE OUTCOME 3:


Drive an equation for the total cost function (including volume-based
discounts)

8.3.1 Equation of a straight-line

The equation of a straight line is represented by the following equation: -

The gradient of the line = P = - b


Q

Lecture Example 2

If the linear function is y =1,500+10x

i. The line would cross the y axis at the point _______


ii. The gradient of the line is _________
iii. The independent variable is _______
iv. The dependent variable is ______

85
8.3.2 Total cost function

The total cost function is linear: -

y = a +bx

a is the fixed cost per period


b is the variable cost per unit
x is the activity level
y is the total cost = fixed cost + variable cost

Lecture Example 3

If the total cost of a product is given as: -

y = 1,800 + 6x

i. The fixed cost is $_______


ii. The variable cost per unit is $_______
iii. The total cost of producing 500 units is $ ________

86
8.3.3 Cost equations including volume-based discounts

Suppliers often offer discounts to encourage the purchase of increased


volumes. Where volume-based discounts are offered, a total cost equation
can be derived for each volume range.

Illustration 2

Variable Cost $5 up to 10,000 units. 10% discount applies on all units


purchased over 10,000 units.

Total Fixed Costs $100,000

x 10,000 , TC = 100,000 + 5x
x 10,001 , TC = 100,000 + 4.5x

8.4 ACCA SYLLABUS GUIDE OUTCOME 4:


Derive and manipulate a straight line demand equation
Determine prices and output levels for profit maximisation using the
demand based approach to pricing (both tabular and algebraic
methods)
Calculate the optimum selling price and quantity for an organisation,
equating marginal cost and marginal revenue

8.4.1 Demand-based approaches (the economists viewpoint)

Most firms recognise that there exists a relationship between the selling price
of their product or service and the demand.

The law of demand explains the inverse relation between quantity and price in
general. It can be stated as follows:

Ceteris Paribus (other things remaining equal), the quantity of a good


demanded will rise (expand) with every fall in its price and the quantity of a
good demanded will fall (contract) with every rise in its price.

87
The price-demand equation is in the form:-

P = a - bQ

Where

P is the selling price

Q is the quantity demanded at that price

a = theoretical maximum price (if price is set at a or above, demand will be


zero), i.e. from the graph above, at a price of $200, demand is zero.

b = the change in price required to change demand by 1 unit (the gradient of


the line) = P
Q

Lecture Example 4

A company sells a product at $25 per unit and has a demand of 150,000 units
per annum. Detailed market research shows that for every $1 increase in
selling price, annual demand would reduce by 25,000 units and for every $1
decrease in selling price, annual demand would increase by 25,000 units.

Required

On the assumption that the price and demand relationship is linear,


derive the equation P = a - bQ.

88
8.4.2 Profit Maximisation Tabular Method

Lecture Example 5

A manufacturing company is considering its pricing policy for next year. The
following are the expected levels of demand at different selling prices :

Selling price Annual Demand


(per unit) (units)
$100 50,000
$120 45,000
$130 40,000
$150 25,000

This product is produced in batches of 100 units. Variable costs per batch are
expected to be $2,000 and total fixed costs are expected to be $200,000.

Required

Identify which of the above four selling prices per unit will result in the
highest annual profit from this product.

8.4.3 Profit Maximisation Algebraic Approach

Economic theory states that, in a perfectly competitive market, the monopolist


maximises profit when

marginal revenue = marginal cost.

Marginal revenue (MR) is the extra revenue that an additional unit of product
will bring. It is the additional income from selling one more unit of a good. It
can also be described as the change in total revenue divided by the change in
the number of units sold.

It is very important to note that the gradient of MR function is twice the


gradient of the demand function: -

MR = a 2bQ

Marginal cost is the change in total cost that arises when the quantity
produced changes by one unit. That is, it is the cost of producing one more
unit of a good.

Example: -

89
Total Marginal Total Marginal
Units Cost/unit Price Profit
cost cost revenue revenue
1 30 30.0 30 35 35.0 35 5
2 55 27.5 25 65 32.5 30 10
3 77 25.7 22 93 31.0 28 16
4 97 24.2 20 116 29.0 23 19
5 116 23.2 19 138 27.6 22 22
6 134 22.3 18 156 26.0 18 22
7 151 21.6 17 170 24.3 14 19
8 168 21.0 17 182 22.8 12 14
9 184 20.4 16 193 21.4 11 9
10 200 20.0 16 203 20.3 10 3

Profits are maximised when MC = MR; in this case, when 6 units are
produced, MC = MR at $18 and total profit is $22.

At point A, MC = MR, i.e. profits are maximised at this point. At output less
than Q, the extra cost of making a unit is less than the extra revenue from
selling it. At output greater that Q, the extra costs of making a unit exceed the
revenue from selling it.

Illustration 3

A company is considering the price of a new product. It has determined that


the variable cost of making the item will be $24 per unit. Market research has
indicated that if the selling price were to be $60 per unit then the demand
would be 1,000 units per week.

However, for every $10 per unit increase in selling price, there would be a

90
reduction in demand by 50 units; and for every $10 reduction in selling price,
there would be an increase in demand of 50 units.

Calculate the optimal selling price.

Note: If Price P = a bx then Marginal Revenue = a 2bx

P = a - bQ

B = P = 10 = 0.2
Q 50

60 = a - 0.2 (1000)
a = 260

P = 260 - 0.2Q
MR = 260 - 0.4Q

If MC = MR
24 = 260 - 0.4Q
0.4 Q = 236
Q = 590

P = 260 - 0.2 (590)


= $142

Lecture Example 6

Following from lecture example 4, calculate the selling price of the product
that will maximise the companys profit if variable costs are $12/unit.

Lecture Example 7

Dino Ltd. is reviewing the selling price of one of its products. The current
selling price of the product is $25 per unit and annual demand is forecast to
be 150,000 units at this price. Market research indicates that the level of
demand would be affected by any change in the selling price. Detailed
analysis from this research shows that for every $1 increase in selling price,
annual demand would reduce by 10,000 units and that for every $1 decrease
in selling price, annual demand would increase by 10,000 units.

A forecast of the annual costs that would be incurred by Dino Ltd in respect of
this product at differing activity levels is as follows:

91
Annual production (units) 100,000 200,000
$ $
Direct material 200,000 400,000
Direct labour 600,000 1,200,000
Overhead costs 880,000 1,460,000

Required

1. Calculate the total variable cost per unit


2. Calculate the selling price of the product that will maximize the
companys profits.

8.5 ACCA SYLLABUS GUIDE OUTCOME 5:


Evaluate a decision to increase production and sales levels,
considering incremental costs, incremental revenues and other factors

When sales and production arise, what is the effect on net profit? Will the
increased contribution exceed any additional fixed costs incurred as a result of
the increased sales level?

Lecture Example 8

A company produces and sells Product A. Its forecast for the next financial
year is as follows:

$000 $000
Sales 200,000 units @ $10 2,000
Variable Costs:
Material 400
Labour 400
Variable Overheads 200 1,000
Contribution 1,000
Fixed costs 650
Net profit 350

The company would like to increase sales for this product. Hence, it would like
to consider two proposals: -

a) To produce some components in-house at present purchased from an


external supplier. Material costs will be reduced by 20% but fixed costs
will increase by 10%.

b) Marketing campaign: Cost $10,000. Sales will increase by 50,000 units


as the selling price on all units will be reduced to $8.

Should the company accept any of these proposals?

92
8.6 ACCA SYLLABUS GUIDE OUTCOME 6:
Explain different price strategies, including:

8.6.1 Cost-plus

Cost-plus pricing involves establishing the unit cost and adding a mark-up or
sales margin.

Full cost-plus pricing is a method of determining the sales price by calculating


the full cost of the product and adding a percentage mark-up for profit.

Advantages of full cost-plus pricing

a. It is a quick, simple and cheap method of pricing which can be


delegated to junior managers.
b. Since the size of the profit margin can be varied, a decision based on a
price in excess of full cost should ensure that a company working at
normal capacity will cover all of its fixed costs and make a profit.

Disadvantages of full cost-plus pricing

a. It fails to recognise that since demand may be determining price, there


will be a profit maximising combination of price and demand.
b. There may be a need to adjust prices to market and demand
conditions.
c. Budgeted output volume needs to be established. Output volume is a
key factor in the overhead absorption rate.
d. A suitable basis for overhead absorption must be selected, especially
where a business produces more than one product.
e. There is no attempt to establish optimum price

Marginal cost-plus pricing/mark-up pricing involves adding a profit margin to


the marginal cost of production/sales.

Advantages of marginal cost-plus pricing

a. It is a simple and easy method to use.


b. The mark-up percentage can be varied, and so mark-up pricing can be
adjusted to reflect demand conditions.
c. It draws management attention to contribution, and the effects of higher
or lower sales volumes on profit.
d. In practice, mark-up pricing is used in businesses where there is a
readily-identifiable basic variable cost.

Disadvantages of marginal pricing

93
a. Although the size of the mark-up can be varied in accordance with
demand conditions, it does not ensure that sufficient attention is paid to
demand conditions, competitors' prices and profit maximisation.
b. It ignores fixed overheads in the pricing decision, but the sales price
must be sufficiently high to ensure that a profit is made after covering
fixed costs.

8.6.2 Market Skimming

Essentially this strategy is used to achieve high unit profits in the early stages
of a products life cycle. This is done by charging a high price on entry to the
market and stimulating demand through advertising and promotion.
Customers are prepared to pay high prices in order to gain the perceived
status of owning the product early. This would enable the company to take
advantage of the unique nature of the product, thus maximising sales from
those customers who like to have the latest technology as early as possible.

The most suitable conditions for this strategy are:


the product is new and different
the product has a short life cycle and high development costs that need
to be recovered quickly
high prices in the early stages of a products life cycle are expected to
generate high initial cash inflows.
since high prices attract competitors, there needs to be barriers to entry
in order to deter competitors from entering the market
the strength and sensitivity of demand are unknown232425.

As the product enters later stages of its life cycle, the price will be reduced.
The approach essentially skims the profit in the early stages of the life cycle
before increased competition leads to lower prices.

One example of market skimming is digital cameras. When these were


introduced, the initial selling price was high. The manufacturers sought to
build profit early in the product life cycle - and to recover the development
costs over a relatively short period.

8.6.3 Penetration Pricing

Market penetration is the term used to describe a policy in which the initial
price is set at a lower level to build a strong market share, and is more likely to
be successful when demand is elastic. The price will make the product
accessible to a larger number of buyers and therefore the high sales volumes

23 June 2013 Qs 3b
24 June 2011 Qs 2b
25 June 2015 Qs 4c

94
will compensate for the lower prices being charged. This allows economies of
scale to be built rapidly so that unit costs can be reduced.

A penetration policy is used to discourage new entrants from entering the


market. It will shorten the initial period of a products life cycle in order to
enter the growth and maturity stages quickly.

8.6.4 Complementary Product

A complementary product is one that is used in conjunction with another


product. For example, tennis balls and tennis rackets, razors and blades,
printers and printer cartridges.

A complementary pricing strategy can take two forms:

1. The major product e.g. a printer or a camera) is priced at a


relatively low figure to encourage the purchase and lock the
consumer into subsequent purchases of relatively high price
consumables (e.g. printer cartridges or memory cards).

2. The major product (e.g. membership sports club) is priced at a


relatively high figure the consumer is locked into subsequent
low-value purchases (e.g. court fees).

8.6.5 Product-line pricing

A product line is a range of products that are intended to meet similar needs
of different target audiences. The products within the product line are related
but may vary in style, colour and quality.

Product-line pricing works by:


1. making the price entry point for the basic product relatively
cheap.
2. pricing other items in the range more highly.

An example of this will be a dinner set where serving plates are priced
relatively cheap but other, less essential matching items in the same range
(e.g. fish bowls) are priced higher. Customers will be prepared to pay a
relatively high price for the less essential items in order to build up a matching
set.

8.6.6 Volume discounting

Customers are offered a lower price per unit if they purchase a particular
quantity of products. There may be two types of discounts: -

1 Quantity discounts for customers that order large quantities.

95
2 Cumulative quantity discounts the discount increases as the
cumulative total ordered increases. This may appeal to those
who do not wish to place large individual orders but who
purchase large quantities over time.

Volume discounting is applied to products with a limited shelf life, e.g. fashion
items and also to clear unpopular items. The discounts discourage the
customers from trying out new suppliers as the cumulative quantity discounts
lock in the customer. Further purchases can b e made at a lower cost per
unit.

8.6.7 Price-discrimination

Price-discrimination occurs where a company sells the same products at


different prices in different markets.

This is possible if:


1. the seller can determine the selling price
2. customers can be segregated into different markets
3. customers cannot buy at the lower price in one market and sell at
the higher price in the other market

8.6.8 Relevant costing

For short-term decisions, the incremental costs of accepting an order should


be presented. Bids should then be made at prices that exceed incremental
costs. For short-term decisions many costs are likely to be fixed and
irrelevent. Short-term pricing decisions should meet the following conditions:

a) spare capacity should be available for all of the resources that are
required to fulfill and order
b) the bid price should represent a one-off price that will not be repeated
for future orders and
c) the order will utilize unused capacity for only a short period and
capacity will be released for use on more profitable opportunities.

For long-term decisions, a firm can adjust the supply of virtually all of the
resources. Therefore, cost information should be presented providing details
of all of the resources that are committed to a product or service. Since fixed
costs should be covered in the long-term by sales revenues there are strong
arguments for allocating such costs for long-run pricing decisions. To
determine an appropriate selling price, a mark-up is added to the total cost of
the resources assigned to the product/service to provide a contribution to
profits.

96
Lecture Example 9

John Robertson, a self employed builder, has been asked to provide a fixed
price quotation for some building work required by a customer. Robertsons
accountant has compiled the following figures, together with some notes as a
basis for a quotation.

$
Direct materials:
Bricks 200,000 at $100 per thousand 20,000 note 1
200,000 at $120 per thousand 24,000
Other materials 5,000 note 2
Direct labour:
Skilled 3,200 hours at $12 per hour 38,400 note 3
Unskilled 2,000 hours at $6 per hour 12,000 note 4
Other costs:
Scaffolding hire 3,500 note 5
Depreciation of general purpose machinery 2,000 note 6
General overheads 5,200 hours at $1 per hour 5,200 note 7
Plans 2,000 note 8

Total cost 112,100
Profit 22,420 note 9

Suggested price $134,520

Notes

1. The contract requires 400,000 bricks, 200,000 are already in stock and
200,000 will have to be bought in. This is a standard type of brick regularly
used by Robertson. The 200,000 in stock were purchased earlier in the year
at $100 per 1,000. The current replacement cost of this type of brick is $120
per 1,000. If the bricks in stock are not used on this job John is confident that
he will be able to use them later in the year.

2. Other materials will be bought in as required; this figure represents the


purchase price.

3. Robertson will need to be on site whilst the building work is performed. He


therefore intends to do 800 hours of the skilled work himself. The remainder
will be hired on an hourly basis. The current cost of skilled workers is $12 per
hour. If John Robertson does not undertake the building work for this
customer he can either work as a skilled worker for other builders at a rate of
$12 per hour or spend the 800 hours completing urgently needed repairs to
his own house. He has recently had a quotation of $12,000 for labour to repair
his home.

97
4. John employs four unskilled workers on contracts guaranteeing them a 40
hour week at $6 per hour. These unskilled labourers are currently idle and
would have sufficient spare time to complete the proposal under
consideration.

5. This is the estimated cost of hiring scaffolding.

6. John estimates that the project will take 20 weeks to complete. This
represents 20 weeks straight line depreciation on equipment used. If the
equipment is not used on this job it will stand idle for the 20 week period.
In either case its value at the end of the 20 week period will be identical.

7. This represents the rental cost of Johns storage yard. If he does not
undertake the above job he can rent his yard out to a competitor who will pay
him rent of $500 per week for the 20 week period.

8. This is the cost of the plans that John has already had drawn for the
project.

9. John attempts to earn a mark up of 20% on cost on all work undertaken.


John is surprised at the suggested price and considers it rather high. He
knows that there will be a lot of competition for the work.

Required:
Using relevant costing principles, calculate the lowest price that John
could quote for the customers building work. Explain your treatment of
each item in the accountants estimate.

(CAT T7 December 2004 Qs 2)

Lecture Example 10

Pride is developing a new product and is trying to establish a suitable price.


Previously they have set a mark-up of 20% to set the selling price.

The standard cost per unit has been estimated as follows:

$
Direct materials
Material 1 20 (4 kg at $5/kg)
Material 2 7 (1 kg at $7/kg)
Direct labour 13 (2 hours at $6.50/hour)
Fixed overheads 7 (2 hours at $3.50/hour)
47

98
Required:

Using the standard costs, calculate two different cost plus prices using two
different bases and explain an advantage and disadvantage of each method.

Further Question26

A company has entered two different new markets.

In market A, it is initially charging low prices so as to gain rapid market share


while demand is relatively elastic.

In market B, it is initially charging high prices so as to earn maximum profits


while demand is relatively inelastic.

Which price strategy is the company using in each market?

A. Penetration pricing in market A and price skimming in market B


B. Price discrimination in market A and penetration pricing in market B
C. Price skimming in market A and penetration pricing in market B
D. Price skimming in market A and price discrimination in market B

26 Specimen Exam Applicable from December 2014

99
CHAPTER 9
Cost Volume Profit Analysis
9.1 ACCA SYLLABUS GUIDE OUTCOME 1:
Explain the nature of CVP analysis

One of the most important decisions that needs to be made before any
business even starts is how much do we need to sell in order to break-even?
By break-even we mean simply covering all our costs without making a profit.
This type of analysis is known as cost-volume-profit analysis (CVP
analysis).

CVP analysis looks primarily at the effects of differing levels of activity on the
financial results of a business. The reason for the particular focus on sales
volume is because, in the short-run, sales price, and the cost of materials and
labour, are usually known with a degree of accuracy. Sales volume, however,
is not usually so predictable and therefore, in the short-run, profitability often
hinges upon it.

9.2 ACCA SYLLABUS GUIDE OUTCOME 2:


Calculate and interpret break even point (BEP) and margin of safety
Prepare break even charts and interpret the information they containe

The break-even point is when total revenues and total costs are equal, i.e.
there is no profit but also no loss made.

There are three methods for ascertaining this BEP:

a) The equation method

Total revenue total variable costs total fixed costs = Profit

Note: total fixed costs are used rather than unit fixed costs since unit fixed
costs will vary depending on the level of output. Also, selling price and costs
are assumed to remain constant per unit of output.

b) The contribution margin method

The contribution margin (contribution per unit) =


selling price per unit less the variable costs per unit.

100
BEP in units = Fixed Costs
Contribution per unit

Illustration 1

ABC Ltd has provided us with the following information: -

Selling price per unit $100


Variable costs per unit $60
Fixed Costs $250,000 per annum

Required:

a. How many units should ABC Ltd. sell in order to break even using the
equation method?

b. How many units should ABC Ltd. Sell In order to break even using the
contribution margin method?

a) Equation Method

TR TVC TFC = Profit


100Q 60Q 250,000 = 0
40Q = 250,000
Q = 6,250 units

b) Contribution Margin Method

BEP (units) = Fixed Costs = 250,000 = 6,250 units


Cont/unit 40

Contribution / unit = SP VC
= $100 - $ 60
= $40

c) The graphical method

With the graphical method, the total costs and total revenue lines are plotted
on a graph; $ is shown on the y axis and units are shown on the x axis.

The point where the total cost and revenue lines intersect is the break-even
point. The amount of profit or loss at different output levels is represented by
the distance between the total cost and total revenue lines.

Figure 1 shows a typical break-even chart for Company A. The gap between
the fixed costs and the total costs line represents variable costs.

101
(Extract from Cost-Volume-Profit Analysis by A. Iron, Student Accountant, 14/2010)
www.accaglobal.com/content/dam/acca/global/PDF-students/2012s/sa_jul10_F5_t_c vp.pdf

Alternatively, a contribution graph could be drawn. This is very similar to a


break-even chart; the only difference being that instead of showing a fixed
cost line, a variable cost line is shown instead. Hence, it is the difference
between the variable cost line and the total cost line that represents fixed
costs. The advantage of this is that it emphasises contribution as it is
represented by the gap between the total revenue and the variable cost lines.

102
(Extract from Cost-Volume-Profit Analysis by A. Iron, Student Accountant, 14/2010)
www.accaglobal.com/content/dam/acca/global/PDF-students/2012s/sa_jul10_F5_t_c vp.pdf

Margin of safety

The margin of safety indicates by how much sales can decrease before a loss
occurs, i.e. it is the excess of budgeted revenues over break-even revenues.

Alternatively, it may be calculated as a percentage:

Budgeted sales (units) break-even sales (units) x 100


budgeted sales (units)

It could be calculated in terms of $ sales revenue as follows:

[Budgeted sales (units) break-even sales (units)] x selling price

103
Lecture Example 1

W Ltd makes bags. It has drawn up the following budget for its next financial
period:

Selling price per unit $10


Variable production cost per unit $4
Sales commission 10% of selling price
Fixed production costs $400,000
Fixed selling and administration costs $200,000
Budgeted Sales 150,000 units

Required:-
a) Calculate the break-even point (in units and selling price).
b) Calculate the margin of safety (in units and as a percentage of
budgeted sales).

9.3 ACCA SYLLABUS GUIDE OUTCOME 3:


Calculate the contribution to sales ratio, in single and multi-product
situations, and demonstrate an understanding of its use

It is often useful in single product situations, and essential in multi-product


situations, to ascertain how much each $ sold actually contributes towards the
fixed costs. This calculation is known as the contribution to sales or C/S
ratio.

The C/S ratio is invaluable in helping us to quickly calculate the break-even


point in $ sales revenue: -

104
BEP (sales revenue) = Fixed Costs
C/S ratio

In single product situations, C/S ratio is calculated as follows: -

Total contribution
Total sales revenue

or

Contribution per unit


Selling price

Lecture Example 2

A company manufactures a single product which it sells for $20 per unit. The
product incurs a variable cost of $12 per unit. The companys weekly break-
even point is sales revenue of $18,000.

Required: -
a) Calculate the contribution to sales ratio.
b) What would be the profit in a week when 1,200 units are sold?

In multi-product situations, a weighted average C/S ratio is calculated by using


the formula:

Total contribution
Total sales revenue

This weighted average C/S ratio can then be used to find CVP information
such as break-even point, margin of safety, required profit (section 9.4 below)
etc.

Breakeven Point (BEP) = Fixed Costs


Average C/S Ratio

105
Lecture Example 3

The following information is available:-

A B C

SP/unit 10 12 15
VC/unit 3 4 6
Units 1,000 700 800

Fixed costs are $120,000

Required :-

a) Calculate the weighted average contribution to sales ratio


b) Calculate the break-even sales revenue (to the nearest $1)

9.4 ACCA SYLLABUS GUIDE OUTCOME 4:


Calculate target profit or revenue in single and multi-product situations,
and demonstrate an understanding of its use

How many units must be sold to earn a target profit or revenue of an $x


amount? The sales volume necessary in order to achieve this profit can be
ascertained using any of the three methods outlined above: -

a) The equation method

Total revenue total variable costs total fixed costs =


Target Profit/Revenue

b) The contribution margin method

The contribution margin (contribution per unit) =

BEP in units = Fixed Costs + Target Profit / Revenue


Contribution per unit

In a multi-product situation,

Required Revenue = Fixed Costs + Target Profit / Revenue


Average C/S Ratio

c) The graphical method

The answer can also be read from the graph. The gap between the total
revenue and total cost line represents profit (after the break-even point) or
loss (before the break-even point).

106
Illustration 2

ABC Ltd has provided us with the following information: -

Selling price per unit $100


Variable costs per unit $60
Fixed Costs $250,000 per annum

ABC Ltd. wants to achieve a target profit of $400,000.

Calculate the number of units which the company should produce and
sell next year in order to achieve the target level of profit.

Equation Method

TR TVC TFC = Profit


100Q 60Q 250,000 = 400,000
40Q = 650,000
Q = 16,250 units

OR

Contribution Margin Method

BEP (units) = FC + TP = 250,000 + 400,000 = 16,250 units


Cont/unit 40

Lecture Example 4

Despard Ltd manufactures and sells a single product. The following data
have been extracted from the current years budget:

Sales and production (units) 5,000


Variable cost per unit $50
Fixed cost per unit $70
Contribution to sales ratio 75%

The selling price per unit for next year is to be 8% above the current years
budgeted figure, whereas both the variable cost per unit and the total fixed
costs are forecast to increase by 12% above their budgeted level in the
current year.

The target for next year is that total profit should remain the same as that
budgeted for the current year.

107
Required:
a) Calculate for the CURRENT YEAR the budgeted:
i. contribution per unit;
ii. total profit
b) Calculate the number of units which the company should produce
and sell next year in order to achieve the target level of profit.

(ACCA Paper 1.2 December 2004 Qs 2)

Lecture Example 5

The following data is available for product Alpha.

Selling price : $120 per unit


Variable cost : $80 per unit.

Fixed costs are $200,000. The company budgets to produce 9,000 units next
year.

Required:

a) Calculate:
i) The break-even point (expressed in units and $ of revenue)
ii) The level of activity required to generate a profit of $90,000
(expressed in units).
iii) The margin of safety in units and as a percentage.

b) Using graph paper, draw a breakeven chart for part (a).

c) On the graph drawn in part (b), show the effect of a change in selling
price to $130 per unit.

9.5 ACCA SYLLABUS GUIDE OUTCOME 5:


Prepare profit volume charts and interpret the information contained
within each, including multi-product situations27

As we have seen in the first chart, the break even chart shows the profit or
loss outlook for a wide range of output levels. The breakeven point is where
the total revenues line and the total costs line intersect.

The profit-volume graph focuses purely on showing a profit/ loss line and
doesnt separately show the cost and revenue lines.

27 Examined Sept/Dec 2015 Qs 4

108
Chart for a single product:

In a multi-product environment, it is common to actually show two lines on the


graph: one straight line, where a constant mix between the products is
assumed; and one bow-shaped line, where it is assumed that the company
sells its most profitable product first and then its next most profitable product,
and so on.

In order to draw the graph, it is therefore necessary to work out the C/S ratio
of each product being sold before ranking the products in order of profitability.

109
(Extract from Cost-Volume-Profit Analysis by A. Iron, Student Accountant, 14/2010)
www.accaglobal.com/content/dam/acca/global/PDF-students/2012s/sa_jul10_F5_t_c vp.pdf

It can be observed from the graph that, when the company sells its most
profitable product first (x) it breaks even earlier than when it sells products in a
constant mix. The break-even point is the point where each line cuts the x
axis.

Lecture Example 6

The following information is available:

Product X Y Z
Selling Price per unit $20 $25 $15
Variable Cost per unit $5 $15 $5
Budgeted Sales volume (in units) 150,000 110,000 200,000

The company expects the fixed costs to be $800,000 for the coming year.

Required:

110
a) Calculate the weighted average C/S ratio for the products (to 2
decimal places).
b) Calculate the break-even sales revenue required.
c) Calculate the amount of sales revenue required to generate a profit of
$500,000.
d) Calculate the margin of safety in terms of sales revenue and also as a
percentage of budgeted sales revenue.
e) Draw a multi-product profit-volume chart assuming the budget is
achieved.

9.6 ACCA SYLLABUS GUIDE OUTCOME 6:


Discuss the limitations of CVP analysis for planning and decision-
making

a) Cost-volume-profit analysis is invaluable in demonstrating the effect on


an organisation that changes in volume (in particular), costs and selling
prices, have on profit. However, its use is limited because it is based on
the following assumptions: either a single product is being sold or, if
there are multiple products, these are sold in a constant mix. We have
considered this above and seen that if the constant mix assumption
changes, so does the break-even point.

b) All other variables, apart from volume, remain constant, i.e. volume is
the only factor that causes revenues and costs to change. In reality,
this assumption may not hold true as, for example, economies of scale
may be achieved as volumes increase. Similarly, if there is a change in
sales mix, revenues will change. Furthermore, it is often found that if
sales volumes are to increase, sales price must fall.

c) The total cost and total revenue functions are linear. This is only likely
to hold true within a short-run, restricted level of activity.

d) Costs can be divided into a component that is fixed and a component


that is variable. In reality, some costs may be semi-fixed, such as
telephone charges, whereby there may be a fixed monthly rental
charge and a variable charge for calls made.

e) Fixed costs remain constant over the relevant range levels of activity
in which the business has experience and can therefore perform a
degree of accurate analysis. It will either have operated at those activity
levels before or studied them carefully so that it can, for example, make
accurate predictions of fixed costs in that range.

f) Profits are calculated on a variable cost basis or, if absorption costing


is used, it is assumed that production volumes are equal to sales
volumes.

111
Illustration 3

Hughes plc has recently developed a personal music player and is now
considering what price to charge for the new product. A market research
company has produced the following forecasts of demand at three potential
selling prices:

Selling price $250 $350 $450


Sales units per annum 10,000 8,000 6,000
Fixed costs per annum $800,000 $ 500,000 $200,000

Variable costs are forecast at $220 per unit at any activity level.

Required:

a) Calculate, for each potential selling price, the budgeted profit, the
break-even point in units and the margin of safety ratio (i.e. the
margin of safety expressed as a percentage).
b) Using the graph paper provided, draw and label a break-even chart
for a selling price of $350 for activity levels between 0 and 8,000
units.
(CAT Paper T7 June 2005 Qs 2 amended)

a)
$ $ $
Selling price 250 350 450
Variable cost 220 220 220
Contribution per unit 30 130 230
Total units 10,000 8,000 6,000
Total contribution 300,000 1,040,000 1,380,000
Fixed cost 800,000 500,000 200,000
Profit (500,000) 540,000 1,180,000

Breakeven point in units = Fixed costs/contribution per unit

Selling price ($) 250 350 450


Contribution per unit (1) 30 130 230
Fixed cost ($) (2) 800,000 500,000 200,000
Breakeven point (units) (2 1) 26,667 3,846 870
Budgeted units 10,000 8,000 6,000
Margin of safety nil 4,154 5,130
Margin of safety ratio nil 52% 86%
MOS = Budgeted Sales Break-Even Sales

Or

112
Budgeted Sales Break-Even Sales
Budgeted Sales

b) Breakeven Chart

Breakeven chart

$000 Total
2,800 Revenue
2,600
2,400
2,200
2,000 Total
1,800 Cost
1,600
1,400
1,200
1,000 Breakeven point
800
600 Margin of Safety
400 Fixed
200 Cost
0
1 2 3 4 5 6 7 8

000 units

Further questions

113
Question 1

A company manufactures a single product which it sells for $20 per unit. The
product has a contribution to sales ratio of 40%. The companys weekly break-
even point is sales revenue of $18,000.

What would be the profit in a week when 1,200 units are sold?

A. $1,200
B. $2,400
C. $3,600
D. $6,000

Question 2

Information concerning Razor Ltds single product is as follows.

$ per unit
Selling price 6.00
Variable production cost 1.20
Variable selling cost 0.40
Fixed production cost 4.00
Fixed selling cost 0.80

Budgeted production and sales for the year are 10,000 units.

How many units must be sold if Razor Ltd wants to achieve a profit of $11,000
for the year?

A. 2,500 units
B. 9,833 units
C. 10,625 units
D. 13,409 units

Question 3

Sky Ltd sells a single product. In the coming month, it is budgeted that this
product will generate total revenue of $300,000 with a contribution of
$125,000. Fixed costs are budgeted at $100,000 for the month.

What is the margin of safety?

A. 0%
B. 10%
C. 20%

114
D. 25%

Question 4

CVC makes and sells a single product which has a selling price of $26, prime
costs are $10 and overheads (all fixed) are absorbed at 50% of prime cost.
Fixed overheads are $50,000.

What is the break-even point (to the nearest whole unit)?

A. 1,923
B. 3,125
C. 4,545
D. 5,000

Question 5

A company manufactures a single product which it sells for $15 per unit. The
product has a contribution to sales ratio of 40%. The companys weekly break-
even point in sales revenue is $18,000.

What would be the profit in a week when 1,500 units are sold?

A. $900
B. $1,800
C. $2,700
D. $4,500

Question 6

The following break-even chart has been drawn showing lines for total cost
(TC), total variable cost (TVC), total fixed cost (TFC) and total sales revenue
(TSR);

$ TSR

115
TC

TVC

TFC

0 675 1200 1500 1700 units

What is the margin of safety at the 1,700 units level of activity?

A. 200 units
B. 300 units
C. 500 units
D. 1,025 units

Question 7

Four vertical lines have been labeled G,H,J and K at different levels of activity
on the following profit-volume chart:

0
output
G

H J

Which line represents the total contribution at that level of activity?

A. Line G

116
B. Line H
C. Line J
D. Line K

Question 828

A company makes a single product which it sells for $2 per unit.

Fixed costs are $13,000 per month.


The contribution/sales ratio is 40%.

Sales revenue is $62,500.

What is the margin of safety (in units)?

A. 15,000
B. 16,250
C. 30,000
D. 31,250

28 Specimen Exam Applicable from December 2014

117
CHAPTER 10
Dealing with Risk and
Uncertainty in Decision-Making
Risk refers to the situation where probabilities can be assigned to a range of
expected outcomes arising from an investment project and the likelihood of
each outcome occurring can therefore be quantified.

For e.g., based on past experience, a sales team may estimate it has a 60%
chance of winning a particular contract.

Uncertainty refers to the situation where probabilities cannot be assigned to


expected outcomes. Investment project risk therefore increases with
increasing variability of returns, while uncertainty increases with increasing
project life. For e.g., it is very difficult to assign probabilities to a new product
entering into a market

The two terms are often used interchangeably in financial management, but
the distinction between them is a useful one.

10.1 ACCA SYLLABUS GUIDE OUTCOME 1:


Suggest research techniques to reduce uncertainty, e.g. focus groups,
market research

Risk Management is the process of understanding and managing the risks


that the organisation will inevitably meet in attempting to achieve its
objectives.

10.1.1 Market Research

Market research assesses and reduces uncertainty about the likely responses
of customers to new products, new advertising campaigns, price changes, etc.
This can be desk-based (secondary) or field-based (primary).

Desk-based research is cheap but can lack focus. It is collected from


secondary sources, i.e. published and other available sources of information.

Field-based research is research by direct contact with a targeted group of


potential customers. It is better than desk-based research in that you can
target your customers and your product area. However, it can be time
consuming and expensive. The internet is bringing down the cost and

118
speeding up this type of research, email is being used to gather information
quickly on the promise of free gifts etc.

Field-based research can be either: -


1. motivational OR
2. measurement

Motivational research the objective is to unearth factors why consumers do


or do not buy particular products. Depth and group interviewing techniques
are used in motivational research.

Measurement research the objective is to build on the motivation research


by trying to quantify the issues involved. Sample surveys are used to find out
how many people buy the products, in what quantity, from where and when.

10.1.2 Focus Groups

Focus groups are a form of market research. They are small groups (typically
eight to ten individuals) selected from a broader population who are
interviewed through discussions in an informal setting. They are questioned in
order to gather their opinions and reactions to a particular subject or
marketing-orientated issues, known as test concepts

These focus groups can provide market researchers with much helpful
information. However, it is difficult to measure the results objectively. Their
cost and logistical complexity is frequently cited as a barrier, especially for
smaller companies.

Focus groups have been used by banks to assess consumer reactions to new
electronic banking products and by television companies to obtain voters
reactions to political elections.

10.2 ACCA SYLLABUS GUIDE OUTCOMES 2 AND 3:


Explain the use of simulation, expected values and sensitivity
Apply expected values and sensitivity to decision-making problems
Apply expected values and explain the problems and benefits

10.2.1 Simulation

Simulation is a modeling technique used mainly in capital investment


appraisal decisions.

Computer models can be built to simulate real life scenarios. The model will
predict what range of returns an investor could expect from a given decision

119
without having risked any actual cash. The models use random number tables
to generate possible values for the uncertainty the business is subject to.
Since the time and costs involved can be more that benefits gained, computer
technology is assisting in bringing down the cost of such risk analysis. Models
can become extremely complex and probability distributions may be difficult to
formulate.

10.2.2 Expected Values (EV)

The expected value rule calculates the average return that will be made if a
decision is repeated again and again. It does this by weighting each of the
possible outcomes with their relative probability of occurring. It is the weighted
arithmetic mean of the possible outcomes.

The likelihood that an event will occur is known as its probability. This is
normally expressed in decimal form with a value between 0 and 1. A value of
0 denotes a nil likelihood of occurrence whereas a value of 1 signifies
absolute certainty. A probability of 0.4 means that the event is expected to
occur four times out of ten. The total of the probabilities for events that can
possibly occur must sum up to 1.0.

An expected value is computed by multiplying the value of each possible


outcome by the probability of that outcome, and summing the results.

EV = px

Where p = probability of the outcome


x= the possible outcome

A risk neutral investor will generally make his decisions based on maximizing
EV.

Illustration 1

Economic State Probability Project A Project B Project C

Good 0.25 $140,000 $95,000 $30,000


Average 0.45 $90,000 $80,000 $60,000
Poor 0.30 ($20,000) $82,000 ($8,000)

Expected Value = px Project A Project B Project C


(140,000 x 0.25) (95,000 x 0.25) (30,000 x 0.25)
Good $35,000 $23,750 $7,500
Average $40,500 $36,000 $27,000
Poor ($6,000) $24,600 ($2,400)
$69,500 $84,350 $32,100

120
Project B should be chosen as it maximizes EV at $84,350

Advantages and disadvantages of EVs

Advantages:

Takes risk into account by considering the probability of each possible


outcome and using this information to calculate an expected value.
The information is reduced to a single number resulting in easier
decisions.
Calculations are relatively simple.

Disadvantages:

The probabilities used are usually very subjective.


The EV is merely a weighted average and therefore has little meaning
for a one-off project.
The EV gives no indication of the dispersion of possible outcomes
about the EV, i.e. the risk.
The EV may not correspond to any of the actual possible outcomes.

Since the expected value shows the long run average outcome of a decision
which is repeated time and time again, it is a useful decision rule for a risk
neutral decision maker. This is because a risk neutral person neither seeks
risk or avoids it; they are happy to accept an average outcome 29.

10.2.2.1 Limitations of EV

1. Forecasts may be inaccurate and the probabilities used are also


usually very subjective.
2. The EV is a weighted average of the probability distribution. It will never
actually occur.
3. Expected values are more valuable as a guide to decision making
where they refer to outcomes which will occur many times over.
Examples would include the probability that so many customers per
day will buy a particular product, the probability that a customer care
assistant will receive a number of phone calls per hour, etc.
4. It ignores risk and the investors attitude to risk. This will be described
further in Section 10.4.1.

29 F5 June 2011 Qs 1c

121
10.2.3 Sensitivity Analysis

Sensitivity analysis can be used to assess the range of values that would still
give the investor a positive return. It is a technique which is similar to the what
if? scenario. The uncertainty may still be there, but the effect that it has on
the investors returns will be better understood.

Sensitivity calculates the % change required in individual values before a


change of decision results. If only a (say) 2% change is required in selling
price before losses result, an investor may think twice before proceeding.

Therefore, sensitivity analysis assesses how the net present value of an


investment project is affected by changes in project variables. Considering
each project variable in turn, the change in the variable required to make the
net present value zero is determined, or alternatively the change in net
present value arising from a fixed change in the given project variable. In this
way the key or critical project variables are determined.

However, sensitivity analysis does not assess the probability of changes in


project variables and so is often dismissed as a way of incorporating risk into
the investment appraisal process.

Illustration 2

Two possible outcomes for a process:-

Probability Outcome
0.4 Loss of $20,000
0.6 Profit of $40,000

1. Expected Value

EV = px

Outcome Probability px
(x) (p)

(20,000) 0.4 (8,000)


40,000 0.6 24,000
EV = 16,000

Should the process be undertaken?

Yes, it has a positive expected value

2. What level of loss would lead this process not to be undertaken?

122
The decision will change if EV from the process is 0 i.e. break-even.

Loss x 0.4 = 40,000 x 0.6


= 24,000

0.4 Loss = 24,000

Loss = 24,000 = 60,000


0.4

If the loss from the process is more than $60,000, the process will not be
under taken.

10.3 ACCA SYLLABUS GUIDE OUTCOME 4:


Apply the techniques of maximax, maximin and minimax regret to
decision-making problems including the production of profit tables

10.3.1 Attitudes to risk by individuals

How do we determine whether or not a risky course of action should be


undertaken? This depends on the decision-makers attitude to risk. There are
3 possible attitudes: -

1. an aversion to risk
2. a desire for risk
3. an indifference to risk

A risk-seeker is one who, given a choice between more or less risky


alternatives with identical expected values, prefers the riskier alternative.
Faced with the same choice, a risk-averter would select the less risky
alternative. The person who is indifferent to risk (risk neutral) would be
indifferent to both alternatives because they have the same expected values.

10.3.2 Maximax

The maximax rule applies to an optimist (risk-seeker) who seeks to maximise


the maximum possible gain of possible outcomes.

10.3.3 Maximin

The maximin decision rule looks at the worst possible outcome at each
supply level and then selects the highest one of these. It is used when the
outcome cannot be assessed with any level of certainty. The decision maker
therefore chooses the outcome which is guaranteed to minimise his losses. In

123
the process, he loses out on the opportunity of making big profits. It is often
seen as the pessimistic approach to decision-making (assuming that the worst
outcome will occur) and is used by decision makers who are risk averse. It
can be used for one-off or repeated decisions.

10.3.4 Minimax Regret Rule

The minimax regret strategy minimises the maximum regret. Regret means
the opportunity loss from having made a wrong decision.

Lecture Example 1

A manufacturing firm has a capacity of 1,200 units per month.

The variable cost per unit is $6 and each unit is sold for $11. However, the
demand per month is uncertain and is as follows:

Demand Probability
400 0.2
500 0.3
700 0.4
900 0.1

A customer is prepared to contract a fixed quantity per month at a price of $9


per unit. The customer is prepared to sign the contract to purchase 300, 500,
700 or 800 units per month.

The company can vary production levels during the month up to a maximum
capacity, but cannot carry forward any unsold units in stock.

(a) Calculate all possible profits that could result.

(b) Determine for what quantity the company should sign the contract,
under each of the following criteria:

i) Expected value
ii) Maximin
iii) Maximax
iv) Minimax regret

124
10.4 ACCA SYLLABUS GUIDE OUTCOME 5:
Calculate the value of perfect and imperfect information

When a decision-maker is faced with a series of uncertain events that might


occur, he or she should consider the possibility of obtaining additional
information about which event is likely to occur.

Perfect information is available when a 100% accurate prediction can be


made about the future.

Imperfect information The concept of perfect information is somewhat


artificial since, in the real world, such perfect certainty rarely, if ever, exists.
For example, predictions for future demand may only be 80% reliable. Hence,
the value of imperfect information will always be less than the value of perfect
information unless both are zero. This would occur when the additional
information would not change the decision.

The approach to calculate the value of perfect and imperfect information is the
same: - compare the expected value of a decision if the information is
acquired against the expected value with the absence of the information. The
difference represents the maximum amount it is worth paying for the
additional information.

Therefore the value of information (either perfect or imperfect) may be


calculated as follows:

Expected Profit (Outcome) WITH the information LESS Expected Profit


(Outcome) WITHOUT the information.

Lecture Example 2

Following from Lecture Example 1

There is a possibility of employing a firm of market consultants who


would be able to provide a perfect prediction of the actual demand.
What is the maximum amount the company should be prepared to pay
the consultants for the additional information?

Lecture Example 330

The Mobile Sandwich Co prepares sandwiches which it delivers and sells to


employees at local businesses each day. Demand varies between 325 and
400 sandwiches each day. As the day progresses, the price of the
sandwiches is reduced and, at the end of the day, any sandwiches not sold
are thrown away.

30 Examiners Report, June 2015

125
The company has prepared a regret table to show the amount of profit that
would be foregone each day at each supply level, given the varying daily
levels of demand: -

Applying the decision criterion of minimax regret, how many sandwiches


should the company decide to supply each day?

A. 325
B. 350
C. 375
D. 400

Lecture Example 431

An ice cream seller has to decide how much ice cream to order (small,
medium or large order), taking into consideration the weather forecast (cold,
warm or hot). There are nine possible combinations of order size and
weather, and the payoffs for each are: -

Order/weather Cold Warm Hot


Probability 0.2 0.5 0.3
Small $250 $200 $150
Medium $200 $500 $300
Large $100 $300 $750

a. Using the information above, determine the order the ice cream seller
should make, under each of the following criteria:

i. Expected Value
ii. Maximin
iii. Maximax
iv. Minimax regret

b. If the ice cream seller can obtain perfect information regarding the
outcome of the weather, what is the maximum amount he is willing to
pay for this perfect information?

31 M. Pogue, The Risks of Uncertainty, Student Accountant October 2009


http://www.accaglobal.com/content/dam/acca/global/pdf/sa_oct09_pogue.pdf

126
c. The ice cream seller is aware that weather forecasts may exhibit
varying degrees of accuracy. In fact, past experience over 100 days
has suggested the following probabilities: -
P (forecast was hot but weather turned out to be cold) 0.3
P (forecast was hot but weather turned out to be warm) 0.4
P (forecast was hot but weather turned out to be hot) 0.7

Calculate the value of this imperfect information

Lecture Example 5

Shifters Haulage (SH) is considering changing some of the vans it uses to


transport crates for customers. The new vans come in three sizes; small,
medium and large. SH is unsure about which type to buy. The capacity is 100
crates for the small van, 150 for the medium van and 200 for the large van.

Demand for crates varies and can be either 120 or 190 crates per period, with
the probability of the higher demand figure being 06.

The sale price per crate is $10 and the variable cost $4 per crate for all van
sizes subject to the fact that if the capacity of the van is greater than the
demand for crates in a period then the variable cost will be lower by 10% to
allow for the fact that the vans will be partly empty when transporting crates.
SH is concerned that if the demand for crates exceeds the capacity of the
vans then customers will have to be turned away. SH estimates that in this
case goodwill of $100 would be charged against profits per period to allow for
lost future sales regardless of the number of customers that are turned away.

Depreciation charged would be $200 per period for the small, $300 for the
medium and $400 for the large van. SH has in the past been very aggressive
in its decision-making, pressing ahead with rapid growth strategies. However,
its managers have recently grown more cautious as the business has become
more competitive.

Required:

(a) Explain the principles behind the maximax, maximin and expected value
criteria that are sometimes used to make decisions in uncertain situations.
(4 marks)

(b) Prepare a profits table showing the SIX possible profit figures per period.
(9 marks)

(c) Using your profit table from (b) above discuss which type of van SH should
buy taking into consideration the possible risk attitudes of the managers.
(6 marks)

127
(d) Describe THREE methods other than those mentioned in (a) above, which
businesses can use to analyse and assess the risk that exists in its decision-
making.
(6 marks)

(ACCA December 08 Question 2)

10.5 ACCA SYLLABUS GUIDE OUTCOME 6:


Draw a decision tree and use it to solve a multi-stage decision problem.

A useful analytical tool for clarifying the range of alternative courses of action
and their possible outcomes is a decision tree. A decision tree is a diagram
showing several possible courses of action and possible events and the
potential outcomes for each course of action. For e.g. deciding whether to
expand the business or not.

There are two main stages to making decisions using decision trees: -

1. The decision tree is drawn and all probabilities and outcome values are
included.

2. Evaluation and recommendation stage: all expected values are


calculated at all outcome points and these are used to make decisions.
A course of action is then recommended.

1. Constructing the decision tree

Draw the tree from left to right, showing appropriate decisions and events
/outcomes.

A square is used to represent a decision point (i.e. where a choice between


different courses of action must be taken within your control).

A circle is used to represent a chance/outcome point. Outcomes are not within


your control. They depend on the external environment, e.g. customers,
suppliers and the economy.

Each alternative course of action or event is represented by a branch. The


branches have probabilities attached to them.

All decision trees must start with a square, representing a decision.

128
Figure 132: -

There are two branches coming off the decision point. The outcome of one of
these choices (the top branch) is certain. However, the lower branch shows
that there are two possible outcomes. There are two more sets of outcomes
for these initial outcomes.

Once the tree has been drawn, label the tree and relevant cash
inflows/outflows and probabilities associated with outcomes. Probabilities will
add up to 1 or 100%.

Evaluation of the decision

Evaluate the tree from right to left, i.e. in the opposite direction to when the
tree was drawn.

a) Calculate an EV at each outcome point by applying the probabilities to


the cashflows.

b) Choose the best option at each decision point.

Finally, a recommendation should be made to management, based on the


option that gives the highest expected value. Remember that expected values
give us a long run average of the outcome which is expected only if a decision

32Extracted from Decision Trees, ACCA Article, January 2013 written by a member of the F5
examining team

129
is to be repeated many times. Since this is a one-off decision, this technique
is not very accurate. Also, expected values assume that the investor is risk
neutral. Hence, this may not be accurate if the attitude to risk is unknown.

Lecture Example 633

A company is deciding whether to develop and launch a new product.


Research and development costs are expected to be $400,000 and there is a
70% chance that the product launch will be successful, and a 30% chance
that it will fail. If it is successful, the levels of expected profits and the
probability of each occurring have been estimated as follows, depending on
whether the products popularity is high, medium or low:

Probability Profits
High 0.2 $500,000 p.a. for 2 years
Medium 0.5 $400,000 p.a. for 2 years
Low 0.3 $300,000 p.a. for 2 years

If it is a failure, there is a 0.6 probability that the research and development


work can be sold for $50,000 and a 0.4 probability that it will be worth nothing
at all.

Required: -

a) Using decision tree analysis, establish the best course of action


for the company.
b) An agency can provide perfect information on whether the launch
is going to be successful and produce high, medium or low profits
or whether it is simply going to fail. What is the maximum amount
that the company should pay to acquire this information?

Lecture Example 7

Firlands Ltd, a retail outlet, is faced with a decision regarding whether or not to
expand and build small or large premises at a prime location. Small premises
would cost $300,000 to build and large premises would cost $550,000.

Regardless of the type of premises built, if high demand exists then the net
income is expected to be $1,500,000. Alternatively, if low demand exists, then
net income is expected to be $600,000.

If large premises are built then the probability of high demand is 0.75. If
smaller premises are built then the probability of high demand falls to 0.6.

33Decision Trees, ACCA Student Accountant, January 2013, written by a member of the F5 examining
team

130
Firlands has the option of undertaking a survey costing $50,000. The survey
predicts whether there is likely to be a good or bad response to the size of the
premises. The likelihood of there being a good response, from previous
surveys, has been estimated at 0.8.

If the survey indicates a good response then the company will build the large
premises. If the survey does give a good result then the probability that there
will be high demand from the large premises increases to 0.95.

If the survey indicates a bad response then the company will abandon all
expansion plans.

Requires:
Using decision tree analysis, establish the best course of action for
Firlands Ltd.

(ACCA Paper 1.2 December 2002 Qs 2)

Further Questions

Question 1

Healthfoods Ltd (HFL) is a well-established company which markets fruit and


vegetables under their Good Health brand name at each of its 6 outlets in the
country of Ateland. During the recent years HFL has marketed organically
grown fruit and vegetables. The directors are now planning to market organic
mushrooms which have a unique eating quality and will be the most nutritious
mushrooms available on the market.

The finance director has collated the following information regarding the
proposed introduction and sale of organic mushrooms within Ateland:

(1) HFL will purchase the organic mushrooms from Orgmush Ltd (OML) and
sell them at each of its six outlets within Ateland.

(2) OML, which is the only grower of this particular type of organic mushroom
within Ateland, has offered HFL a choice of four different contracts in
respect of the forthcoming year. OML has the capacity to produce 360,000
kilograms of organic mushrooms for each of the six outlets.

Contract Number of kilograms Cost per kilogram ($)


A 160,000 4.45
B 240,000 3.70
C 280,000 3.55
D 360,000 3.35
Note: The same contract type must be chosen in respect of each outlet.

131
(3) HFL will charge $5.50 per kilogram for all sales of organic mushrooms.

(4) Any unsold produce will be sold to the Animal Farm Group for $0.25 per
kilogram.

(5) HFL must decide in advance of the forthcoming year which size of
contract to enter.

(6) HFL uses acclaimed dieticians, international athletes or international film


stars to promote its products via television advertisements and has
estimated the following probability distribution of advertisements to be held
during the forthcoming year:

Category of advertisement: %
Acclaimed dietician 20
International athlete 40
International film star 40

Market research has indicated that where an acclaimed dietician appears in


an advertisement, HFL can be reasonably assured of selling 160,000
kilograms of mushrooms per outlet and where an international athlete appears
in an advertisement then 234,000 kilograms of mushrooms per outlet will be
sold. HFL expects to sell 360,000 kilograms of mushrooms per outlet when an
international film star appears in an advertisement.

Required:

(a) Using the expected values, advise HFL regarding which contract
should be entered into with OML.
Your answer should show the expected annual contribution from each contract.
(12 marks)

(b) Determine whether your decision in (a) would change if you were to
use each of the Maximin and Minimax regret decision criteria.
Your answer should be supported by relevant workings.
(6 marks)

(c) Briefly discuss why the directors of HFL might choose contract D
irrespective of whether or not contract D would have been selected
using expected values as per part (a).
(2 marks)

(20 marks)

(ACCA Paper 3.3 June 2007 Qs 4)

ContributionTable

132
Demand Contract A Contract B Contract C Contract D

160,000 (1) 1,008,000 (4) 72,000 (7) -504,000 (10) -1,656,000


234,000 (2) 1,008,000 (5) 2,403,000 (8) 1,827,000 (11) 675,000
360,000 (3) 1,008,000 (6) 2,592,000 (9) 3,276,000 (12) 4,644,000

(1) 160,000 x (5.50 4.45) = 168,000 x 6 outlets


= 1,008,000

Since amount of mushrooms is restricted to 160,000 kgs per outlet,


contribution of (2) and (3) are equal to (1).

(4) 160,000 x (5.50 3.70) x 6 = 1,728,000


240,000 160,000 = 80,000 x (0.25 3.70) = (276,000) x6 (1,656,000)

Therefore net contribution = 1,728 1,656


= 72,000

(5) 234,000 x (5.50 3.70) x 6 = 2,527,200


240,000 234,000 = 6,000 x (0.25 3.70) = (20,700) x6 (124,200)

Therefore net contribution = 2,527,200 124,200


= 2,403,000

(6) 240,000 x (5.50 3.70) x 6 = 2,592,000

(7) 160,000 x (5.50 3.55) x 6 = 1,872,000


280,000 160,000 = 120,000 x (0.25 3.55) x6 = (2,376,00)

Therefore net contribution = 1,872 2,376


= (504,000)

(8) 234,000 x (5.50 3.55) x 6 = 2,737,800


280,000 234,000 = 46,000 x (0.25 3.55) x6 = (910,800)

Therefore net contribution = 2,737,800 910,800


= 1,827,000

(9) 280,000 x (5.50 3.55) x 6 = 3,276,000

(10) 160,000 x (5.50 3.55) x 6 = 2,064,000


360,000 160,000 = 200,000 x (0.25 3.35) x6 = (3,720,000)

Therefore net contribution = (1,656,000)

(11) 234,000 x (5.50 3.70) x 6 = 3,018,600

133
360,000 234,000 = 126,000 x (0.25 3.55) x6 = (2,343,600)

Therefore net contribution = 675,000

(12) 360,000 x (5.50 3.35) x 6 = 4,644,000

EV Table

Contact
Demand Probability A B C D
([1656] x
(1008 x 0.2) (72 x 0.2) ([504] x 0.2) 0.2)
160,000 0.2 201,600 14,400 (100,800) (331,200)
234,000 0.4 403,200 961,200 730,800 270,000
360,000 0.4 403,200 1,036,800 1,310,400 1,857,600
1.0 1,008,000 2,012,400 1,940,400 1,796,400
Choose Contract B highest EV.

b) Maximin maximize the minimum outcome (use contribution table).

Contract Min. Return

Contract A 1,008,000 Choose Contract A


Contract B 72,000
Contract C (504,000)
Contract D (1,656,000)

Minimax Regret

Opportunity cost table

Contact
Demand A B C D
(1008 - 72) (1008 - [504]) (1008 - [1656])
160,000 0 936,000 1,512,000 2,664,000
234,000 1,395,000 0 576,000 1,728,000
360,000 3,636,000 2,052,000 1,368,000 0
Max Regret = 3,636,000 2,052,000 1,512,000 2,664,000

Contract C minimise the maximum opportunity cost.


c) The 360,000 kgs ensure that all demand is met and the competitive
advantage will be held.

134
Question 2

BC must choose between one of two machines machine A and machine B.


Machine A is most suited to low-level demand whereas machine B is suited to
high-level demand. Assume there are only two possible demand levels low
and high and the estimated probability of each of these events is 0.5. The
estimated profits for each demand level are as follows:

Low Demand High Demand


Machine A $100,000 $160,000
Machine B $10,000 $200,000

Required:

a. On the basis of maximising expected value, advise BC whether it


should choose machine A or machine B.

b. There is a possibility of employing a firm of market consultants


who would be able to provide a perfect prediction of the actual
demand. What is the maximum amount the company should be
prepared to pay the consultants for the additional information?

a) EV Table

Demand Prob Mach A Mach B


Low 0.5 50,000 5,000
High 0.5 80,000 100,000
130,000 105,000

b) EV with perfect info 150,000


EV without perfect info 130,000
Exp value of perfect info 20,000

Low 100,000 x 0.5 = 50,000


High 200,000 x 0.5 = 100,000
150,000

Question 3

135
The probability of an organization making a profit of $180,000 next month is
half the probability of it making a profit of $75,000.

What is the expected profit for next month?

A. $110,000
B. $127,500
C. $145,000
D. $165,000

Question 4

Which of the following is correct with regard to expected values?

A. Expected values provide a weighted average of anticipated outcomes


B. The expected value will always equal one of the possible outcomes
C. Expected values will show the decision makers attitude to risk
D. The expected value will never equal one of the possible outcomes

Question 5

There is a 60% chance that a company will make a profit of $300,000 next
year and a 40% chance of making a loss of $400,000.

What is the expected profit or loss for next year?

A. $120,000 loss
B. $20,000 loss
C. $20,000 profit
D. $120,000 profit

136
CHAPTER 11
Budgeting

A budget is a quantitative detailed plan prepared for a specific time period. It


is normally expressed in financial terms and prepared for one year.

Forecasting is a technique used to arrive at estimates based on judgment and


experience.

The main objectives for producing budgets are: -

1. To Compel Planning

One of the key purposes of a budgeting system is to require planning to occur


so that the organisations objectives are achieved.

2. To Co-ordinate Activities

Budgeting is a method of bringing together the activities of all the different


departments into a common plan. If an advertising campaign is due to take
place in a company in three months time, it is important that the production
department know about the expected increase in sales so that they can scale
up production accordingly.

3. To Communicate Activities

The budgeting system facilitates communication within the organisation both


vertically (for example between senior and junior managers) and horizontally
(for example between different organisational functions).

4. To establish a system of control

One of the most important purposes of a budgeting system is to facilitate cost


control through the comparison of budgeted costs and actual costs. Variances
between budgeted and actual costs can be investigated in order to determine
the reason why actual performance has differed from what was planned.

5. To motivate managers to perform well

The budgeting system can influence the behaviour of managers and


employees, and may motivate them to improve their performance if the target
represented by the budget is set at an appropriate level.

137
6. To evaluate performance

Managerial performance is often evaluated by the extent to which budgetary


targets for which individual managers are responsible have been achieved
responsibility accounting. Managerial rewards such as bonuses or
performance-related pay can also be linked to achievement of budgetary
targets

The overall planning and control cycle is summarized in the diagram below:

Set mission

Identify objectives

Search for alternative courses of


action

Long-term
planning process Gather data about alternatives

Select course of action

Implement long-term plan in the


form of annual budgets

Budget
process Monitor actual results Control
process

Respond to divergences from


plan

138
Stage 1:- Set Mission

This involves establishing the broad overall aims and goals of the organization
its mission may be both economic and social. Most organizations now
prepare and publish their mission in a mission statement.

Mission statements often include the following information:


Purpose and aim(s) of the organization
The organization's primary stakeholders: clients/customers,
shareholders, congregation, etc.
How the organization provides value to these stakeholders, for example
by offering specific types of products and/or services

Stage 2:- Identify Objectives

This requires the company to specify objectives towards which it is working.


The objectives chosen must be quantified and have a timescale attached to
them. Objectives should be SMART: -

Specific
Measurable
Achievable
Relevant
Time limited

Stage 3:- Search for possible courses of action

A series of specific strategies should be developed. Strategy is the course of


action, including the specification of resources required, that the company will
adopt to achieve its specific objective.

To formulate its strategies, the firm will consider the products it makes and the
markets it serves. E.g. of strategies are: -
Developing new markets for existing products
Developing new products for existing markets
Developing new products for new markets

Stage 4:- Gathering data about alternatives and measuring pay-offs

Stage 5:- Select course of action

Having made decisions, long-term plans based on those decisions are


created.

139
Stage 6:- Implementation of short-term (operating) plans

This stage shows the move from long-term planning to short-term plans the
annual budget. The budget provides the link between the strategic plans and
their implementation in management decisions.

Stage 7:- Monitor actual outcomes

Detailed financial and other records of actual performance are compared with
budget targets (variance analysis).

Stage 8:- Respond to divergences from plan

This is the control process in budgeting, responding to divergences from plan


either through budget modifications or through identifying new courses of
action.

11.1 ACCA SYLLABUS GUIDE OUTCOME 1:


Explain how budgetary systems fit within the performance hierarchy.

As discussed in point 6 above, budgets provide benchmarks against which to


compare actual results. Through variance analysis, then the company needs
to develop corrective measures. However, budgets need to be flexible in
order to meet the changing needs of the business.

11.1.1 The Performance Hierarchy

140
Strategic Planning

Senior management formulate long-term (e.g. 5 to 10 years) objectives and


plans for an organization. Such plans include overall profitability, the
profitability of different segments of the business, capital equipment needs
and so on.

Tactical Planning

Senior management make medium-term, more detailed plans for the next
year, for e.g. decide how the resources of the business should be employed,
and to monitor how they are being and have been employed. An example
would be: - how many people should be employed next year?

Operational Planning

All managers are involved in making day-to-day decisions. 'Front-line'


managers such as foremen or senior clerks have to ensure that specific tasks
are planned and carried out properly within a factory or office.

Operational information is derived almost entirely from internal sources. It is


prepared frequently and is highly detailed. It is mainly quantitative.

If a manager achieves operational plans, it is more likely of meeting the


tactical objectives and ultimately the strategic goals.

How are planning and control inter-related?

Control involves measuring actual results and comparing them against the
original plan. Any deviation from plan requires control action to make the
results conform with the plan.

11.1.2 Behavioural aspects in budgeting

The purpose of a budgetary control system is to assist management in


planning and controlling the resources of their organisation by providing
appropriate control information. The information will only be valuable,
however, if it is interpreted correctly and used purposefully by managers and
employees. The correct use of control information therefore depends not only
on the content of the information itself, but also on the behaviour of its
recipients.

Goal congruence exists when managers working in their best interests also
act in harmony with the goals of the organisation as a whole.

141
A number of behavioural problems can arise: -

(a) The managers who set the budget or standards are often not the
managers who are then made responsible for achieving budget targets.

(b) The goals of the organisation as a whole, as expressed in a budget, may


not coincide with the personal aspirations of individual managers. This is
known as dysfunctional behaviour.

(c) When setting the budget, there may be budgetary slack (or bias). Budget
slack is a deliberate over-estimation of expenditure and/or under-estimation of
revenues in the budgeting process. This results in meaningless variances and
a budget which has no use for control purposes. It may also lead to the
misallocation of resources.

11.1.2.1 Participation in setting targets

Participation in the budgeting process will improve motivation and so will


improve the quality of budget decisions and the efforts of individuals to
achieve their budget targets. However, this may be time consuming and may
result in a wide range of targets which are seen as unfair.
There are basically two ways in which a budget can be set: from the top
down (imposed budget) or from the bottom up (participatory budget).

Top down budget

This is also called an authoritative or non-participative budget as it is set


without allowing the ultimate budget holder the opportunity to participate in the
budgeting process.

These budgets will begin with upper level management establishing


parameters under which the budget is to be prepared. Lower-level personnel
have very little input in setting the overall goals of the organization as they are
essentially reduced to doing the basic budget calculations consistent with
directives.

Advantages of a top down budget

1. Top-down budgets can set a tone for the organization. They signal
expected sales and production activity that the organization is
supposed to reach.
2. Budgets will be in line with corporate objectives.
3. Decisions taken by experienced managers.
4. Budgetary slack reduced.

142
Disadvantages of a top down budget

1. Lower-level managers may view the budget as dictatorial.


2. Such budgets can sometimes provide ethical challenges, as lower-level
managers may find themselves put in a position of ever-reaching to
attain unrealistic targets for their units.

Bottom up budget34

The budget holders have the opportunity to participate in setting their own
budgets. In fact, the lowest level organisational units are asked to submit their
estimates of expenditure for the next year. Senior management, meanwhile,
has made a forecast of the income it expects to receive. There maybe a
negative variance between the forecast revenue and the sum of the
departments budgets. The variance is resolved by lengthy discussions or
arbitrary decisions. This type of budget is also called participative budget.

Advantages of a bottom up budget

1. Improved employee morale and job satisfaction hence increased


motivation.
2. The budget is prepared by those who have the best knowledge of their
own specific areas of operation better information is pooled which
improves the accuracy of the budget
3. Increases managers understanding and commitment.
4. Better communication and co-ordination between departments.

Disadvantages of a bottom up budget

1. Generally more time consuming and expensive to develop and


administer.
2. Some managers may try to "pad" their budget, giving them more room
for mistakes and inefficiency - budgetary slack (bias).
3. Budgets may not be in line with corporate objectives dysfunctional
behaviour.
4. Decisions made by inexperienced managers.
5. The budgeting process may have to be started earlier than a non-
participative budget would need to start because of the length of time it
takes to complete the process.
6. Disagreements may occur between the staff involved, which may
cause delays and dissatisfaction.

34 Examined December 2013 Qs 3

143
Lecture Example 1

Which of the following statements about imposed budgets are correct?

i. Imposed budgets are likely to set realistic targets because senior


management have the best idea of what is achievable in each part of
the business.
ii. Imposed budgets can be less effective than budgets set on a
participative basis, because it is difficult for an individual to be
motivated to achieve targets set by someone else.
iii. Imposed budgets are generally quicker to prepare and finalise than
participative budgets.

A. (i) and (ii) only


B. (i) and (iii) only
C. (ii) and (iii) only
D. (iii) only

11.1.2.2 Setting the difficulty level of a budget

Targets will motivate employees if they are at the right level.

An expectations budget is a budget set at current achievable levels.


Therefore, it is unlikely to motivate managers to improve.

An aspirations budget is a budget set at a level which is higher than the


achievable level. Therefore, this will motivate managers to improve if it seen
as achievable.

The relationship between budget difficulty and actual performance

144
11.2 Budgeting in Public Sector Organisations vs. Private Sector
Organisations35

In the public sector, the objectives of the organisation are more difficult to
define in a quantifiable way than the objectives of a private company.

For e.g., a private companys objectives may be to maximise profit. This can
then be set out in the budget by aiming for a % increase in sales revenue and
perhaps the cutting of various costs.

On the other hand, if the public sector organisation is a hospital, then the
objectives may be largely qualitative, such as ensuring that all outpatients are
given an appointment within six weeks of being referred to the hospital. This is
difficult to define in a quantifiable way.

Just as objectives are difficult to define quantifiably, so too are the


organisations outputs. In a private company the output can be measured in
terms of sales revenue. In a hospital, on the other hand, it is difficult to define
a quantifiable relationship between inputs and outputs. What is easier to
compare is the relationship between how much cash is available for a
particular area and how much cash is actually needed. Therefore, budgeting
naturally focuses on inputs alone, rather than the relationship between inputs
and outputs.

Finally, public sector organisations are always under pressure to show that
they are offering good value for money, i.e. providing a service that is
economical, efficient and effective (value for money approach). Therefore,
they must achieve the desired results with the minimum use of resources.
This, in itself, makes the budgeting process more difficult.

11.3 ACCA SYLLABUS GUIDE OUTCOME 2:


Select and explain appropriate budgetary systems for an organisation,
including top-down, bottom-up, rolling, zero-base, activity-base,
incremental and feed-forward control.

Indicate the usefulness and problems with different budget types (zero-
base, activity-based, incremental, master, functional and flexible).

Both the top-down and bottom-up budgets have been discussed in the
previous section.

35 Examined December 2010 Qs 5a

145
11.3.1 Rolling Budget

A rolling budget is sometimes called a continuous budget. Here, a portion of


the budget period is replaced on a regular basis so that the overall budget
period remains unchanged. For example, with a budget period of one year, at
the end of each quarter a new quarter could be added to the end of the
budget period and the elapsed quarter could be deleted, so that the budget
will always be looking one year ahead.

A cash budget is often a rolling budget because of the need to keep tight
control of this area of financial management. A rolling budget is also
supported by the availability of cheap and powerful information processing via
personal computers and computer networks.

11.3.1.1 Advantages of rolling budgets are: -

1. They reduce the element of uncertainty in budgeting because they


concentrate detailed planning and control on short-term prospects
where the degree of uncertainty is much smaller.
2. They force managers to reassess the budget regularly, and to produce
budgets which are up to date in the light of current events and
expectations.
3. Planning and control will be based on a recent plan which is likely to be
far more realistic than a fixed annual budget made many months ago.
4. Realistic budgets are likely to have a better motivational influence on
managers.
5. There is always a budget which extends for several months ahead. For
example, if rolling budgets are prepared quarterly there will always be a
budget extending for the next 9 to 12 months. This is not the case
when fixed annual budgets are used.

11.3.1.2 Disadvantages of rolling budgets are: -

1. They involve more time, effort and money in budget preparation.


2. Frequent budgeting might have an off-putting effect on managers who
doubt the value of preparing one budget after another at regular
intervals.
3. Revisions to the budget might involve revisions to standard costs too,
which in turn would involve revisions to stock valuations. This could
replace a large administrative effort from the accounts department
every time a rolling budget is prepared.

146
Lecture Example 236

B Co produces quarterly rolling budgets and had forecast the costs of material
purchases for the next four quarters (quarters 1, 2, 3 and 4).

Purchases for quarter 1 were budgeted to be $220,000 and it was anticipated


that the cost of materials would rise at a rate of 2% per quarter.

At the end of quarter 1:


Actual material purchases were recorded at $210,000. This was due to a
change of material supplier during the quarter.
A revised estimate for the increase in material purchase costs was made.
The rise was now predicted to be only 1% per quarter.
The budget was updated.

What estimate for total annual material purchases should be recorded in the
updated budget?
A. $896,754
B. $852,684
C. $861,211
D. $1,071,211

11.3.2 Incremental Budget37

Incremental budgeting is a process whereby this years budget is set by


reference to last years actual results after an adjustment for inflation and
other incremental factors.

Benefits of incremental budgeting 38: -

1. It is easy to prepare and is therefore quick. Since it is easy to prepare, it is


also easily allocated to more junior members of staff.
2. As well as being easy to prepare, it is easy to understand.
3. Less preparation time leads to lower preparation costs.
4. Prevents conflict between departmental managers since a consistent
approach is adopted throughout the organisation.
5. The impact of change can be seen quickly.

There are problems involved with incremental budgeting:

36 September 2015 Examiners Report


37 June 2013 Qs 5b
38 Comparing budgeting techniques, April 2013, http://www.accaglobal.com/gb/en/student/acca-

qual-student-journey/qual-resource/acca-qualification/f5/technical-articles/comparing-budgeting-
techniques.html

147
1. It builds on wasteful spending (inefficiencies). If the actual figures for
this year include overspends caused by some form of error then the
budget for the next year would potentially include this overspend again.
2. It encourages organisations to spend up to the maximum allowed
(encourages slack) in the knowledge that if they dont do this then they
will not have as much to spend in the following years budget.
3. Assessing the amount of the increment can be difficult.
4. It is not appropriate in a rapidly changing business.
5. Can ignore the true (activity based) drivers of a cost leading to poor
budgeting.

Lecture Example 3

The following information relates to 2011:-

Bags produced 4,000


Total costs incurred $120,000
60% of these costs are variable

In 2012, all costs are expected to increase by 5%.

Required

What is the budgeted variable cost per bag in 2012?

11.3.3 Activity-Based Budget

Activity-based budgeting (ABB) would need a detailed analysis of costs and


cost drivers so as to determine which cost drivers and cost pools were to be
used in the activity-based costing system. However, whereas activity-based
costing uses activity-based recovery rates to assign costs to cost objects,
ABB begins with budgeted cost-objects and works back to the resources
needed to achieve the budget.

The budgeted activity levels are determined in the same way as for
conventional budgeting in that a sales budget and a production budget are
drawn up. ABB then determines the quantity of activity cost drivers (e.g.
number of purchase orders, number of set-ups) needed to support the
planned sales and production. Standard cost data would be compiled that
include details of the activity cost drivers required to produce a product or
number of products.

The resources needed to support the budgeted quantity of activity cost drivers
would then be determined (e.g. number of labour hours to process purchase
orders, number of maintenance hours needed to complete set-ups). This
resource need would then be matched against the available capacity (i.e.

148
number of purchase clerks to process purchase orders) to see whether any
capacity adjustment were needed.

11.3.3.1 Advantages of ABB are: -

1. Organisational resources are allocated more efficiently due to the


detailed cost and activity information obtained by implementing an ABB
system.
2. It avoids slack that is often linked to incremental budgeting due to its
detailed assessment of the activities and resources needed to support
planned sales and production.
3. In ABB the costs of support activities are not seen as fixed costs to be
increased by annual increments, but as depending to a large extent on
the planned level of activity.
4. It provides a useful basis for monitoring and controlling overhead costs,
by drawing management attention to the actual costs of activities and
comparing actual costs with what the activities were expected to cost.

11.3.3.2 Disadvantages of ABB are: -

1. A considerable amount of time and effort might be needed to establish


an ABB system, for example to identify the key activities and their cost
drivers.
2. ABB might not be appropriate for the organisation and its activities and
cost structures.
3. A budget should be prepared on the basis of responsibility centres, with
identifiable budget holders made responsible for the performance of
their budget centre. A problem with ABB could be to identify clear
individual responsibilities for activities.
4. It could be argued that in the short term many overhead costs are not
controllable and do not vary directly with changes in the volume of
activity for the cost driver. The only cost variances to report would be
fixed overhead expenditure variances for each activity.

Lecture Example 4

Berry Ltd. has prepared an activity-based budget for May 2011. The budgeted
costs are:

Activity Cost driver Budgeted cost

Set-up costs Number of set-ups $4000/set-up


Machining Number of machine hrs $5/machine hr
Receiving goods Number of deliveries recd $40/delivery
Ordering goods Number of orders done $20/order

149
Actual results for May 2011 were:

Activity Quantity Actual cost

Set-up costs 5 set-ups $19000


Machining 1280 machine hrs $6500
Receiving goods 100 deliveries recd $4500
Ordering goods 40 orders done $1000

Prepare a report for the month of May 2011, showing any variances
which have arisen during the month.

11.3.4 Feed-Forward Control

Feed-forward control is defined as the forecasting of differences between


actual and planned outcomes and the implementation of actions before the
event, to avoid such differences.

Whereas feedback is based on a comparison of historical actual results with


the budget for the period to date, feed-forward compares:
the target or objectives for the period, and
the actual results forecast.

Its advantages are: -

1. It informs managers of what is likely to happen unless control action is


taken.
2. It encourages managers to be proactive and deal with problems before
they occur.
3. Reforecasting on a monthly or continuous basis can save time when it
comes to completing a quarterly or annual budget.

Its disadvantages are: -

1. It may be time consuming as control reports must be produced


regularly.
2. It may require a sophisticated forecasting system, which might be
expensive.

11.3.5 Zero-Based Budgeting

Zero-based budgeting requires that activities be re-evaluated as part of the


budget process so that each activity, and each level of activity, can justify its
consumption of the economic resources available. This is in contrast to
incremental budgeting, where the current budget is increased to allow for

150
expected future conditions. Zero-based budgeting prevents the carrying
forward of past inefficiencies that can be a feature of incremental budgeting
and focuses on activities rather than departments or programmes. Each
activity is treated as though it was being undertaken for the first time and is
required to justify its inclusion in the budget in terms of the benefit expected to
be derived from its adoption.

11.3.5.1 Steps in zero-based budgeting: -

Zero-based budgeting involves three main stages 3940:

1. Activities are identified by managers. These activities are then


described in what is called a decision package. This decision
package: -
a. Analyses the cost of the activity
b. States its purpose
c. Identifies alternative methods of achieving the same purpose
d. Establishes performances measures for the activity
e. Assesses the consequence of not performing the activity at all or
of performing it at different levels.

This decision package is prepared at the base level, representing the


minimum level of service or support needed to achieve the
organisations objectives. Further incremental packages may then be
prepared to reflect a higher level of service or support.

2. Management will then rank all the packages in the order of decreasing
benefits to the organisation. This will help management decide what to
spend and where to spend it.

3. The resources are then allocated based on order of priority up to the


spending level41.

11.3.5.2 Advantages claimed for zero-based budgeting are 42: -

1. It eliminates the inefficiencies that can arise with incremental


budgeting.
2. It fosters a questioning attitude towards current activities rather than
just accepting the status quo.

39 Examined June 2015 Section B Qs 5a


40 Comparing budgeting techniques, April 2013, http://www.accaglobal.com/gb/en/student/acca -
qual-student-journey/qual-resource/acca-qualification/f5/technical-articles/comparing-budgeting-
techniques.html
41 F5 December 2010 Qs 5c and June 2013 Qs 5c
42 Examined June 2015 Section B Qs 5c

151
3. It focuses attention on the need to obtain value for money from the
consumption of organisational resources.
4. It leads to a more efficient allocation of resources.
5. All of the organisations activities and operations are reviewed in depth.
6. ZBB focuses attention on outputs in relation to value for money. This is
particularly important in the public sector where the 3 Es (economy,
efficiency and effectiveness) are often used to measure performance.

11.3.5.3 Disadvantages claimed for zero-based budgeting are: -

1. Departmental managers will not have the skills necessary to construct


decision packages. They will need training for this and training takes
time and money.
2. In a large organisation, the number of activities will be so large that the
amount of paperwork generated from will be unmanageable.
3. Ranking the packages can be difficult, since many activities cannot be
compared on the basis of purely quantitative measures. Qualitative
factors need to be incorporated but this is difficult.
4. The process of identifying decision packages, determining their
purpose, costs and benefits is massively time consuming and therefore
costly.
5. Since decisions are made at budget time, managers may feel unable to
react to changes that occur during the year. This could have a
detrimental effect on the business if it fails to react to emerging
opportunities and threats.

It could be argued that ZBB is more suitable for public sector than for private
sector organisations. This is because, firstly, it is far easier to put activities into
decision packages in organisations which undertake set definable activities.
Local government, for example, has set activities including the provision of
housing, schools and local transport.

Secondly, it is far more suited to costs that are discretionary in nature or for
support activities. Such costs can be found mostly in not for profit
organisations or the public sector, or in the service department of commercial
operations.

Since ZBB requires all costs to be justified, it would seem inappropriate to use
it for the entire budgeting process in a commercial organisation. Why take so
much time and resources justifying costs that must be incurred in order to
meet basic production needs? It makes no sense to use such a long-winded
process for costs where no discretion can be exercised anyway. Incremental
budgeting is, by its nature, quick and easy to do and easily understood. These
factors should not be ignored.43

43 F5 December 2010 Qs 5d

152
Lecture Example 5

Which of the following is the best description of zero based budgeting?

A. A method of budgeting which requires each cost item in the budget to


be specifically justified.
B. A method of budgeting which assumes a zero level of cost inflation
from the previous year.
C. A method of budgeting which requires under performing cost centres to
accept a budget allowance of zero.
D. A method of budgeting that by recognising cost behaviour patterns,
changes costs as volume of activity changes.

11.4 ACCA SYLLABUS GUIDE OUTCOME 3:


Explain the beyond budgeting model, including the benefits and
problems that may be faced if it is adopted in an organisation.

In recent years, there have been many changes in the business environment:
- shorter product lifecycles, advancement in technology, greater focus on
quality.

Unfortunately, the traditional budget serves only to stifle innovation and


responsiveness to change: - it prevents managers from responding quickly to
changes in the business environment.

Various commentators have identified the drawbacks of traditional budgets.


These traditional budgets: -

1. rarely focus on strategy and are often contradictory


2. are time consuming and costly to put together
3. constrain responsiveness and flexibility
4. often deter change
5. add little value, especially given the time taken to prepare them
6. focus on cost reduction rather than value creation
7. strengthen vertical command and control44.

As a result of these drawbacks, the Beyond Budgeting model was introduced


to try and resolve the weaknesses and limitations of traditional approaches to
budgets. Beyond budgeting does not introduce new tools or techniques. The
tools already exist e.g., balanced scorecards, rolling forecasts, customer
relationship management, benchmarking, shareholder value models,
enterprise wide information systems and activity based management amongst
others. It is a management philosophy based on a set of principles: -

44 CIMA, Beyond Budgeting, October 2007

153
Extracted from: -
http://www.cimaglobal.com/Documents/ImportedDocuments/betterbudgeting_joint.pdf

The benefits of the Beyond Budgeting model45: -

1. Goals are agreed via reference to external benchmarks as opposed to


internally-negotiated fixed targets. Managerial focus shifts from beating
other managers for a slice of resources to beating the competition.
2. It motivates people by giving them challenges, responsibilities and
clear values as guidelines. Rewards are team-based. This uses the
'know-how' of individuals and teams interfacing with the customer,
which in turn enables a far more rapid adaptation to changing market
needs.
3. It empowers operational managers to act by removing resource
constraints. Key ratios are set, rather than detailed line-by-line budgets.
4. It establishes customer-orientated teams that are accountable for
profitable customer outcomes.

45 Beyond Budgeting, August 2015, http://www.accaglobal.com/an/en/student/exam-support-


resources/professional-exams-study-resources/p5/technical-articles/beyond-budgeting.html

154
5. It creates transparent and open information systems throughout the
organisation, which should provide fast, open and distributed
information to facilitate control at all levels.

Problems of adopting Beyond Budgeting model: -

1. Managers should be clear what the expectations are and what they
have to do. They will need to be challenged and motivated.
2. It may be difficult for managers to change the budgeting system as they
would have been using the traditional budgeting system for a long time.
3. There is no one simple recipe to apply beyond budgeting. It will depend
on each companys culture, structure, history, IT infrastructure etc.

.
11.3.6 Master Budget

The master budget is a summary of all of the budgets which generally


comprises a budgeted income statement, a budgeted statement of financial
position and a budgeted cash flow statement.

Assuming that the level of demand is the principal budget factor, the various
functional, departmental and master budgets will be drawn up in the following
order.

11.3.7 Functional Budget

Functional budgets are prepared and consolidated to produce the master


budget. These would include raw materials budget, raw material usage and
purchases budgets, sales budget and production budget.

11.3.8 Fixed Budget

A fixed budget is one prepared in advance of the relevant budget period which
is not changed or amended as the budget period progresses. This budget
represents a periodic approach to budgeting, since a new budget is prepared
towards the end of the budget period for the subsequent budget period. In this
way, an organisation may set a new budget on an annual basis.

A fixed budget is likely to be useful in circumstances where the organisational


environment is relatively stable and can be predicted with a reasonable
degree of certainty.

11.3.9 Flexible Budget

155
A flexible budget is a budget which, by recognising different cost behaviour
patterns, is designed to change as volumes of output change.

Lecture Example 6

Budgeted production 7000 units


Budgeted overheads $280,000

40% of overheads are fixed and the remainder vary with total labour hours.
Fixed overheads are absorbed on a unit basis.

Labour skilled 4hrs / unit @ $15/hr


unskilled 6hrs/ unit @ $9/hr

Actual production 7250 units


Actual overheads
Variable $185,000
Fixed $105,000
Actual labour costs
Skilled $16.25/hr $568,750
Unskilled $8/hr $332,400

Actual labour hours taken per unit remained as budgeted.

Required

(a) Prepare a flexed budget for the actual activity for the year.

(b) Calculate the variances between the actual and flexed budget.

11.5 ACCA SYLLABUS GUIDE OUTCOME 4:


Describe the information used in budget systems and the sources of
the information needed.

Past data may be used as a starting point for the preparation of budgets but
other information from a wide variety of sources will also be used. Each
function of the organisation will be required to estimate revenue and
expenditure for the budget period. For example, marketing, personnel and
research and development.

The main sources of information for budgeting purposes are: -

1. Previous years actual results.


2. Other internal sources which may include managers knowledge.
3. Long-term requirements of individual customers, etc.

156
4. Estimates of costs of new products using work study techniques,
technical estimates from research and development, etc.
5. Statistical techniques such as linear regression may help to forecast
sales. EOQ may be used to forecast optimal inventory levels.
6. External sources of information may include suppliers' price lists,
estimates of inflation and exchange rate movements, strategic analysis
of the economic environment. Senior managers may incorporate
assumptions concerning competitor actions based on the analysis of
the market.

11.6 ACCA SYLLABUS GUIDE OUTCOME 5:


Explain the difficulties of changing a budgetary system.

It has been argued that traditional budgets are too rigid and prevent fast
response to changing conditions.

However, an organisation which decides to change its type of budget used, or


budgetary system, will face a number of difficulties. A change in the system
must be carefully planned and implemented.

a. Resistance by employees. Employees will be familiar with the current


system and may have built in slack so will not easily accept new
targets. New control systems that threaten to alter existing power
relationships may be thwarted by those affected.
b. Loss of control. Senior management may take time to adapt to the new
system and understand the implications of results.
c. Training. In order for the new budget to operate effectively, everyone
within the organisation will need to be fully trained. This is time-
consuming and expensive.
d. Costs of implementation. Any new system or process requires careful
implementation which will have cost implications.
e. Lack of accounting information. The organisation may not have the
systems in place to obtain and analyse the necessary information.

11.7 ACCA SYLLABUS GUIDE OUTCOME 6:


Explain how budget systems can deal with uncertainty in the
environment. 46

Uncertainty arises largely because of changes in the external environment


over which a company will sometimes have little control.

Causes of uncertainty in the budgeting process include:

a. Customers. They may decide to buy more or less than forecast

46 Examined June 2014 Qs 4d

157
b. Products/services. In the modern business environment, organisations
need to respond to customers' rapidly changing requirements.
c. Inflation and movements in interest and exchange rates.
d. Volatility in the cost of materials.
e. Competitors may steal some of an organisation's expected customers,
or some competitors' customers may change their buying allegiance.
f. Employees. They may not work as hard as was hoped, or they may
work harder.
g. Machines may break down unexpectedly and hence may fail to meet
production.
h. There may be political unrest (terrorist activity), social unrest (public
transport strikes) or minor or major natural disasters (storms, floods)
which affect productivity

Rolling budgets and flexible budgets are a way of trying to reduce the element
of uncertainty in the plan. There are other planning methods which try to
analyse the uncertainty such as probabilistic budgeting, sensitivity analysis
(what if?), scenario planning and simulation. These methods are suitable
when the degree of uncertainty is quantifiable from the start of the budget
period and actual results are not expected to go outside the range of these
expectations.

11.8 ACCA SYLLABUS GUIDE OUTCOME 7:


Explain the benefits and dangers inherent in using spreadsheets in
budgeting.

A spreadsheet is a type of general purpose software package with many


business applications, not just accounting ones. It can be used to build a
model, in which data is presented in these rows and columns, and it is up to
the model builder to determine what data or information should be presented
in it, how it should be presented and how the data should be manipulated by
the spreadsheet program.

Spreadsheets provide a tool for calculating, analysing and manipulating


numerical data. Spreadsheets make the calculation and manipulation of data
easier and quicker.

For example, the spreadsheet above has been set up to calculate the totals
automatically. If you changed your estimate of sales for one of the
departments, the totals will change automatically.

Spreadsheets can be used for a wide range of tasks. Some common


applications of spreadsheets are:

1. Management accounts

158
2. Cash flow analysis and forecasting
3. Reconciliations
4. Revenue analysis and comparison
5. Cost analysis and comparison
6. Budgets and forecasts

Spreadsheets have disadvantages if they are not properly used:

1. A minor error in the design of a model at any point can affect the
validity of data throughout the spreadsheet. Such errors can be very
difficult to trace.
2. Even if it is properly designed in the first place, it is very easy to corrupt
a model by accidentally changing a cell or inputting data in the wrong
place.
3. It is possible to become over-dependent on them, so that simple one-
off tasks that can be done in seconds with a pen and paper are done
on a spreadsheet instead.
4. The possibility for experimentation with data is so great that it is
possible to lose sight of the original intention of the spreadsheet.
5. Spreadsheets cannot take account of qualitative factors since they are
invariably difficult to quantify. Decisions should not be made on the
basis of quantitative information alone.

Further Questions

Question 1

The following statements relate to the participation of junior management in


the budgeting setting process:

(i) it speeds up the budget setting process.


(ii) it increases their commitment to budgets.
(iii) it incorporates their knowledge into budgets.
(iv) it improves their morale and motivation.

Which of the statements are true?

A. (i), (ii) and (iii)


B. (i), (ii) and (iv)
C. (ii), (iii) and (iv)
D. (ii) and (iii) only

Question 2

Which of the following statements about participative budgeting is/are false?

A. Morale and motivation are improved

159
B. They may cause managers to introduce budgetary slack
C. They are quicker to produce than non-participative budgets

Question 347

The following statements have been made about zero based budgeting:

(1) Employees will focus on eliminating wasteful expenditure


(2) Short-term benefits could be emphasised over long-term benefits

Which of the above statements is/are true?

A. 1 only
B. 2 only
C. Neither 1 nor 2
D. Both 1 and 2

Question 448

The following statements have been made about changing budgetary


systems:

(1) The costs of implementation may outweigh the benefits


(2) Employees will always welcome any new system which improves planning
and control within the organisation

Which of the above statements is/are true?

A. 1 only
B. 2 only
C. Neither 1 nor 2
D. Both 1 and 2

Question 5

Which of the following statements are correct?

i. A fixed budget is a budget that considers all of an organisations costs


and revenues for a single level of activity
ii. A flexible budget is a budget that is produced during the budget period
to recognize the effects of any changes in prices and methods of
operation that have occurred
iii. Organizations can use budgets to communicate objectives to their
managers
A. (i) and (ii) only

47 Specimen Exam Applicable from December 2014


48 Specimen Exam Applicable from December 2014

160
B. (i) and (iii) only
C. (ii) and (iii) only
D. All of them

Question 6

Are the following statements, which refer to different types of budgets, true or
false?

Statement 1 An annual budget that can be broken down into monthly


budgets, which differ depending on the number of working days in each
month, is called a flexible budget.

Statement 2 An annual budget set before the start of a year based on


estimated sales and production volumes is called a fixed budget.

Statement 1 Statement 2
A. True true
B. False false
C. True false
D. False true

161
CHAPTER 12
Quantitative Analysis in
Budgeting
This chapter will be looking at five quantitative techniques: -

1. The high-low method


2. Least squares linear regression
3. Learning curves

12.1 ACCA SYLLABUS GUIDE OUTCOME 1:


Analyse fixed and variable cost elements from total cost data using
high/low

There are two main methods which analyse semi-variable costs into their fixed
and variable elements: -

High/low method
Least squares regression (this will not be directly examined in F5 from
2013 onwards)

12.1.1 High-low method

The main steps are: -

1. Review records of costs in previous periods.

Select the period with the highest activity level.


Select the period with the lowest activity level.

2. Find the variable cost per unit

Total cost at high activity level - total cost at low activity level
Total units at high activity level - total units at low activity level

3. Find the fixed costs

Total cost at high activity level (Total units at high activity level Variable
cost per unit)

162
Advantages of the High-Low Method

1. Easy to use
2. Easy to understand
3. Quick method

Limitations of the High-Low Method

1. It relies on historical cost data predictions of future costs may not be


reliable.
2. It assumes that the activity level is the only factor affecting costs.
3. It uses only two values to predict costs all data falling between the
highest and lowest values are ignored.
4. Bulk discounts may be available at large quantities.

Lecture Example 1

The following table shows the number of units produced each month and the
total cost incurred:

Units Cost
($)
January 100 40,000
February 400 65,000
March 200 45,000
April 700 85,000
May 600 70,000
June 500 70,000
July 300 50,000

Estimate the variable cost per unit, and the fixed cost per month.

12.1.2 Regression Analysis

Even though it will not be directly examinable from 2013 onwards, lets have a
look at the main points: -

Two variables are said to be correlated if a change in the value of one variable
is accompanied by a change in the value of another variable. Examples of
variables which might be correlated are:

A person's height and weight


The distance of a journey and the time it takes to make it

One way of showing the correlation between two related variables is on a


scattergraph or scatter chart, plotting a number of pairs of data on the graph.

163
E.g. a scattergraph showing total costs incurred at various output levels

A scattergraph can be used to make an estimate of fixed and variable costs


by drawing a line of best fit through the points which represents all the points
plotted.

The line of best fit will be of the form

y = a + bx

where a = fixed costs


b = variable costs

12.2 ACCA SYLLABUS GUIDE OUTCOMES 2 and 3:


Estimate the learning rate and the learning effect
Apply the learning curve to a budgetary problem, including calculations
on steady states

Learning Curve Theory is concerned with the idea that when a new job,
process or activity commences for the first time, it is likely that the workforce
involved will not achieve maximum efficiency immediately. Repetition of the
task is likely to make the people more confident and knowledgeable and will
eventually result in a more efficient and rapid operation. Eventually the
learning process will stop after continually repeating the job (this is known as
steady state). As a consequence the time to complete a task will initially
decline and then stabilise once efficient working is achieved.

The cumulative average time per unit is assumed to decrease by a constant


percentage every time that output doubles. Cumulative average time refers to
the average time per unit for all units produced so far, from and including the
first one made.

The following is an example of an 80% learning curve: - the cumulative


average time required per unit of output is reduced to 80% of the previous
cumulative average time when output is doubled. When output is low, the
learning curve is really steep but the curve becomes flatter as cumulative

164
output increases, with the curve eventually becoming a straight line when the
learning effect ends.

The learning curve effect can be calculated by:


1. reducing cumulative average time by the learning rate each time output
doubles in a table:
2. using the formula:
y = axb
where
y = cumulative average time (or average cost) per unit or per batch to produce
x units
a = time (or cost) taken for the first unit or batch
b = log r/log 2 (r = index of learning, expressed as a decimal)
x = cumulative output in units or in batches

Application of the Learning Curve Theory

1. the activity is labour intensive (rather than highly mechanized)


2. a repetitive process for each unit
3. low turnover of labour
4. early stages of production
5. no prolonged breaks in production

Cessation of the Learning Curve Theory The Steady State

165
The learning effect will only apply for a certain range of production. Once the
steady state is reached, the direct labour hours will not reduce any further and
this will become the basis on which the budget is produced.

The steady state will occur when: -

1. machine efficiency restricts further improvements


2. machines have reached the limits of safe running speeds
3. there is a limit to human dexterity
4. some processes cannot be speeded up any more (for e.g. a chemical
reaction)
5. new, inexperienced staff will replace practiced ones from time to time,
slowing things down again.

The importance of the learning curve effect49

Learning curve models enable users to predict how long it will take to
complete a future task. Management accountants must therefore be sure to
take into account any learning rate when they are carrying out planning,
control and decision-making. If they fail to do this, serious consequences will
result. Eg. A company is introducing a new product on the market. The
company wants to make its price as attractive as possible to customers but
still wants to make a profit, so it prices it based on the full absorption cost plus
a small 5% mark-up for profit. The first unit of that product may take one hour
to make. If the labour cost is $15 per hour, then the price of the product will
be based on the inclusion of that cost of $15 per hour. Other costs may total
$45. The product is therefore released onto the market at a price of $63.
Subsequently, it becomes apparent that the learning effect has been ignored
and the correct labour time per unit should is actually 0.5 hours. It is obvious
that the product will have been launched onto the market at a price which is
far too high. This may mean that initial sales are much lower than they
otherwise would have been and the product launch may fail. Worse still, the
company may have decided not to launch it in the first place as it believed it
could not offer a competitive price.

Why is the learning curve important in planning and control? If standard


costing is to be used, it is important that standard costs provide an accurate
basis for the calculation of variances. If standard costs have been calculated
without taking into account the learning effect, then all the labour usage
variances will be favourable because the standard labour hours that they are
based on will be too high. This will make their use for control purposes
pointless.
Finally, it is worth noting that the use of learning curve is not restricted to the
assembly industries it is traditionally associated with. It is also used in other
49The Learning Rate and Learning Effect http://www.accaglobal.com/ng/en/student/acca -qual-
student-journey/qual-resource/acca-qualification/f5/technical-articles/the-learning-rate-and-
learning-effect.html

166
less traditional sectors such as professional practice, financial services,
publishing and travel. In fact, research has shown that just under half of users
are in the service sector.

Lecture Example 2

The time taken to produce the first unit is 100 hours.

There is a learning rate of 75%.

How long will it take to produce the next 7 units?

Lecture Example 3

Straws Ltd has just produced the first full batch of Product Exe taking 200
hours.

There is a learning rate of 85%.

(a) How long will it take to produce the next 15 batches?

(b) Straws expects that after the 30 th batch has been produced, the
steady state is reached.

From the 31st batch onwards, each batch will take the same time as
the 30th batch. What time per batch should be budgeted for the 31st
batch?

Lecture Example 450

P Co operates a standard costing system. The standard labour time per batch
for its newest product was estimated to be 200 hours, and resource allocation
and cost data were prepared on this basis.

The actual number of batches produced during the first six months and the
actual time taken to produce them is shown below:

Incremental number Incremental labour


Month of batches produced hours taken to
each month produce the batches

50 The Learning Rate and Learning Effect, January 2014,


http://www.accaglobal.com/ng/en/student/acca-qual-student-journey/qual-resource/acca-
qualification/f5/technical-articles/the-learning-rate-and-learning-effect.html

167
June 1 200
July 1 152
August 2 267.52
September 4 470.8
October 8 1090.32
November 16 2180.64

Required
(a) Calculate the monthly learning rate that arose during the period.
(b) Identify when the learning period ended and briefly discuss the
implications of this for P Co.

Lecture Example 551

The first batch of a new product took six hours to make and the total time for
the first 16 units was 42.8 hours, at which point the learning effect came to an
end.

Calculate the rate of learning.

12.3 ACCA SYLLABUS GUIDE OUTCOME 4:


Discuss the reservations with the learning curve

The learning curve theory is likely to have consequences in a number of


areas: -

1. A standard costing system would need to set standard labour times


after the learning curve had reached a plateau.
2. A budget will need to incorporate a learning cost factor until the plateau
is reached.
3. A budgetary control system incorporating labour variances will have to
make allowances for the anticipated time changes.
4. Identification of the learning curve will permit the company to better
plan its marketing, work scheduling, recruitment and material
acquisition activities.
5. The decline in labour costs will have to be considered when estimating
the overhead apportionment rate.
6. As the employees gain experience they are more likely to reduce
material wastage.

The learning curve theory has a number of limitations: -

51 The Learning Rate and Learning Effect, January 2014,


http://www.accaglobal.com/ng/en/student/acca -qual-student-journey/qual-resource/acca-
qualification/f5/technical-articles/the-learning-rate-and-learning-effect.html

168
1. The stable conditions necessary for the learning curve to take place
may not be present unplanned changes in production techniques or
labour turnover will cause problems and affect the learning rate.
2. The employees need to be motivated, agree to the plan and keep to
the learning schedule these assumptions may not hold.
3. Accurate and appropriate learning curve data may be difficult to
estimate.
4. Inaccuracy in estimating the initial labour requirement for the first unit.
5. Inaccuracy in estimating the output required before reaching a steady
state time rate.
6. It assumes a constant rate learning factor.

Further Questions

Question 1

Berry has recorded the following costs over the last six months:

Month Total cost Units produced


000 000
1 74.00 3.00
2 72.75 1.75
3 73.25 2.00
4 75.00 2.50
5 69.50 1.50
6 72.75 2.00

Using the high low method what would be the total cost equation?

A. Total cost = 61,250 + 5.5 x Quantity


B. Total cost = 65,000 + 3 x Quantity
C. Total cost = 70,250 + 1.25 x Quantity
D. Total cost = 71,000 + 1 x Quantity

Question 252

Tech World is a company which manufactures mobile phone handsets. From


its past experiences, Tech World has realised that whenever a new design
engineer is employed, there is a learning curve with a 75% learning rate which
exists for the first 15 jobs.
A new design engineer has just completed his first job in five hours.

Note: At the learning rate of 75%, the learning factor (b) is equal to 0415.

52 Specimen Exam Applicable from December 2014

169
How long would it take the design engineer to complete the sixth job?

A. 2377 hours
B. 1442 hours
C. 2564 hours
D. 5 hours

170
CHAPTER 13
Budgeting and Standard Costing
13.1 ACCA SYLLABUS GUIDE OUTCOME 1:
Explain the use of standard costs

A standard cost is a predetermined estimated unit cost of a product or service.


Therefore, a standard cost represents a target cost.

Standard costing has a variety of uses: -


1. It is useful for planning, control and motivation.
2. It is used to value inventories and cost production for cost accounting
purposes.
3. It acts as a control device by establishing standards (planned costs),
highlighting activities that are not conforming to plan and thus alerting
management to areas which may be out of control and in need of
corrective action.

13.2 ACCA SYLLABUS GUIDE OUTCOME 2:


Outline the methods used to derive standard costs and discuss the
different types of cost possible

A standard cost is based on technical specifications for the materials, labour


time and other resources required and the prices and rates for the materials
and labour.

A standard cost card shows full details of the standard cost of each product.

171
Standard Cost Card Product A

$ $
Direct Material x kgs / ltrs x
Direct Labour x hrs @ $x x
Direct Expenses x
Standard Direct C ost (Prime Cost) x
Variable production overheads x
Standard variable cost of (Marginal
production x Costing)
Fixed production overhead x
(Absorption
Standard full production cost x Costing)
Administration & marketing
overhead x
Standard cost of sale x
Standard profit x
Standard sales price X

There are four main types of cost standards: -

1. Basic standards these are long-term standards which remain


unchanged over a period of years. They are used to show trends over
time. Basic standards may become increasingly easy to achieve as
time passes and hence, being undemanding, may have a negative
impact on motivation. Standards that are easy to achieve will give
employees little to aim at.

2. Ideal standards these standards are based upon perfect operating


conditions. Therefore, they include no wastage, no scrap, no
breakdowns, no stoppages, no idle time. Since perfect operating
conditions are unlikely to occur for any significant period, ideal
standards will be very demanding and are unlikely to be accepted as
targets by the staff involved as they are unlikely to be achieved. Using
ideal standards as targets is therefore likely to have a negative effect
on employee motivation.

3. Attainable standards these standards are based upon efficient but


not perfect operating conditions. These standards include allowances
for the fatigue, machine breakdown and normal material losses.
Attainable standards motivate performance as they can be achieved

172
can be used for product costing, cost control, inventory valuation,
estimating and as a basis for budgeting.

4. Current standards these standards are based on current level of


efficiency and incorporate current levels of wastage, inefficiency and
machine breakdown. They do not provide any incentive to improve on
the current level of performance. Their impact on motivation will be a
neutral one. Current standards are useful during periods of high
inflation.

13.3 ACCA SYLLABUS GUIDE OUTCOME 3:


Explain and illustrate the importance of flexing budgets in performance
management
Prepare a flexed budget and comment on its usefulness

A flexible budget is a budget which, by recognising different cost behaviour


patterns, is designed to change as volume of activity changes.

A flexed budget is a budget prepared to show the revenues, costs and profits
that should have been expected from the actual level of production and sales.

Budgetary control involves drawing up budgets for the areas of responsibility


for individual managers and of regularly comparing actual results against
expected results. The differences between actual results and expected results
are called variances and these are used to provide a guideline for control
action by individual managers.

Lecture Example 1

From the following information, prepare a budgetary control statement on a


flexible budget basis.

Activity Level 60% 70% 90%


$ $ $

Direct materials 60,000 70,000 90,000


Direct labour 48,000 56,000 72,000
Production overhead 58,600 64,000 71,200

During 2011, actual activity was 4,200 units (which was equal to 75% activity)
and actual costs were:

173
$
Direct materials 44,000
Direct labour 58,000
Production overhead _60,000
162,000

13.4 ACCA SYLLABUS GUIDE OUTCOME 4:


Explain and apply the principle of controllability in the performance
management system

The principle of controllability is that managers of responsibility centres should


only be held accountable for costs over which they have some influence.

Budgetary control is based around a system of budget centres. Each budget


centre will have its own budget and a manager will be responsible for
managing the budget centre and ensuring that the budget is met.

Responsibility accounting is a system of accounting that segregates revenue


and costs into areas of personal responsibility in order to monitor and assess
the performance of each part of an organisation.
Responsibility accounting attempts to associate costs, revenues, assets and
liabilities with the managers most capable of controlling them. As a system of
accounting, it therefore distinguishes between controllable and uncontrollable
costs.

Most variable costs within a department are thought to be controllable in the


short term because managers can influence the efficiency with which
resources are used, even if they cannot do anything to raise or lower price
levels.

Some costs are non-controllable, such as increases in expenditure items due


to inflation.

Other costs are controllable, but in the long term rather than the short term.
For example, production costs might be reduced by the introduction of new
machinery and technology, but in the short term, management must attempt
to do the best they can with the resources and machinery at their disposal.

Managers should only be held accountable for costs over which they have
some influence. They may become demotivated if they are made responsible
for non-controllable costs.

174
Further Questions

Question 1

A basic standard is:

A. A standard set at an ideal level, which makes no allowance for


normal losses, waste and machine downtime.
B. A standard which assumes an efficient level of operation, but which
includes allowances for factors such as normal loss, waste and
machine downtime.
C. A standard which is kept unchanged over a period of time.
D. A standard which is based on current price levels.

Question 253

The following statements have been made about different types of standards
in standard costing systems:

(1) Basic standards provide the best basis for budgeting because they
represent an achievable level of productivity.

(2) Ideal standards are short-term targets and useful for day-to-day control
purposes.

Which of the above statements is/are true?

A. 1 only
B. 2 only
C. Neither 1 nor 2
D. Both 1 and 2

53 Specimen Exam Applicable from December 2014

175
CHAPTER 14
Basic Variance Analysis
Although the syllabus for 2013 is excluding Basic Variances and Operating
Statements, an overview of these variances is still being included as these
are required to be able to work out the advanced variances.

14.1 Sales Variances

Total Sales Variance

Sales Price Sales Volume


Variance Variance

The sales price variance shows the effect on profit of selling at a different
price from that expected.

The sales price variance = Actual units should have sold for $x
Actual units did sell $x
Sales Price Variance $ x (F/A)

The sales volume variance = Budgeted sales volume x units


(absorption costing) Actual sales volume x units
Sales Volume Variance in units x units (F/A)
x standard profit per unit $x
Sales Volume Variance in $ $x (F/A)

176
The sales volume variance = Budgeted sales volume x units
(marginal costing) Actual sales volume x units
Sales Volume Variance in units x units (F/A)
x standard contribution per unit $x
Sales Volume Variance in $ $x (F/A)

The sale price variance is a measure of the effect on expected profit of a


different selling price to standard selling price. It is calculated as the difference
between what the sales revenue should have been for the actual quantity
sold, and what it was.

The sales volume profit variance is the difference between the actual units
sold and the budgeted (planned) quantity, valued at the standard profit (under
absorption costing) or at the standard contribution (under marginal costing)
per unit. In other words, it measures the increase or decrease in standard
profit as a result of the sales volume being higher or lower than budgeted
(planned).

Possible causes of sales variances: -

1. unplanned price increases


2. unplanned price reduction to attract additional business
3. unexpected fall in demand due to recession
4. increased demand due to reduced price
5. failure to satisfy demand due to production difficulties

14.2 Materials Variances

177
The direct material total variance can be subdivided into the direct material
price variance and the direct material usage variance.

Total Materials Variance

Materials Price Materials Usage


Variance Variance

The direct material total variance = actual units should have cost $x
actual units did cost $x
Direct Material Total Variance $ x (F/A)

The direct material price variance = actual kgs should have cost $x
actual kgs did cost $x
Direct Material Price Variance $ x (F/A)

Actual units should have


The direct material usage variance = used x kgs
Actual units did use x kgs
Usage Variance in kgs x kgs (F/A)
x standard cost per kg $x
Usage Variance in $ $x (F/A)

The direct material total variance is the difference between what the output
actually cost and what it should have cost, in terms of material.

The direct material price variance calculates the difference between the
standard cost and the actual cost for the actual quantity of material used or
purchased. In other words, it is the difference between what the material did
cost and what it should have cost.
The direct material usage variance is the difference between the standard
quantity of materials that should have been used for the number of units

178
actually produced, and the actual quantity of materials used, valued at the
standard cost per unit of material. In other words, it is the difference between
how much material should have been used and how much material was used,
valued at standard cost.

Variance Favourable Adverse


Material price Unforeseen discounts received Price increase
More care taken in purchasing Careless purchasing
Change in material standard Change in material standard
Material usage Material used of higher quality than Defective material
Standard Excessive waste
More effective use made of material Theft
Errors in allocating material to jobs Stricter quality control
Errors in allocating material to jobs

14.3 Labour Variances

The total labour variance can be subdivided between labour rate variance and
labour efficiency variance.

Total Labour Variance

Labour Rate Labour Efficiency


Variance Variance

The direct labour total variance = Actual units should have cost $x
Actual units did cost $x
Direct Labour Total Variance $ x (F/A)

The direct labour rate variance = Actual hrs should have cost $x
Actual hrs did cost $x

179
Direct Labour Rate Variance $ x (F/A)

Actual units should have


The direct labour efficiency variance = taken x hrs
Actual units did take x hrs
Efficiency Variance in hrs x hrs (F/A)
x standard rate per hr $x
Efficiency Variance in $ $x (F/A)

The direct labour total variance is the difference between what the output
should have cost and what it did cost, in terms of labour.

The direct labour rate variance is the difference between the standard cost
and the actual cost for the actual number of hours paid for. In other words, it is
the difference between what the labour did cost and what it should have cost.

The direct labour efficiency variance is the difference between the hours that
should have been worked for the number of units actually produced, and the
actual number of hours worked, valued at the standard rate per hour. In other
words, it is the difference between how many hours should have been worked
and how many hours were worked, valued at the standard rate per hour.

Variance Favourable Adverse


Labour rate Use of apprentices or other workers Wage rate increase
at a rate of pay lower than standard Use of higher grade labour
The idle time variance is always
Idle time adverse Machine breakdown
Non-availability of material
Illness or injury to worker
Lost time in excess of standard
Labour efficiency Output produced more quickly than allowed
expected because of work Output lower than standard set
motivation, because of
better quality of equipment or deliberate restriction, lack of
materials, training, or
or better methods. sub-standard material used
Errors in allocating time to jobs Errors in allocating time to jobs

14.4 Variable Overhead Variances

180
The variable production overhead total variance can be subdivided into the
variable production overhead expenditure variance and the variable
production overhead efficiency variance (based on actual hours).

Total Variable Overhead Variance

Variable overhead Variable overhead


expenditure efficiency
variance variance

Variable overhead total variance = Actual units should have cost $x


Actual units did cost $x
Var Overhead Total Variance $ x (F/A)

Variable overhead expenditure variance = Actual hrs should cost $x


Actual hrs did cost $x
Var Overhead Exp Variance $ x (F/A)

Actual units should have


Variable overhead efficiency variance = taken X hrs
Actual units did take X hrs
Efficiency Variance in hrs X hrs (F/A)
x standard rate per hr $x
Efficiency Variance in $ $x (F/A)

The variable production overhead expenditure variance is the difference


between the amount of variable production overhead that should have been
incurred in the actual hours actively worked, and the actual amount of variable
production overhead incurred.

The variable production overhead efficiency variance is exactly the same in


hours as the direct labour efficiency variance, but priced at the variable
production overhead rate per hour.

181
Variance Favourable Adverse
Variable overhead Savings in costs incurred Increase in cost of overheads used
expenditure More economical use of overheads Excessive use of overheads
Change in type of overheads used
Labour force working less
Variable overhead Labour force working more efficiently efficiently
efficiency (favourable labour efficiency) (adverse labour efficiency)
Better supervision or staff training Lack of supervision

14.5 Fixed Overhead Variances

The fixed production overhead total variance can be subdivided into an


expenditure variance and a volume variance. The fixed production overhead
volume variance can be further subdivided into an efficiency and capacity
variance.

Total Fixed Overhead Variance

Fixed overhead Fixed overhead


expenditure volume
variance variance

Fixed overhead Fixed


capacity overhead
variance efficiency
variance

Fixed overhead total variance = Overhead incurred $x


Overhead absorbed $x
Fix Overhead Total Variance $ x (F/A)

182
budgeted overhead
Fixed overhead expenditure variance = expenditure $x
actual overhead expenditure $x
Fix Overhead Exp Variance $ x (F/A)

Fixed overhead volume variance = actual units produced x units


budgeted units produced x units
Volume Variance in units x units (F/A)
x standard rate per unit $x
Volume Variance in $ $x (F/A)

The volume efficiency variance is calculated in the same way as the labour
efficiency variance.

actual units should have


Fixed overhead vol efficiency variance = taken x hrs
Actual units did take x hrs
Vol Efficiency Variance in hrs x hrs (F/A)
x standard OAR rate per hr $x
Vol Efficiency Variance in $ $x (F/A)

The volume capacity variance is the difference between the budgeted hours of
work and the actual active hours of work (excluding any idle time).

Fixed overhead vol capacity variance = budget hours of work X hrs


actual hours of work X hrs
Vol Capacity Variance in hrs X hrs (F/A)
x standard OAR rate per hr $x
Vol Capacity Variance in $ $x (F/A)

Fixed overhead total variance is the difference between fixed overhead


incurred and fixed overhead absorbed. In other words, it is the under or over-
absorbed fixed overhead.

Fixed overhead expenditure variance is the difference between the budgeted


fixed overhead expenditure and actual fixed overhead expenditure.
Fixed overhead volume variance is the difference between actual and
budgeted (planned) volume multiplied by the standard absorption rate per
unit.

183
Fixed overhead efficiency variance is the difference between the number of
hours that actual production should have taken, and the number of hours
actually taken (that is, worked) multiplied by the standard absorption rate per
hour.

Fixed overhead capacity variance is the difference between budgeted


(planned) hours of work and the actual hours worked, multiplied by the
standard absorption rate per hour.

Variance Favourable Adverse


Fixed overhead Savings in costs incurred Increase in cost of services used
Expenditure Changes in prices relating to fixed Excessive use of services
overhead expenditure Change in type of services used
Fixed overhead volume Labour force working more efficiently Labour force working less efficiently
Efficiency Lost production through strike
Fixed overhead volume Labour force working overtime Machine breakdown, strikes, labour
Capacity Shortages

Illustration 1

Hard Work Ltd has prepared the following standard cost card for its product
HW: -

$ per unit
Materials (4kg at $4 per kg) 16
Labour (5 hrs at $6 per hr) 30
Variable overheads (5hrs at $2 per hr) 10
Fixed overheads (5 hrs at $3 per hr) 15
71

Budgeted selling price is $76 per unit.

Budgeted production and sales 10,000 units

The actual results are as follows:

Sales and production 10,500 units for $835,000

$
Materials (38,000 kg) 159,600
Labour (48,000 hrs) 310,000
Variable overheads 100,000

184
Fixed overheads 170,000

Required:

Calculate variances in as much detail as possible.

Sales Price Variance

10,500 units should earn 798,000 (x76)


did earn 835,000
37,000 (F)

Sales Volume Profit Variance

actual sales 10,500


budgeted sales 10,000
500 (F)
x5
2,500 (F)

Sales Volume Contribution Variance

500 (F) x 20 = 10,000 (F)

Material Price Variance

38,000 kg should cost 152,000 (x4)


did cost 159,600
7,600 (A)

Material Usage Variance

kg
10,500 units should use 42,000 (x4)
did use 38,000
4,000 (F)
x $4
16,000 (F)

Labour Rate Variance

48,000 hrs should cost 288,000 (x6)


did cost 310,000

185
22,000 (A)

Labour Efficiency Variance

10,500 units should take 52,500 (x5)


did take 48,000
4,500 (F)
X $6
27,000 (F)

Variable Overheads Expenditure Variance

48,000 hrs should cost 96,000 (x2)


did cost 100,000
_ 4,000 (A)

Variable Overheads Efficiency Variance

4,500 x $2 = 9,000 (F)

Fixed Overheads Volume Variance

actual units 10,500


budgeted units 10,000
500 (F)
x 15
7,500 (F)

Fixed Overhead Expenditure Variance

actual 170,000
budgeted (15 x 10,000) 150,000
20,000 (A)

Fixed Overhead Capacity Variance

budgeted hours (10,000 x 5) 50,000


actual hours 48,000
2,000 (A)
x $3
6,000 (A)

Fixed Overhead Efficiency Variance

4,500 x $3 = $13,500 (F)

186
Operating Statement Absorption Costing

Budgeted profit (10,000 x 5) 50,000


Sales volume variance 2,500 (F)
Sales price variance 37,000 (F)
89,500 (F)

Cost variances Favourable Adverse


Material price 7,600
usage 16,000
Labour rate 22,000
efficiency 27,000
Variable ohead expenditure 4,000
efficiency 9,000
Fixed ohead expenditure 20,000
efficiency 13,500
capacity 6,000
65,500 59,600 5,900 (F)
Actual profit 95,400

Operating Statement Marginal Costing

Budgeted contribution (10,000 x 20) 200,000


Sales volume variance 10,000 (F)
Sales price variance 37,000 (F)
247,000 (F)

Cost variances Favourable Adverse


Material price 7,600
usage 16,000
Labour rate 22,000
efficiency 27,000
Variable ohead expenditure 4,000
efficiency 9,000 _____
52,000 33,600 18,400 (F)
Actual contribution 265,400
Budgeted fixed overhead 150,000
Expenditure variance 20,000 (A)
Actual fixed overheads 170,000
Actual profit 95,400

When should variances be investigated?

When deciding which variances to investigate, the following factors should be


considered:

187
1. Reliability and accuracy of the figures. Mistakes in calculating budget
figures, or in recording actual costs and revenues, could lead to a
variance being reported where no problem actually exists (the process
is actually in control).

2. Materiality. The size of the variance may indicate the scale of the
problem and the potential benefits arising from its correction.

3. Possible interdependencies of variances. Sometimes a variance in one


area is related to a variance in another. For example, a favourable raw
material price variance resulting from the purchase of a lower grade of
material, may cause an adverse labour efficiency variance because the
lower grade material is harder to work with. These two variances would
need to be considered jointly before making an investigation decision.

4. The inherent variability of the cost or revenue. Some costs, by nature,


are quite volatile (oil prices, for example) and variances would therefore
not be surprising. Other costs, such as labour rates, are far more stable
and even a small variance may indicate a problem.

5. Adverse or favourable? Adverse variances tend to attract most


attention as they indicate problems. However, there is an argument for
the investigation of favourable variances so that a business can learn
from its successes.

6. Trends in variances. One adverse variance may be caused by a


random event. A series of adverse variances usually indicates that a
process is out of control.

7. Controllability/probability of correction. If a cost or revenue is outside


the managers control (such as the world market price of a raw
material) then there is little point in investigating its cause.

8. Costs and benefits of correction. If the cost of correcting the problem is


likely to be higher than the benefit, then there is little point in
investigating further.

Further questions:

Question 1

188
In month 2 the following data applies:

Standard costs for 1 tonne of brown rice

14 tonnes of rice seeds are needed at a cost of $60 per tone


It takes 2 labour hours of work to produce 1 tonne of brown rice and labour
is normally paid $18 per hour. Idle time is expected to be 10% of hours paid;
this is not reflected in the rate of $18 above.
2 hours of variable overhead at a cost of $30 per hour
The standard selling price is $240 per tonne
The standard contribution per tonne is $56 per tone

Budget information for month 2 is

Fixed costs were budgeted at $210,000 for the month


Budgeted production and sales were 8,400 tonnes

The actual results for month 2 were as follows:

Actual production and sales were 8,000 tonnes

12,000 tonnes of rice seeds were bought and used, costing $660,000
15,800 labour hours were paid for, costing $303,360
15,000 labour hours were worked
Variable production overhead cost $480,000
Fixed costs were $200,000
Sales revenue achieved was $1,800,000

Required:

Calculate the variances for month 2 in as much detail as the information


allows and reconcile the budget profit to the actual profit using marginal
costing principles. You are not required to comment on the performance of the
business or its managers for their performance in month 2.

(16 marks)

ACCA F5 June 2008 Qs1 (Part)

189
CHAPTER 15
Advanced Variance Analysis
15.1 ACCA SYLLABUS GUIDE OUTCOME 1:
Calculate, identify the cause of, and explain mix and yield variances
Identify and explain the interrelationship between price, mix and yield

In Chapter 14, we have looked at the total material variance. The total
material variance is divided into the price and usage variances. Moving a step
forward, the usage variance is divided into mix and yield variances.

Total Material Variance

Price Usage

Mix Yield

15.1.1 Material Mix Variance

The mix variance is calculated as the difference between the actual total
quantity used in the standard mix and the actual quantities used in the actual
mix, valued at standard costs. So, it is inputs which are being considered.54

The standard mix shows the proportion of a material that we expect to use in
a given mix. The mix variance identifies the amount by which the actual
proportion differs from the standard mix.

54 December 2014 F5 Qs 5 b(i)

190
A favourable material mix variance would suggest that a higher proportion of a
cheaper material is being used, hence reducing the overall average cost per
unit. An adverse material mix variance indicates that more of the expensive
material was used in the actual input than indicated by the standard mix.

15.1.2 Material Yield Variance

A material yield variance arises when the output which was achieved is
different from the output which would have been expected from the inputs. So,
the yield variance focuses on outputs.55

A favourable material yield variance indicates that more output was produced
from the quantity of material used than expected by the standard. The
increase in yield is likely to be the result of employing more skilled labour, or
introducing more efficient working practices. An adverse material yield
variance suggests that less output has been achieved for a given input, i.e.
the total input in volume is more than expected for the output achieved.

15.1.3 When to calculate the mix and yield variance 56

It is only appropriate to calculate and interpret material mix and yield


variances if quantities in the standard mix can be varied. It has also been
argued that calculating yield variances for each material is not useful, as yield
is related to output overall rather than to particular materials in the input mix. A
further complication is that mix variances for individual materials are inter-
related and so an explanation of the increased use of one material cannot be
separated from an explanation of the decreased use of another.

Lecture Example 1

Carat plc, a premium food manufacturer, is reviewing operations for a three-


month period of 20X3. The company operates a standard marginal costing
system and manufactures one product, ZP, for which the following standard
revenue and cost data per unit of product is available:

Selling price $12.00


Direct material A 2.5 kg at $1.70 per kg
Direct material B 1.5 kg at $1.20 per kg
Direct labour 0.45 hrs at $6.00 per hour

Fixed production overheads for the three-month period were expected to be


$62,500.

55 December 2014 F5 Qs 5 b(i)


56 Examined Sept/Dec 2015 Qs 3

191
Actual data for the three-month period was as follows:
Sales and production 48,000 units of ZP were produced and sold
for $580,800.
Direct material A 121,951 kg were used at a cost of
$200,000.
Direct material B 67,200 kg were used at a cost of $84,000.
Direct labour Employees worked for 18,900 hours, but
19,200 hours were paid at a cost of
$117,120.
Fixed production overheads $64,000

Budgeted sales for the three-month period were 50,000 units of Product ZP.

Required:

a. Calculate the following variances:


(i) sales volume contribution and sales price variances
(ii) materials price, mix and yield variances
(iii)labour rate, labour efficiency and idle time variances. (8 marks)

b. Critically discuss the types of standard used in standard costing


and their effect on employee motivation. (7 marks)

(ACCA Paper 2.4 December 2003 Qs 5)

Note: In the December 2015 examiners report, the examining team wrote that
for Section B Question 1 part b,

reasons why an adverse yield variance might arise, common errors/issues


included were as follows:

1. Failing to read the question properly and giving instead reasons for a
favourable variance.
2. Failure to expand upon why an issue may cause an adverse impact
upon the yield.
3. Many references to expensive materials which relates to price
variances rather than yield variances.

15.2 ACCA SYLLABUS GUIDE OUTCOME 2:


Explain the wider issues involved in changing mix, e.g. cost, quality and
performance management issues

The materials mix variance indicates the cost of a change in the mix of
materials and the yield variance indicates the productivity of the
manufacturing process. A change in the mix can have wider implications.

192
For example, rising raw material prices may cause pressure to change the mix
of materials. Even if the yield is not affected by the change in the mix, the
quality of the final product may change. This can have an adverse effect on
sales if customers do not accept the change in quality. The production
managers performance may be measured by mix and yield variances but
these performance measures may fail to indicate problems with falling quality
and the impact on other areas of the business. Quality targets may also be
needed.

Lecture Example 2

The Finance Director of Borgia Ltd has produced the table below showing the
variance results for the first three months of 2011.

$ $ $
January February March
Material Price variance 3000 (A) 2000 (A) 1000 (A)
Material Mix variance 2000 (A) 750 (A) 100 (F)
Material Yield variance 4000 (A) 2000 (A) 50 (F)
Total Material variance 9000 (A) 4750 (A) 850 (A)

The sales director has commented that sales are significantly increasing.
Customers are very happy with the quality of the products. Both the
production and purchasing managers have joined the company at the
beginning of the year.

Required:

i) Explain the significance of the three variances above (the price,


mix and yield variances). Assess the extent to which each
variance is controllable by the purchasing manager and the
production manager.

ii) Compare the performance of the purchasing manager and the


production manager taking into account the cost variance results
and the comments of the sales director.

15.3 ACCA SYLLABUS GUIDE OUTCOME 3:


Suggest and justify alternative methods of controlling production
processes

In a modern manufacturing environment with an emphasis on quality


management, mix and yield variances for control purposes may not be
possible or may be inadequate. Other performance measures and targets
could be more useful.

193
Rates of wastage
Average cost of input calculations
Percentage of deliveries on time
Customer satisfaction ratings
Yield percentage calculations or output to input conversion rates

15.4 ACCA SYLLABUS GUIDE OUTCOME 4:


Calculate, identify the cause of, and explain sales mix and quantity
variances.
Identify and explain the relationship of the sales volume variances with
the sales mix and quantity variances.

Where a company sells several different products that have different profit
margins, the sales volume variance can be divided into a sales quantity
(sometimes called a sales yield variance) and sales mix variance.

The quantity variance measures the effect on profit of selling a different total
quantity from the budgeted total quantity57.

The mix variance measures the effect on profit of changing the mix of actual
sales from the standard mix58.

The variances can be measured either in terms of contribution margins or


profit margins.

Lecture Example 3

The budgeted sales for the Milano Company for a period were:

Unit Total
Units Contribution Margin Contribution
($) ($)
Product X 8,000 (40%) 20 160,000
Y 7,000 (35%) 12 84,000
Z 5,000 (25%) 9 45,000
20,000 289,000

and the actual sales were:

Unit Total
Units Contribution Margin Contribution
($) ($)
Product X 6,000 20 120,000

57 Examined June 2014 Qs 5


58 Examined June 2014 Qs 5

194
Y 7,000 12 84,000
Z 9,000 9 81,000
22,000 285,000

Assume that actual selling prices and unit costs are identical to standard
costs/prices.
Required:

Calculate the sales volume contribution, sales mix and sales quantity
variances.

Lecture Example 4

BRK Co operates an absorption costing system and sells three products, B, R


and K which are substitutes for each other. The following standard selling
price and cost data relate to these three products:

Product Selling price Direct material Direct labour


per unit per unit per unit

B $14.00 3.00 kg at $1.80 per kg 0.5 hrs at $6.50 per hr


R $15.00 1.25 kg at $3.28 per kg 0.8 hrs at $6.50 per hr
K $18.00 1.94 kg at $2.50 per kg 0.7 hrs at $6.50 per hr

Budgeted fixed production overhead for the last period was 81,000. This was
absorbed on a machine hour basis. The standard machine hours for each
product and the budgeted levels of production and sales for each product for
the last period are as follows:

Product B R K
Standard machine hours per unit 0.3 hrs 0.6 hrs 0.8 hrs

Budgeted production and sales (units) 10,000 13,000 9,000

Actual volumes and selling prices for the three products in the last period were
as follows:

Product B R K
Actual selling price per unit $14.50 $15.50 $19.00

Actual production and sales (units) 9,500 13,500 8,500

195
Required:

a) Calculate the following variances for overall sales for the last
period:
(i) sales price variance;
(ii) sales volume profit variance;
(iii) sales mix profit variance;
(iv) sales quantity profit variance

and reconcile budgeted profit for the period to actual sales less
standard cost.

b) Discuss the significance of the sales mix profit variance and


comment on whether useful information would be obtained by
calculating mix variances for each of these three products.

(ACCA Paper 2.4 June 2005 Qs 3)

15.5 ACCA SYLLABUS GUIDE OUTCOME 5:


Calculate planning and operational variances for sales, including
market size and market share, materials and labour.

A planning and operational approach to variance analysis divides the total


variance into those variances which have arisen because of inaccurate
planning or faulty standards (planning variances) and those variances which
have been caused by adverse or favourable operational performance,
compared with a standard which has been revised in hindsight (operational
variances).

A planning variance (or revision variance) compares an original standard


with a revised standard that should or would have been used if planners had
known in advance what was going to happen. A planning variance is deemed
not controllable by management, i.e. management may not be held
responsible.

An operational variance (or operating variance) compares an actual result


with the revised standard. It is deemed controllable by management. Hence,
management is held responsible for operational variances.

196
Original (flexed)
budget Planning
Variances

Revised (flexed)
budget
Operating
Variances
Actual
results

15.5.1 Planning and operational variances for sales

For sales, two variances have to be calculated 59: -

Planning market volume/size variance

Operational market share variance

Lecture Example 5

The following information is available for product Blair: -

Budget

Selling Price: $14/unit


Sales Volume: 200,000 units
Standard Profit: $3/unit

Actual

Selling Price: $12.50/unit


Sales Volume: 180,000 units
Actual Profit: $1.50/unit

In 2011, the market demand for the product grew by 10% from 2,000,000
units to 2,200,000 units.

59 Examined in June 2012 Qs 4

197
Required

a) Calculate the sales price and volume variances.


b) Analyse the volume variances into market share and market size.

15.5.2 Planning and operational variances for materials and labour

For materials and labour, planning and operational variances can be


calculated by comparing original and revised budgets (planning) and revised
budgets with actual results (operational).

A material price planning variance is really useful to provide feedback on just


how skilled managers are in estimating future prices.

The operational variance is more meaningful as it measures the purchasing


departments efficiency given the market conditions that prevailed at that time.
It ignores factors which cannot be controlled by purchasing department. 60

Lecture Example 6

The standard cost per kg of raw material was estimated to be $10. The
general market price at the time of purchase was $10.50 per kg and the actual
price paid for the raw material was $10.40 per kg.

20,000 kgs of the raw materials were purchased during the period.

Required: -

a) Calculate the material price variance.


b) Also calculate the planning and operational materials price variances.

Lecture Example 7 (taken from the article Measuring planning variances by


Geoff Cordwell (Student Accountant April 2009)
(http://www.accaglobal.com/content/dam/acca/global/pdf/sa_may09_cordwell.pdf)

Lowland Skiing had planned, when it originally produced its budget, to buy its
artificial snow for $5/per kg. However, due to subsequent improvements in
technology, manufacturers around the world reduced their prices to $4.85 per
kg. This latter figure is now considered to be a fair target price for the purpose
of performance assessment for the budget period.

The actual price paid was $4.75, as the Lowland Skiing buying department
negotiated strongly for a better price.

60 Examined in Dec 2010 Qs 1

198
Further information: -

Original Budget Revised Budget Actual

Material used 10kg per unit 9.5kg per unit 108,900 kg


Budgeted production 10,000 units
Actual production 11,000 units

Required

Calculate material price and usage planning and operational variances.

Lecture Example 8

Direct labour rate variance $ 5,000 Adverse


Direct labour efficiency variance $14,040 Favourable
Standard hours to produce 1 unit 8 hours
Standard rate $18 / hour
Actual units produced 660 units
Actual hours worked 4,500 hours
Actual labour cost $86,000

The production manager has informed us that the standard time of 8 hours
per unit was the time taken to produce the first unit only. A 90% learning rate
applies for the first 800 units.

Required:

Calculate the direct labour efficiency planning and operating variances.

15.5.3 Advantages of a system of planning and operational variances

1. The analysis highlights those variances which are controllable and


those which are non-controllable.
2. Managers' acceptance of the use of variances for performance
measurement, and their motivation, is likely to increase if they know
they will not be held responsible for poor planning and faulty standard
setting.
3. The planning and standard-setting processes should improve;
standards should be more accurate, relevant and appropriate.
4. Operational variances will provide a 'fairer' reflection of actual
performance.

199
15.5.4 The limitations of planning and operational variances, which must
be overcome if they are to be applied in practice.

1. What should a realistic/achievable standard be? This may be difficult to


decide.
2. It may become too easy to justify all the variances as being due to bad
planning, so no operational variances will be highlighted.
3. Revising and analysing variances into planning and operating will take
time and can be costly.
4. Do managers use correctly this meaningful information? How can it
improve their performance? Does it lead to better decision-making?

15.6 ACCA SYLLABUS GUIDE OUTCOME 6:


Identify and explain those factors that could and could not be allowed
to revise an original budget.
Explain any manipulation issues in revising budgets.

A budget forms the basis of many performance management systems. Once


set, it can be compared to the actual results of an organisation to assess
performance. A change to the budget can be allowed in some circumstances
but these must be carefully controlled if abuse is to be prevented.

Allow budget revisions when something has happened that is beyond the
control of the organisation (for e.g. a supplier has gone into liquidation; a rapid
increase in world market prices of a particular material) which renders the
original budget inappropriate for use as a performance management tool.
These adjustments should be approved by senior management who should
attempt to take an objective and independent view.

Disallow budget revisions for operational issues. Any item that is within the
operational control of an organisation should not be adjusted. This type of
decision is often complicated and each case should be viewed on its merits.
The direction of any variance (adverse or favourable) is not relevant in this
decision.

15.7 ACCA SYLLABUS GUIDE OUTCOME 7:


Analyse and evaluate past performance using the results of past
performance
Use variance analysis to assess how future performance of an
organization or business can be improved

Variance analysis is used for control purposes: - to compare actual


performance with standard cost. Hence, it is used to analyse and evaluate
past performance. The managers responsible for variances should be
identified. They are expected to account for the variance and indicate the
corrective actions they will be taking.

200
As already discussed, operational managers should only be held responsible
for operational variances. Unless these managers are also involved in the
budgeting process, they cannot be held responsible for planning variances.

Hence, a sales price variance is usually the responsibility of a sales or


marketing manager, a material usage variance is normally the responsibility of
a production manager whereas a material price variance is usually the
responsibility of the purchasing manager.

Variance analysis must also provide guidance how the future performance of
an organization can be improved. Control measures affect the future. They
cost money and should only be taken if the benefits arising from improved
performance outweigh the costs.

What action should be taken if we have

1. an adverse material price variance: - search for a lower price without


affecting quality; buy in bulk to obtain quantity discounts

2. an adverse sales volume variance: - reduce selling price to increase


demand for the product. Although this will increase quantity demanded, it will
lead to an adverse sales price variance.

15.8 ACCA SYLLABUS GUIDE OUTCOME 8:


Describe the dysfunctional nature of some variances in the modern
environment of JIT and TQM

The just-in-time (JIT) approach to conducting the activities of an organisation


is often incorrectly considered to relate solely to the manufacturing
environment. In fact, just-in-time is a very broad philosophy that emphasises
simplification and continuously reducing waste in each and every area of
business activity.

Total quality management (TQM) is an approach that seeks to improve quality


and performance which will meet or exceed customer expectations. It applies
a zero defect philosophy and develops a culture of continuous improvement
which focuses on meeting customers expectations.

Where the management of an organisation is considering the implementation


of a just-in-time philosophy they need to give detailed consideration to the
following:

1. Employee involvement should be actively encouraged. The successful


operation of just-in-time requires that workers possess a flexibility of
both attitude and aptitude.
2. The fundamental requirement to ensure that the level of quality
satisfies the customer.

201
3. A constant focus on the simplification of products and processes in
order to maximise the utilisation of available resources.
4. The creation of a uniform factory load which will enable the speed of
manufacture to mirror the demand of customers.
5. The minimisation of set-up times as no value is added at this point in
the manufacturing process.
6. The factory layout to be adopted. The majority of factories operating
just-in-time manufacturing operations have adopted a U-shaped layout
of machinery. This layout facilitates the flow of components, thereby
minimising transportation activities while maximising efficiency.
7. The operation of a 'pull' system which produces products for the time
when they are required by customers.
8. The fundamental need for excellent relationships with suppliers, putting
emphasis on flexibility and good communication channels.

Therefore, the key features of companies operating in a JIT and TQM


environment can be summarized as follows: -

1. high level of automation


2. high levels of overheads and low levels of direct labour costs
3. customized products produced in small batches
4. low stocks
5. emphasis on quality

15.9 ACCA SYLLABUS GUIDE OUTCOME 9:


Describe the behavioural problems resulting from using standard costs
in rapidly changing environments

Variance analysis may not be appropriate in modern manufacturing


companies because:

1. Standard product costs apply to manufacturing environments in which


quantities of an identical product are output from the production
process. They are not suitable for manufacturing environments where
products are nonstandard or are customised to customer
specifications.

2. It is doubtful whether standard costing is of much value for


performance setting and control in automated manufacturing
environments. In practice, where manufacturing systems are highly
automated, the rates of production output and materials consumption,
are controlled by the machinery rather than the workforce.

3. Variances are the difference between actual performance and


standard, measured in cost terms. The significance of variances for
management control purposes depends on the type of standard cost
used. For example, adverse variances with an ideal standard have a

202
different meaning from adverse variances calculated with a current
standard.

4. Standard costing and adherence to a preset standard is inconsistent


with the concept of continuous improvement, which is applied within
TQM and JIT environments.

5. Variance analysis is often carried out on an aggregate basis (total


material usage variance, total labour efficiency variance and so on) but
in a complex and constantly changing business environment more
detailed information is required for effective management control.

6. Shorter product life cycles in the modern business environment mean


that standard costs will need to be reviewed and updated frequently.

15.10 ACCA SYLLABUS GUIDE OUTCOME 10:


Discuss the effect that variances have on staff motivation and action

Staff's acceptance of the use of variances for performance measurement, and


their motivation, is likely to increase if they know they will not be held
responsible for poor planning and faulty standard setting.

The theory of motivation suggests that having a clearly defined target results
in better performance than having no target at all, that targets need to be
accepted by the staff involved, and that more demanding targets increase
motivation provided they remain accepted.

15.10.1 Standards and Motivation

A basic standard is one that remains unchanged for several years and is used
to show trends over time. Basic standards may become increasingly easy to
achieve as time passes and hence, being undemanding, may have a negative
impact on motivation. Standards that are easy to achieve will give employees
little to aim at.

Ideal standards represent the outcome that can be achieved under perfect
operating conditions, with no wastage, inefficiency or machine breakdowns.
Since perfect operating conditions are unlikely to occur for any significant
period, ideal standards will be very demanding and are unlikely to be
accepted as targets by the staff involved as they are unlikely to be achieved.
Using ideal standards as targets is therefore likely to have a negative effect on
employee motivation.

Current standards are based on current operating conditions and incorporate


current levels of wastage, inefficiency and machine breakdown. If used as

203
targets, current standards will not improve performance beyond its current
level and their impact on motivation will be a neutral one.

Attainable standards are those that can be achieved if operating conditions


conform to the best that can be practically achieved in terms of material use,
efficiency and machine performance. Attainable standards are likely to be
more demanding than current standards and so will have a positive effect on
employee motivation, provided that employees accept them as achievable.

15.10.2 Participation in setting standards

As discussed in Chapter 11, participation in standard setting (bottom-up


budgets) can be a way of achieving improvements in performance.

However, the effectiveness of participation in setting standards depends on a


variety of factors, such as the type of staff involved, the attitudes of their
managers, the organisation structure and culture, and the nature of the work.
(Refer back to Chapter 11 for the pros and cons of participation in setting
budgets).

15.10.3 Pay as a motivator

Motivation can be provided in the form of higher pay or other rewards if


targets are reached or exceeded. This will only happen if employees perceive
the standards set as attainable. Also, as mentioned before in Chapter 7,
where rewards are linked to targets, there may be slack when setting standard
costs.

Further Question (extracted from Article Materials Mix and Yield Variances,
A. Irons, F5 examiner, Student Accountant, April 2010)61

A chemical C uses both chemicals A and B to make it. Chemical A has a


standard cost of $20 per litre and chemical B has a standard cost of $25 per
litre. Research has shown that various combinations of chemicals A and B
can be used to make C, which has a standard selling price of $30 per litre.
The best two of these combinations have been established as: -

Mix 1: 10 litres of A and 10 litres of B will yield 18 litres of C


Mix 2: 8 litres of A and 12 litres of B will yield 19 litres of C

Assuming that the quality of C produced is exactly the same in both instances,
find the optimum mix of materials A and B.

61 http://www.accaglobal.com/content/dam/acca/global/pdf/Feb10_matvariances_F5.pdf

204
Answer

To find the optimum mix of materials A and B, calculate the contribution


achieved by Mix 1 and Mix 2

Mix 1:
SR from Product C (18 x $30) 540
Cost Chemical A (10 x $20) 200
Cost Chemical B (10 x $25) 250
450
Contribution 90

Mix 2:
SR from Product C (19 x $30) 570
Cost Chemical A (8 x $20) 160
Cost Chemical B (12 x $25) 300
460
Contribution 110
Therefore, the optimum mix that minimises the cost of the inputs compared to
the value of the outputs is mix 2.

Find the standard cost per litre of C for Mix 1 and Mix 2

Mix 1 = total cost of mix 1 divided by the litres of C


= $450 / 18 = $25/litre of C

Mix 2 = total cost of mix 2 divided by the litres of C


= $460 / 19 = $24.21/litre of C

To confirm: total contribution of Mix 1 = (30-25) x 18 = $90


total contribution of Mix 2 = (30-24.21) x 19 = $110

To Calculate Material Mix Variance

The materials mix and yield variances are simply a detailed breakdown of the
materials usage variance.

The material mix variance refers to the quantity of each material that is used
to make our product. The material mix variance focuses on inputs.

205
Previous example contd.

Standard mix 8 litres of A and 12 litres of B will yield 19 litres of C (Mix 2)


Actual information 1,850 litres of C produced using a total of 900 litres of
Chemical A and 1,100 litres of B (2,000 litres in total). The actual costs of A
and B were at the standard costs of $20 and $25/ litre respectively.

Calculate the material mix variance

Chemical Std Should Did Mix Difference Std Variance


Mix Mix cost

A 8 8/20 x 900 100(A) 20 2000(A)


2000 =
800
B 12 12/20 x 1100 100(F) 25 2500(F)
2000 =
1200
20 2000 2000 500(F)

To Calculate Material Yield Variance

Where there is a difference between the actual level of output for a given
set of inputs and the standard output for a given set of inputs, a
materials yield variance arises.

Chemical Std Std Qty at Actual Difference Std Variance


Mix Std Mix Qty at cost
Std Mix

A 8 1947.37/20 800 21.05 (A) 20 421(A)


x8
=778.95
B 12 1947.37/20 1200 31.58 (A) 25 789(A)
x 12 =
1168.42
20 1947.37 2000 1210(A)

19 litres of output require 20 litres of input (std)


1850 litres of actual output require how many litres of input?

20/19 x 1850 = 1947.37

OR

206
2000ltrs input should yield (see working below) 1900 ltrs
did yield 1850 ltrs
50 ltrs (A) of output x std cost
per ltr of output C
= 50 ltrs x 24.21 = 1210 (A)

20 ltrs of input yield 19 ltrs output (standard loss of 1/20 = 5%)


2000 ltrs of input should yield how much?
19 / 20 x 2000 = 1900ltrs

Materials mix, yield and usage variances

As can be deduced from the above example, there is a direct relationship


between materials mix and materials yield variances. By using a cheaper mix
of materials ($500(F)), we had a significantly lower yield of Product C of
$1210(A). This yield variance is over double the amount that we saved by
using a cheaper mix of materials.

By netting the two variances off against each other, we have an adverse
material usage variance of $710. Lets confirm: -

Chemical A: -
1850ltrs of Product C should use 778.95
did use 900
121.05(A) x $20 = $2421 (A)

19 litres of C use 8 litres of A


1850 ltrs use ?

8/19 x 1850 = 778.95

Chemical B: -
1850ltrs of Product C should use 1168.42
did use 1100
68.42(F) x $25 = $1711 (F)

19 litres of C use 12 litres of B


1850 ltrs use ?

12/19 x 1850 = 1168.42


Total usage variance = 2421 (A) + 1711 (F) = 710(A)

Materials Mix and Yield Variances: The Effect on Quality

207
It is very important in a business to produce products of good quality.
Although we may change the product mix to make savings, the quality of the
product may be adversely affected and this can damage the reputation of the
business. Although poor quality input materials may in some cases yield
volumes similar to those achieved with higher quality materials, the yield may
not be of same quality.

The use of poor quality material will tend to affect sales volume in the long
term. It will also affect labour efficiency in the shorter term (usually same
period) as it may be more difficult to work with. Hence this will result in higher
overhead costs and lower profits.

Very important in the exam, discuss the interdependencies between


variances (e.g. favourable materials mix variance leads to decreased yield,
poorer quality, higher labour costs, lower sales volume and lower profits).

Further questions

Question 162

To produce 19 litres of product X, a standard input mix of 8 litres of chemical


A and 12 litres of chemical B is required.

Chemical A has a standard cost of $20 per litre and chemical B has a
standard cost of $25 per litre.

During September, the actual results showed that 1,850 litres of product X
were produced, using a total input of 900 litres of chemical A and 1,100 litres
of chemical B (2,000 litres in total).

The actual costs of chemicals A and B were at the standard cost of $20 and
$25 per litre respectively.

It was expected that an actual input of 2,000 litres would yield an output of
1,900 litres (95%). The actual yield for September was only 1,850 litres, which
was 50 litres less than expected.

For the total materials mix variance and total materials yield variance, was
there a favourable or adverse result in September?

A. The total mix variance was adverse and the total yield variance was
favourable

62 Specimen Exam Applicable from December 2014

208
B. The total mix variance was favourable and the total yield variance
was adverse
C. Both variances were adverse
D. Both variances were favourable

Question 263

The market for leather bound diaries has been shrinking as the electronic
versions become more widely available and easier to use. Spike Co has
produced the following data relating to leather bound diary sales for the year
to date:

Budget
Sales volume 180,000 units
Sales price $1700 per unit
Standard contribution $700 per unit

The total market for diaries in this period was estimated in the budget to be
18m units. In fact, the actual total market shrank to 16m units for the period
under review.

Actual results for the same period


Sales volume 176,000 units
Sales price $1640 per unit

Required:

Analyse the sales volume variance into: -

Market size _______________

Market share ______________

CHAPTER 16
63 F5, December 2007, Question 3 part

209
Performance Management
Information Systems
Performance management information systems are an integral part in
producing the information required by management accountants to enable
performance measurement.

First, lets have a look at the difference between data and information.

Data consits of the raw facts and figures, e.g. numbers, letters and
transactions, that have not yet been processed into a form suitable to make
decisions.

Information is summarised data, or otherewise manipulated data, that is


useful for decision-making. The data has been processed and it can be used
to improve the quality of decisions taken. Hence, information is meaningful
data.

A data processing system records the day-to-day transactions taking place


within an organisation. It records, analyses, sorts, summarises, calculates and
stores data.

An information system uses this data and turns it into useful information
required to run the organisation.

Extracted from: http://tie532.wikispaces.com/Data+processing

16.1 ACCA SYLLABUS GUIDE OUTCOME 1:

210
Identify the accounting information requirements and describe the
different types of information systems used for strategic planning,
management control and operational control and decision-making.

16.1.1 Why do managers need information?

Managers need information to formulate plans and take decisions. If actual


results diverge from plans, control action is required.

16.1.2 Levels of Planning and Control within an Organisation

There are three levels of planning and control within an organisation:


strategic, tactical and operational levels.

Level of Control Activity

Strategic Planning Plans the long-term strategies of the


organisation e.g. investment decisions
Management Control Medium term planning and control decisions
(Tactical Planning) Responsible with implementing decisions of
strategic managers
Ensures that the organisations objectives are
achieved by using resources efficiently and
effectively
Operational Control Concerned with controlling the day-to-day
operations of the organisation
Short term decisions

16.1.3 Management accounting information for: -

1. Strategic Planning, Control and Decision-Making

For strategic planning, management accounting systems require information


from both internal and external sources. They require information from many
areas of the business and help to ensure goal congruence.

Due to the long-term nature of such planning and the element of uncertainty in
the long term, management accounting information should include risk and
uncertainty analysis. Discounted cash flow techniques are expected to be
used in project evaluation.

Strategic Management Accounting

211
Strategic Management Accounting has been defined as "a form of
management accounting in which emphasis is placed on information which
relates to factors external to the firm, as well as non-financial information and
internally generated information." 64

Whereas traditional management accounting fails to assess


competition(inward-looking) and ignores the impact of other activities,
strategic management accounting is the merging of strategic business
objectives with management accounting information to provide a forward
looking model that assists management in making business decisions.

Unlike traditional management accounting -- which has an internal focus


strategic management accounting evaluates external information regarding
trends in costs, prices, market share and cash flow, and their impacts on
resources, to determine the appropriate tactical response. The strategic
element of management accounting requires enhanced information about
competitors, suppliers and technologies.

Illustration of Strategic Management Accounting: -

According to research from McGraw-Hill, Tesco determined that the


company's primary fixed asset base was its stores. Based on this factor
management created strategic partnerships with construction companies to
lower costs and maintain quality. In addition, Tesco monitored competitor
product pricing to reduce customer prices and gain market share. Tesco also
enhanced its technology by offering store cards that track customer purchase
patterns.65

2. Management Control

Whereas strategic planning involves setting objectives and targets,


management control is concerned with implementing decisions of strategic
managers and ensuring that the organisations objectives are achieved by
using resources efficiently and effectively.

Tactical level managers require information in much more summarised form.


Much of this information will be given to the tactical manager by the
operational managers in the form of reports, hence they are primarily
generated internally e.g. productivity measurements, manning levels. This
information is usually prepared on a regular basis such as on a weekly or
monthly basis. Tactical information is very often quantitative and expressed in
monetary terms, e.g. variance analysis reports, cash flow forecasts, profit
results for a particular department.

64 K. Simmonds in M.L. Inman, Strategic Management Accounting, Student Accountant, Nov. 1999,
http://www2.accaglobal.com/archive/sa_oldarticles/43981
65 Extracted from http://smallbusiness.chron.com/examples -strategic-management-accounting-

18149.html

212
Tactical planning and management control decisions or strategic
planning?

1. Launch a new product strategic plan


2. Preparing the functional budgets for next year management control
planning
3. Reducing employee turnover and operational costs in the next 12
months management control decision
4. The board of directors take a decision to increase market share by 10%
for, at least, 5 years is a strategic plan. How to achieve that increase falls
under management control decisions (tactical planning) e.g. the senior
sales manager is planning advertising and marketing campaigns to
increase the market share by 10% next year.

Management control is usually carried out regularly by comparing the planning


activities with the outcomes, e.g. budget targets are compared with actual and
action taken to remove any resulting variances. If circumstances have
changed, the budget will be altered to obtain a better picture of the company.

3. Operational Control

The operational level of management is primarily concerned with making sure


that the day-to-day tasks are carried out efficiently and effectively. E.g. the
sales manager of the Buskett branch has set weekly sales targets for each of
his sales representatives. This is an example of an operational control
decision.

Most of the information required in operational control decisions is obtained


from internal sources and this must be detailed and precise. E.g. information
required to determine how many operative staff will be required to complete a
task. In fact, operational information is expected to be more detailed than
tactical information, which in turn, is expected to be more detailed than
strategic information. Unlike tactical information, operational information is
often expressed in units, hours, kgs, etc.

The following figure shows the information characteristics at the different


management levels for planning, control and decision making: -

213
http://www.thebooktailor.com/URL%20Data/Information%20Systems%20-
%20Types%20of%20Information%20Systems.html

Lecture Example 1

The following are examples of business information:


(1) Annual forecasts of revenues and costs for a department
(2) Product development plans for the next 23 years
(3) Targets agreed by key managers at their performance appraisal interviews

Which of the above would be classified as tactical information?


A. 1, 2 and 3
B. 1 and 3 only
C. 2 and 3 only

Lecture Example 2

Which of the following statements are correct?


(i) Strategic information is mainly used by senior management in an
organisation.
(ii) Productivity measurements are examples of tactical information.
(iii) Operational information is required frequently by its main users.

A. (i) and (ii) only


B. (i) and (iii) only
C. (ii) and (iii) only
D. (i), (ii) and (iii)

214
Lecture Example 3

The following statements refer to strategic planning:


(i) It is concerned with quantifiable and qualitative matters.
(ii) It is mainly undertaken by middle management in an organisation.
(iii) It is concerned predominantly with the long term.

Which of the statements are correct?


A. (i) and (ii) only
B. (i) and (iii) only
C. (ii) and (iii) only
D. (i), (ii) and (iii)

Lecture Example 4

Which of the following would be best described as a short-term tactical plan?


A. Reviewing cost variances and investigate as appropriate
B. Comparing actual market share to budget
C. Lowering the selling price by 15%
D. Monitoring actual sales to budget

16.2 ACCA SYLLABUS GUIDE OUTCOME 2:


Define and identify the main characteristics of transaction processing
systems; management information systems; executive information
systems; and enterprise resource planning systems.

Extracted from: http://www.samueljtanner.com/information-systems/sample-


page/systems/

215
16.2.1 Transaction processing systems (TPS)
A transaction processing system is a computerized system that performs and
records the daily, routine transactions necessary to conduct the business. It
collect, stores, modifies and retrieves the transactions of an organization.

TPS are built to handle regular, high volume, data processing needs, e.g.
invoicing, order processing. They tend to be inflexible (every transaction
processed in the same way) and they are often developed with strong
emphasis on efficiency and reliability to save time and money. TPS are mainly
used by the operational managers to make day-to-day decisions.

Two Types of Transaction Processing Systems

1. Batch transaction processing collects transaction data as a group,


with the actual updating of the database later when it is scheduled or
there is enough data.

E.g. the way that credit card companies process billing. The customer
does not receive a bill for each separate credit card purchase but one
monthly bill for all of that months purchases. The bill is created
through batch processing, where all of the data is collected and held
until the bill is processed as a batch at the end of the billing cycle.

Another example is the clearing of presented cheques.

2. Real time transaction processing is the immediate processing of


data. A number of users can perform transactions at the same time.
Access to a central online database is required. Examples include
reservation systems for flights, point of sale terminal and library loans.

16.2.2 Management Information Systems (MIS)

A management information system provides managers with information on the


basis of which they can make better, more informed decisions and exercise
oversight and control of the parts of the organisations they are responsible for.
It collates information from the TPS and provides periodic reports to middle
managers to control the business.

For example:

1. Data on goods sold is collected by the data processing system, using a


barcode scanner and an EPOS system, and stored on a computer file;
2. An operational information system then reads this data and produces a
list of items that need reordering;

216
3. A management information system may analyse the sales data to
highlight sales trends and use this information to plan a new marketing
campaign, adjust price levels or plan an increase or reduction in
production facilities.

16.2.3 Decision support systems (DSS)

Decision support systems offer enhanced ability to manipulate the data, model
it, and allow a user (decision maker) to explore alternative scenarios. This
Information system helps the decision maker to make his own informed
decisions and exercise his own judgement.

Examples of DSS in accounting include:

Cost accounting system


Capital budgeting system
Budget variance analysis system
General decision support system

16.2.4 Executive information systems (EIS)

An executive information system provides information to senior managers and


executives who need a broad mix of internal and external information to
support strategic decisions. It draws data from the MIS and also includes data
from external sources, for e.g. competition.

EIS are designed to help senior managers get information quickly and
effectively. They include data analysis (e.g. interactive graphical displays)
and modeling tools, e.g. what-if-analysis.

16.2.5 Enterprise resource planning system (ERP system)

Enterprise resource planning systems integrate internal and


external management information across an entire organization, e.g.
finance/accounting, manufacturing, sales and service, purchasing and human
resources are integrated into one single system using an
integrated software application. The purpose of ERP system is to facilitate the
flow of information between all business functions inside the boundaries of the
organization and manage the connections to outside stakeholders.

Where an ERP system is implemented, this has had a significant impact on


the work of management accountants.
1. Less time spent on data collection
2. Less time spent on data analysis and routine report generation

217
3. More involvement in business decision-making
4. More focus on internal reporting, e.g. performance measures and
control issues
5. More focus on the external environment, e.g. benchmarking
6. Focus on forward looking rather than historical analysis
7. Need for improved communication skills66

16.3 ACCA SYLLABUS GUIDE OUTCOME 3:


Define and discuss the merits of, and potential problems with, open
and closed systems with regard to the needs of performance
management.

An open system is connected to and interacts with its external environment.


Healthy open systems continuously exchange feedback with their
environments, analyze that feedback, adjust the systems as needed to
achieve goals, and then transmit information back out to the environment.
A good example of an open system organizational structure is the rise of the
virtual corporation where companies, their suppliers as well as customers use
information networks to interact and collaborate as one large virtual
corporation. This is an open system because the suppliers and customers
serve as the environment, feeding input back to companies. Companies then
process the input and produce better products for customers and better
interactions with suppliers.

Example: a sports manufacturer regularly seeks feedback from sports athletes


before designing its products.

A closed system doesnt interact with its environment. It is separated from its
environment by the systems boundary and is usually referred to as an
isolated system. There is no exchange of information, material or manpower
between the system and its environment. Such systems are rare as
interaction with the environment is necessary for the business to survive.

An assembly line can be treated as a closed system if it does not interact for
the supply of raw materials. Workers on an assembly line are generally only
responsible for completing their tasks on the line. The accounting system and
the research department can also be examples of a closed system.

66 Management Accountants: A Profession Dramatically Changed by ERP Systems, CIMA,


http://www.cimaglobal.com/Documents/Thought_leadership_docs/cid_ressum_management_accou
ntants_and_erp_dec08.pdf

218
Extracted from:
http://www.emeraldinsight.com/journals.htm?articleid=841622&show=html

In practice, systems tend to be open to some influences and closed to others.


But what are the advantages of an open system?

As already discussed, an open system: -


Encourages communication with its environment which leads to
better interaction
Highlights the inter-dependencies of the different processes and
operations
Takes into consideration modern cultural changes and needs,
allowing the company to produce products and services that meet
those needs
Help business managers to focus on external factors to predict and
respond to potential problems

219
Further questions

Question 167

The following statements have been made about management information


systems:

(1) They are designed to report on existing operations


(2) They have an external focus

Which of the above statements is/are true?

A. 1 only
B. 2 only
C. Neither 1 nor 2
D. Both 1 and 2

Question 268

The following are all types of costs associated with management information.

(i) Use of bar coding and scanners


(ii) Payroll departments processing of personnel costs
(iii) Completion of timesheets by employees

Which of the above are examples of direct data capture costs?

A. (i) only
B. (i) and (ii) only
C. (i) and (iii) only
D. All of the above

Question 369

The following are some of the areas which require control within a division:

(i) Generation of revenues


(ii) Investment in non-current assets
(iii) Investment in working capital
(iv) Apportioned head office costs

67 Specimen Exam Applicable from December 2014


68 Specimen Exam Applicable from December 2014
69 Specimen Exam Applicable from December 2014

220
Which of the above does the manager have control over in an investment
centre?

A. (i), (ii) and (iii) only


B. (ii), (iii) and (iv) only
C. (i), (ii) and (iv) only
D. All of the above

221
CHAPTER 17
Sources of Management
Information
17.1 ACCA SYLLABUS GUIDE OUTCOME 1:
Identify the principal internal and external sources of management
accounting information. Demonstrate how these principal sources of
management information might be used for control purposes. Discuss
the limitations of using externally generated information.

Management information comes from multiple sources. The challenge for a


business is to capture and use relevant and reliable information and the
benefit of such information must exceed the cost of obtaining this information.
Modern IT systems have reduced these costs significantly but skilled highly-
paid staff is required to run these information systems.

17.1.1 Internal Sources

Accounting records are a prime source of internal information. They detail the
transactions of the business in the past, which may be used as the basis for
planning for the future (e.g. preparing a financial budget or forecast). Daily
books such as sales day book, purchase day book and cashbook can provide
useful information to management.

The accounting records are primarily used to record what happens to the
financial resources of a business. For example, how cash is obtained and
spent; what assets are acquired; what profits or losses are made on the
activities of the business.

However, accounting records can provide much more than financial


information. For e.g., details of the products manufactured and delivered from
a factory can provide useful information about whether quality standards are
being met. Data analysed from customer sales invoices provides a profile of
what and to whom products are being sold.

Most internal information is connected to accounting systems - but is not


directly part of them. For e.g.:

a) From the payroll system, records of the people employed by the


business (personal details; what they get paid; skills and experience;
training records)

222
b) Data on the costs associated with business processes (e.g. costings for
contracts entered into by the business)

c) Data from the production department (e.g. number of machines;


capacity; repair record)

d) Data from activities in direct contact with the customer (e.g. analysis of
calls received and missed in a call centre)

Internal information is also provided informally. For example, regular meetings


of staff and management will result in the communication of relevant
information. For e.g. minutes of board meetings may include future strategies
on new products and markets.

17.1.2 External Sources

This is information that is obtained from outside the business. Such external
information tends to be more relevant to strategic and tactical decisions rather
than to operational decisions.

There are many sources of external information: -

a) Government publications such as monetary and fiscal policies,


industry statistics, inflation rates.
b) Press releases such as newspapers, technical magazines, journals
which provide information about share price, technological
developments and information on competitors and their products.
c) Banks can provide information on potential customers and on nation
markets.
d) Financial statements of other businesses provide useful information
to the company.
e) Correspondence received from suppliers, customers and tax
authorities etc.
f) Internet websites, social networking sites, forums etc.
g) Databases held by public bodies and businesses e.g. providing
online information on money market interest rates and foreign
exchange rates.
h) Data warehouses which contain data from both internal and
external sources. They store current as well as historical data and
are used for creating trending reports for senior management
reporting such as annual and quarterly comparisons.

There are many advantages of using the internet to obtain information: - it


offers loads of information which you can acquire efficiently and quickly. It is
relatively inexpensive to acquire information from the internet. But the quality

223
of this information cannot be guaranteed. This will be discussed in more detail
later on.

17.1.3 Information for Control Purposes

Both internal and external sources of information are required by management


for control purposes.

To control (comparing actual results against the plan), internal information is


required. Variance reports are very useful is arriving at control action. For e.g.,
to identify sales variances, information can be obtained internally about the
sales volumes over different time periods.

For example, information about stock (inventory) levels within the organization
helps management identify ways how to increase the sales of slow moving
lines of stock. A reduction in the sales of a product will help management
identify the reasons why this has occurred: has competition introduced a
similar product? Has competition reduced the prices of rival products? Were
there any quality issues regarding our product?

External information is useful for benchmarking (e.g. competitive


benchmarking - comparisons with competitors in the business sector).

17.1.4 Limitations of Externally Generated Information

External information does provide benefits to companies to improve their


decisions by responding appropriately to the environment surrounding them.
However, it does have a number of limitations.

i. Published information, especially from the internet, may not be


accurate and reliable, it may be also out of date.
ii. Published information may be too general and may not meet the exact
needs of the organisation.
iii. It may be difficult to gather external information, especially re
customers and competitors.
iv. Published information may create information overload.

As can be seen in point (i) above, one of the main limitations of external data
is the quality of such information. The quality will depend on a number of
factors: - e.g. who produced the data, how it was collected, when it was
collected and why.

224
17.2 ACCA SYLLABUS GUIDE OUTCOME 2:
Identify and discuss the direct data capture and process costs of
management accounting information.
Identify and discuss the indirect costs of producing information.

17.2.1 Costs of Obtaining Internal Information

The most expensive cost of producing information is probably the cost of


wages. People are required to collect data, input data into the system,
process the data and then output the resulting information70.

An organization can divide the costs of collecting, processing and producing


internal information into three types:

1. Direct data capture costs


2. Processing costs
3. Indirect costs

17.2.1.1 Direct Data Capture Costs

Direct data capture is a type of data input in which there is no data entry. Data
is input into the computer through a reader. It is collected for a particular
purpose (e.g. barcodes being read at a supermarket so that the product can
be identified or account details being read directly from the chip embedded in
the credit card71.) There are obviously costs related to direct data capture, e.g.
in linking the different barcodes to the different products.

17.2.1.2 Processing Costs

This would include costs relating to processing and analyzing the information,
e.g. costs relating to input data and to analyse that data to get more useful
information.

17.2.1.3 Indirect Costs

Information can be used inefficiently. For example,


Reduced quality of information, due to information overload
(sometimes information is collected but not required);
Poor decision making, due to information overload;

70 http://www.teachict.com/as_a2_ict_new/ocr/AS_G061/311_data_info_knowledge/cost_of_inform

ation/miniweb/pg5.htm

71 http://wiki.answers.com/Q/What_is_the_difference_between_direct_data_entry_and_indirect_dat

a_entry

225
Too many areas to focus on so issues are not followed up;
Focus on the wrong things i.e. only on those business areas and
targets that are easy to measure and report on.

17.2.2 Costs of Obtaining External Information

There are five types of costs: -

1. Direct search costs, for e.g. subscription costs to databases, journals,


etc; costs of marketing research surveys (an example of primary data)

2. Indirect access costs: time spent on finding useful information for a


particular project, etc.

3. Management costs: the costs of recording, processing and


disseminating information. As already discussed, sometimes time is
wasted as a result of information overload or duplication of information.

4. Infrastructure costs: the use of the internet requires installation of


networks, servers etc.

5. Time-theft: lost and wasted time even through the unauthorised use
of internet facilities.

Lecture Example 1

The directors of X-tra Ltd, a manufacturing company, are considering


expanding their operations and setting up in a new country.

Required: -

1. Identify and briefly discuss four sources of external information that the
directors may wish to consider.

2. There are specific costs in obtaining external data. Identify these


different types of costs involved and give examples.

Management Reports

226
Management reports should contain good information and should be
communicated via the right channels = ACCURATE

1. ACCURATE
2. COMPLETE
3. COST/BENEFIT its cost must be less than the value of the benefits it
provides
4. UNDERSTANDABLE to the manager using it
5. RELEVANT and not excessive
6. ACCESSIBLE via the right channels
7. TIMELY - communicated at the proper time
8. EASY TO USE

17.3 ACCA SYLLABUS GUIDE OUTCOME 3:


Discuss the principal controls required in generating and distributing
internal information.

17.3.1 Controls over generating internal information

Controls have to be in place when generating internal information for routine


and ad-hoc reports. Internal information is confidential, and very often,
commercially sensitive.

Before starting any report (routine or ad-hoc), a cost/benefit analysis has to be


undertaken: what are the costs involved in preparing such a report versus the
benefits obtained by having such information?

For routine reports, it is important that the company carries out certain
procedures: -
1. Formats, layouts and definitions used in reports should be consistent
so as to avoid misinterpretations and time wastage on amended
layouts etc. The writer/s of such report should be specified.
2. Users requirements should be clearly understood so that the report will
be targeted to meet those requirements. Once prepared, the report
should be assessed against those requirements.
3. The report must also indicate the actions that users can take on the
information presented in such a report.

Ad-hoc reports should only be prepared if the requested information does not
already exist in a different format. Such reports cost money and therefore, the
writers of such reports should be given only relevant and up-to-date
information. In order to increase the reliability of such reports, the writers
should be clearly identified.

17.3.2 Controls over distributing internal information

227
The organization should set out a number of controls re the distribution of
information. Information should be available to authorized persons and third
parties only. Distribution lists should be prepared, indicating who should
receive the reports. Procedure manuals are usually also available indicating
the formats of such reports and when routine reports should be issued.

All employees should be required not to divulge confidential information to


third parties. This is usually specified in contracts of employment.

The use of email is also very important in distributing information. Emails are
usually undesirable for confidential information and long reports.

Information should be physically safeguarded. Reports should be locked in


and protected by passwords. Firewalls should also be used to protect data
from external access.

17.4 ACCA SYLLABUS GUIDE OUTCOME 4:


Discuss the procedures that may be necessary to ensure security of
highly confidential information that is not for external consumption.

Each individual granted access to electronic and/or hard copy data holds a
position of trust within the organisation and must preserve the security and
confidentiality of the information to which he/she is granted access to.
Therefore, the organisation must ensure that no data, in any format, is
divulged to unauthorized third parties.

Confidential information must be stored in such a way as to ensure that only


authorised persons can access it. Policies and procedures must be in place
for the secure disposal/destruction of confidential information.

How can we ensure the security of highly confidential information that is not
for external consumption?

1. Passwords: Passwords can be a very good means of control. Each


password is allocated suitable access rights. Users must follow good
security practices in the selection and use of passwords.

2. Physical and Logical access control: Physical access control is


concerned with preventing unauthorized persons gaining access to the
hardware, e.g. locks, doors etc. Logical access control prevents
unauthorized persons from gaining access to data or software.

3. Database security controls: These controls are required as, in theory,


the database can be accessed by a large number of people. Security
risks to database systems include unauthorized access, deletion or
damage to the data or programs, leakage of confidential information.

228
Computer security inference controls attempt to prevent users from
inferring this information.

4. Firewalls: A firewall can be software-based or hardware-based and is


used to prevent unauthorized access into company systems.

5. Encryption: Encoding messages (or information) in such a way that


eavesdroppers or hackers cannot read it, but authorized parties can.

6. Anti-virus and anti-spyware software: This software has been


developed to counteract computer viruses and spyware. Spyware is
software that monitors a users computer. It can collect any type of
data, including personal information, and can change computer settings
and install additional software. Hence, anti-spyware software scans the
software for any spyware and blocks any software which represents
spyware.

7. Personal data: Today, countries have introduced the Data Protection


Act to safeguard individuals from having their personal information
transferred to unauthorized third parties.

Further questions

Question 172

The following are all types of control within an organisation:

(i) Logical access controls


(ii) Database controls
(iii) Hierarchical passwords
(iv) Range checks

Which of the above controls help to ensure the security of highly confidential
information?

A. (i) and (ii) only


B. (i) and (iii) only
C. (i), (ii) and (iii) only
D. All of the above

72 Specimen Exam Applicable from December 2014

229
CHAPTER 18
Performance Analysis in Private
Sector Organisations
18.1 ACCA SYLLABUS GUIDE OUTCOME 1:
Describe, calculate and interpret financial performance indicators
(FPIs) for profitability, liquidity and risk in both manufacturing and
service businesses. Suggest methods to improve these measures

A key aspect of performance measurement is ratio analysis. Ratios are of little


use in isolation. Firms can use ratio analysis to compare: -
1. budgets, for control purposes
2. last years figures to identify trends
3. competitors results and/or industry averages to assess performance

18.1.1 Measuring Profitability

18.1.1.1 Sales Growth

Sales in current year - Sales in previous year


Sales in previous year

In looking at sales growth, we usually consider other factors such as inflation.


Hence, we analyse sales also in real terms.

18.1.1.2 Return on Capital Employed

The main ratio to measure profitability in an organization is return on capital


employed (ROCE).

ROCE = Net Profit before interest and tax x 100


Capital Employed

Capital employed is defined as total assets less current liabilities or share


capital and reserves plus long term capital. It represents the percentage of
profit being earned on the total capital employed; and relates profit to capital
invested in the business. Capital invested in a corporate entity is only
available at a cost corporate bonds or loan stock finance generate interest
payments and finance from shareholders requires either immediate payment
of dividends or the expectation of higher dividends in the future.

230
The primary ratio measuring overall return is analysed in more detail by using
secondary ratios:

Asset turnover
Net Profit margin net profit before interest and tax as a percentage of
sales

These two separate factors, or a combination of both, influence the return


achieved by the business entity.

18.1.1.3 Asset Turnover

The asset turnover is a measure of utilisation and management efficiency. It


indicates how well the assets of a business are being used to generate sales
or how effectively management have utilised the total investment in
generating income.

Asset Turnover = Turnover


Capital Employed

18.1.1.4 Net Profit Margin

The profit margin indicates how much of the total revenue remains to provide
for taxation and to pay the providers of capital, both interest and dividends.
This return to sales can be directly affected by the managements ability to
control costs and determine the most profitable sales mix.

Net Profit Margin = Net Profit x 100


Turnover

18.1.1.5 Gross Profit Margin

Gross Profit Margin = Gross Profit x 100


Turnover

18.1.2 Measuring Liquidity

Liquidity is the ability of an organization to pay its debts when they fall due.
There are two main measures of liquidity: -

1. the current ratio


2. the quick (or acid test) ratio

231
18.1.2.1 Current Ratio

The current ratio is expressed as:

Current assets : Current Liabilities

If current assets exceed current liabilities then the ratio will be greater than 1
and indicates that a business has sufficient current assets to cover demands
from creditors. However, the speed at which stock can be converted into cash
flow is such that it is not prudent to regard stock as available to cover
creditors.

18.1.2.2 Quick (Acid Test) Ratio

This is expressed as:

Current assets Stocks: Current Liabilities

If this ratio is 1:1 or more, then clearly the company is unlikely to have liquidity
problems. If the ratio is less than 1:1 we would need to analyse the structure
of current liabilities, to those falling due immediately and those due at a later
date.

Measures of utilisation (measures of efficiency) include:

1. Debtors collection period


2. Creditors payment period
3. Stock turnover or stock days

18.1.2.3 Debtors (receivables) collection period

This is a measure of managements efficiency and is expressed as:

Debtors x 365 days


Sales

This is an indicator of the effectiveness of the companys credit control


systems and policy. The control of debtor days is an important element of
working capital management.

232
18.1.2.4 Creditors (payables) period

The balance between debtor and creditor days is influenced by the working
capital cycle. The creditor days is a measure of how much credit, on average,
is taken from suppliers. It is expressed as:

Creditors (trade) x 365 days


Purchases

This ratio is an aid to assessing company liquidity, as an increase in creditor


days is often a sign of inadequate working capital control.

18.1.2.5 Inventory holding period

This is expressed as:

Inventory x 365 days


Cost of sales

The holding period may increase because of: -

1. Build-up of inventory levels as a result of increased capacity following


expansion of non-current assets.
2. Increasing inventory levels in response to increased demand for
product.

18.1.2.6 Work-in-progress period

This is expressed as:

Value of WIP x 365 days


Cost of Sales

18.1.2.7 Finished goods period

It is expressed as:

Value of Finished Goods x 365


Cost of Sales

This is a further measure of working capital management and relates to stock


turnover. Controls need to be maintained so that liquidity is not sacrificed.

233
18.1.3 Measuring Risk

Measurement of risk considers the financial risk incurred by borrowing.

18.1.3.1 Financial Gearing

Financial gearing can be calculated as: -

Debt x 100%
Equity

OR

Debt x 100%
Debt + Equity

If the firm has excessive debt, then the need to pay interest before dividends
will increase the risks faced by shareholders if profits fall.

18.1.3.2 Interest Cover

Interest cover is expressed as:

Profit before interest and tax = Number of times

Interest paid

This ratio represents the number of times that interest could be paid out of
profit before interest and tax.

18.1.3.3 Dividend Cover

This is determined by dividing profit available to equity holders by the dividend


for the year.

It is expressed as:

Earnings after tax and preference dividends = Number of times

Ordinary dividend

This is an indication of dividend policy whether profits tend to be distributed


or reinvested.

234
18.1.3.4 Operating Gearing

Operating gearing refers to the proportion of a companys operating costs that


are fixed as opposed to variable.

Operating gearing = Fixed Costs


Total Costs

The higher the proportion of fixed costs, the higher the operating gearing.
Companies with high operating gearing tend to have volatile operating profits.
This is because fixed costs remain the same, no matter the volume of sales.

Thus, if sales increase, operating profit increases by a larger percentage. But


if sales volume falls, operating profit falls by a larger percentage. Generally, it
is a high-risk policy to combine high financial gearing with high operating
gearing. High operating gearing is common in many service industries where
many operating costs are fixed.

Lecture Example 1

Which of the following would be used to assess the liquidity of a


company?

i. Return on capital employed


ii. Gross profit margin
iii. Quick / acid test ratio
iv. Debt to equity

A. (i) and (ii) only


B. (iii) only
C. (iv) only
D. (iii) and (iv) only

Lecture Example 2

Thatcher International Park (TIP) is a theme park and has for many years
been a successful business, which has traded profitably. About three years
ago the directors decided to capitalise on their success and reduced the
expenditure made on new thrill rides, reduced routine maintenance where
possible (deciding instead to repair equipment when it broke down) and made
a commitment to regularly increase admission prices. Once an admission
price is paid customers can use any of the facilities and rides for free.

235
These steps increased profits considerably, enabling good dividends to be
paid to the owners and bonuses to the directors. The last two years of
financial results are shown below.

2008 2009
$ $
Sales 5,250,000 5,320,000
Less expenses:
Wages 2,500,000 2,200,000
Maintenance routine 80,000 70,000
Repairs 260,000 320,000
Directors salaries 150,000 160,000
Directors bonuses 15,000 18,000
Other costs (including depreciation) 1,200,000 1,180,000
Net profit 1,045,000 1,372,000

Book value of assets at start of year 13,000,000 12,000,000


Dividend paid 500,000 650,000
Number of visitors 150,000 140,000

TIP operates in a country where the average rate of inflation is around 1% per
annum.

Required:

(a) Assess the financial performance of TIP using the information given
above. (14 marks)

(ACCA Paper F5 December 2009 Qs 4a)

18.2 ACCA SYLLABUS GUIDE OUTCOME 2:


Describe, calculate and interpret non-financial performance indicators
(NFPIs) and suggest method to improve the performance indicated

In recent years, the trend in performance measurement has been towards a


broader view of performance, covering both financial and non-financial
indicators. The most well-known of these approaches is the balanced
scorecard proposed by Kaplan and Norton, which we will be describing in
Section 18.4.

Areas to measure should relate to an organisation's critical success factors.


Critical success factors (CSFs) are performance requirements which are
fundamental to an organisation's success (for example innovation in a
consumer electronics company) and can usually be identified from an
organisation's mission statement, objectives and strategy. Key performance
indicators (KPIs) are measurements of achievement of the chosen critical
success factors. Key performance indicators should be:

236
specific (i.e. measure profitability rather than 'financial performance', a
term which could mean different things to different people)
measurable (i.e. be capable of having a measure placed upon it, for
example, number of customer complaints rather than the 'level of
customer satisfaction')
relevant, in that they measure achievement of a critical success factor.

The following table demonstrates critical success factors and key performance
indicators of a college training ACCA students73.

Perspective Critical Success Key Performance Indicators


Factor
Financial Shareholder Dividend yield; % increase in share price
success wealth
Cashflow Actual vs Budget
Debtor days
Exam success College pass rate vs national average
Customer Premier college status
satisfaction Tutor grading by students
Flexibility Average number of course variants per
subject (eg full-time, day release,
evening)
Process Resource % room occupancy
efficiency utilisation Average class size
Average tutor teaching load (days)
Growth Innovation % of sales from < 1 year old
products Number of online enrolments
Information
technology

18.3 ACCA SYLLABUS GUIDE OUTCOME 3:


Analyse past performance and suggest ways for improving financial
and non-financial performance

We need to measure performance in order to assess how the organization


has performed. How do we analyse performance?

73 Performance Measurement by S. Jay, Student Accountant, April 2004,


http://www.accaglobal.com/en/student/qualification-resources/acca-qualification/acca-exams/f5-
exams/exams-f52/performance-measurement.html

237
As already discussed, we need to compare actual performance against a
target, which is usually the budget, or against prior performance. Hence, we
can identify whether there is any cause for concern.

If there is a cause for concern, we need to identify the reasons leading to this
poor or unexpected performance. What has labour efficiency declined? Are
we providing the necessary training to our staff? Are we recruiting staff with
the right skills or having the necessary experience?

Once these reasons have been identified, we need to consider ways to


improve performance. Hence, if labour efficiency has declined over the last
year, we may need to take different courses of action: e.g. provide the right
training to all staff, improve the recruitment process by hiring more
experienced staff and targeting more specialists to the job.

18.4 ACCA SYLLABUS GUIDE OUTCOME 4:


Explain the causes and problems created by short-termism and
financial manipulation of results and suggest methods to encourage a
long term view

Short-termism is when there is a bias towards short-term rather than long-


term performance. Managers may manipulate results especially managers'
performance is measured on short-term results. For example:

a. Postponing capital expenditure projects in order to protect short term


cash flow and profits.
b. Cutting R&D expenditure to save operating costs, and so reducing the
prospects for future product development.
c. Reducing quality control, to save operating costs.
d. Reducing the level of customer service, to save operating costs.
e. Cutting training costs or recruitment of new employees.
f. Postponing maintenance of machinery to later years.

Steps should be taken to encourage managers to take a long-term view: -

a. Making short-term targets realistic.


b. Providing sufficient management information to allow managers to see
what trade-offs they are making. Managers must be kept aware of long-
term aims as well as shorter-term (budget) targets.
c. Evaluating managers' performance in terms of contribution to long-term
as well as shorter objectives.
d. Link managers' rewards to share price.
e. Set quality based targets as well as financial targets.

238
18.5 ACCA SYLLABUS GUIDE OUTCOME 5:
Explain and interpret the Balanced Scorecard, and the Building Block
model proposed by Fitzgerald and Moon
Discuss the difficulties of target setting in qualitative areas

18.5.1 Balanced Scorecard

The Balanced Scorecard was popularised by Robert Kaplan and David Norton
in 1992. The rationale for the development of the Balanced Scorecard was a
growing dissatisfaction with traditional, financial measures of performance.
The balanced scorecard approach emphasises the need to provide
management with a set of information which covers all relevant areas of
performance in an objective and unbiased fashion.

The scorecard designed by Kaplan and Norton contains four key groupings of
performance measures. These four groupings, called perspectives by Kaplan
and Norton, were considered sufficient to track the key drivers of both current
and future financial performance of the firm. The perspectives focused on the
achievements of the firm in four areas 74: -

1. The financial perspective concentrates on how the firm appears to its


shareholders and considers what the firms financial objectives are.
The measures used to assess whether these objectives are being
achieved typically include, profit, sales, ROI, cash flow or economic
value added (EVA).

2. The customer perspective focuses on the question, what must the


firm do to satisfy its customers so as to achieve its financial objectives?

Outcome measures for the customer perspective generally include


measures of customer satisfaction, market share, customer retention
and customer profitability. These outcome measures can be sub-
divided into driver measures, such as measures relating to lead times,
on-time delivery, product quality and product cost.

3. The internal business perspective considers the question, what must


the firm do well internally in order to support the product/market
strategy and to achieve its financial objectives? Typical outcome
measures include those relating to innovation (product and process)
and operations (cycle times, defect rates).

4. In the learning and growth perspective, the measures focus on the


question what infrastructure must the firm build to create long-term
growth and improvement? In other words, what capabilities must be
improved or acquired to achieve the long-term targets for the customer

74 Examined December 2014 F5 Qs 4a

239
and internal business process perspectives? Outcome measures may
include metrics on employee satisfaction, training and retention.

Perspective Question Explanation


Customer To achieve our vision, how Gives rise to targets that matter to
should we appear to our customers: cost, quality, delivery,
customers? inspection, handling and so on.
Internal What processes must we excel Aims to improve internal processes
at to achieve our financial and and decision making.
customer objectives?
Learning and Can we continue to improve Considers the business's capacity to
growth and create future value? maintain its competitive position
through the acquisition of new skills
and the development of new
products.
Financial How do we create value for our Covers traditional measures such as
shareholders? growth, profitability and shareholder
value but set through talking to the
shareholder or shareholders direct.

The following is an example of a balanced scorecard: -

240
The balanced scorecard approach to performance measurement offers
several advantages:

1. it measures performance in a variety of ways, rather than relying on one


figure
2. managers are unlikely to be able to distort the performance measure - bad
performance is difficult to hide if multiple performance measures are used
3. it takes a long-term perspective of business performance
4. success in the four key areas should lead to the long-term success of the
organisation
5. it is flexible - what is measured can be changed over time to reflect
changing priorities
6. 'what gets measured gets done' - if managers know they are being
appraised on various aspects of performance they will pay attention to
these areas, rather than simply paying 'lip service' to them.

241
The main difficulty with the balanced scorecard approach is setting standards
for each of the KPIs. This can prove difficult where the organisation has no
previous experience of performance measurement. Benchmarking with other
organisations is a possible solution to this problem75.

Lecture Example 4

Following from Lecture Example 3: -

During the early part of 2008 TIP employed a newly qualified management
accountant. He quickly became concerned about the potential performance of
TIP and to investigate his concerns he started to gather data to measure
some non-financial measures of success. The data he has gathered is shown
below:

Table 1

2008 2009
Hours lost due to breakdown of rides (see note 1) 9,000 hours 32,000 hours
Average waiting time per ride 20 minutes 30 minutes

Note 1: TIP has 50 rides of different types. It is open 360 days of the year for
10 hours each day.

Required:

(a) Assess the quality of the service that TIP provides to its customers
using Table 1 and any other relevant data and indicate the risks it is
likely to face if it continues with its current policies. (6 marks)

(ACCA Paper F5 December 2009 Qs 4b)

Lecture Example 576

A companys sales and cost of sales figures have remained unchanged for the
last two years. The following information has been noted:

75 Performance Measurement by S. Jay, Student Accountant, April 2004,


http://www.accaglobal.com/en/student/qualification-resources/acca-qualification/acca-exams/f5-
exams/exams-f52/performance-measurement.html
76 Examiners report, June 2015

242
The following statements have been made about the companys performance
for the most recent year:
(i) Customers are taking longer to pay and this may have
contributed to the decline in the companys current ratio.
(ii) Inventory levels have decreased and this may have
contributed to the decline in the companys quick ratio.

Which of the above statements is/are true?


A. (i) only
B. (ii) only
C. Both (i) and (iii)
D. Neither (i) nor (ii)

Lecture Example 6

Why would a company want to encourage the use of non-financial


performance indicators?

A. To encourage short termism


B. To look at the fuller picture of the business
C. To enable results to be easily manipulated to the benefit of the manager
D. To prevent goal congruence

Lecture Example 7

Clown plc monitors the % of total sales derived from products


developed in the last year. Which part of the balanced scorecard would
this measure be classified under?

A. Financial perspective
B. Customer perspective
C. Internal perspective
D. Learning and growth perspective

243
Lecture Example 8

PS Ltd has calculated the following indicators


i. Return on capital employed
ii. Training costs as a percentage of total costs

Which of the balanced scorecard perspectives would these measures


relate to?

(i) (ii)
A. Financial Financial
B. Financial Internal
C. Internal Learning and growth
D. Financial Learning and growth

Lecture Example 9

Take-out Ltd is a food delivery company. Which of the following would


be appropriate to measure customer perspective and internal
perspective of the balanced scorecard?

Customer Internal
A. Number of customer complaints Time taken from order to deliver food
B. Cost per delivery Cost of time spent on training
C. Number of late deliveries Profit per delivery
D. Cost of delivery vehicles Gross profit percentage

18.5.2 The Building Block Model

Fitzgerald et al (1993) and Fitzgerald & Moon (1996) consider performance


measurement in service businesses. Their work attempts to overcome the
problems associated with performance measurement of service businesses.

There are particular characteristics of service businesses which will affect


performance and its measurement77. These are:

Simultaneity/ spontaneity (production and consumption of the service


coinciding);
perishability (the inability to store the service);
heterogeneity (variability in the standard of performance of the
provision of the service);
intangibility (of what is provided to and valued by individual customers).

77 Examined June 2012 Qs 2b

244
The figure found after section 18.4.2.2 shows their building blocks for
dimensions, standards and rewards for performance measurement systems.

Dimensions can be divided into two sets: -


The results, measured by financial performance (profitability,
capital structure) and competitiveness (sales growth, market
share)
The determinants (what determines competitive and financial
performance): -
1. Quality of service
2. Flexibility
3. Resource utilization
4. Innovation

18.5.2.1 Standards

These are ownership, achievability and equity.

a. Employees need to participate in the budget and standard-setting


processes. They are then more likely to accept the standards.
b. Standards need to be set high enough to ensure that there is some
sense of achievement in attaining them, but not so high that there is a
demotivating effect because they are unachievable.
c. Equity is seen to occur when applying standards for performance
measurement purposes.

18.5.2.2 Rewards

If the performance measurement system is to operate successfully; clarity,


motivation and controllability are required.

a. Objectives need to be clearly understood by those whose performance


is being appraised.
b. Individuals should be motivated to work in pursuit of the organisation's
strategic objectives.
c. Managers should be accountable for their areas of responsibility. For
example they should not be held responsible for costs over which they
have no control.

245
The building blocks for performance measurement systems
(Fitzgerald and Moon 1996)

Dimensions
Profit
Competitiveness
Quality
Resource utilisation
Flexibility
Innovation

Rewards
Standards
Clarity
Ownership
Motivation
Achievability
Controllability
Equity

How to answer Performance Measurement questions 78

Present calculations in a referenced list.


Dont consider any one piece of information or number in isolation.
Use headings wherever possible and avoid writing a sea of words.
When you are writing a statement, e.g. sales have increased by 1.3%
always ask yourself the question why do I care?. This will help you
make a meaningful point and take a thought through to its logical
conclusion.
Read all the requirements and make sure that you dont start talking
about, e.g. requirement (b) in requirement (a), as you will then find that
you have nothing to say when you get to requirement (b).
Use the marks available as a guide as to how much to write. There are
no set marking rules such as one mark per valid point. Marks vary
from question to question.

Further Questions

Question 1

Nicholson sells mobile telephones. It supplies its customers with telephone


handsets and wireless telephone connections. Customers pay an annual fee
plus a monthly charge based on calls made. The company has recently
employed a consultant to install a balanced scorecard system of performance

78 Article Performance Measurement by A. Irons, Student Accountant, 2010,


http://www.accaglobal.com/content/dam/acca/global/pdf/sa_jul10_f5_perf_meas.pdf

246
measurement and to benchmark the results against those of Nicholsons
competitors. Unfortunately the consultant was called away before the work
was finished. You have been asked to complete the work. The following data
is available:

Nicholson Operating data for the year ended 30 November 2007

Sales revenue $480 million


Sales attributable to new products $8 million
Average capital employed $192 million
Profit before interest and tax $48 million
Average number of customers 1,960,000
Number of telephones returned for repair 10,000
Number of bill queries 12,000
Number of customer complaints 21,600
Number of customers lost 117,600
Average number of bill queries unresolved
at the end of each day 118
Average number of telephones unrepaired
at the end of each day 804

Required:

a. Calculate the following ratios and other statistics for Nicholson for the
year ended 30 November 2007:
i. Return on capital employed;
ii. Return on sales (net profit percentage);
iii. Asset turnover;
iv. Annual number of complaints per thousand customers;
v. Percentage of customers lost per annum;
vi. Average time to resolve billing queries;
vii. Average wait for a telephone repair;
viii. Percentage of sales attributable to new products.

b. The following information is for the mobile phone industry for the year
ended 30 November 2007.

Industry average statistics Mobile Telephones

Annual number of complaints per 1,000 customers 5


Percentage of customers lost per annum 3%
Average time to resolve billing queries 14 days
Average wait for a telephone repair 2 days
Percentage of sales attributable to new products 20%
Return on capital employed 15%
Return on sales (net profit percentage) 5%
Asset turnover 3 times

247
Required:

Using the industry average information and your answer to part (a), discuss
the performance of Nicholson in the year ending 30 November 2007 under the
four balanced scorecard headings of:
a. financial success;
b. customer satisfaction;
c. process efficiency; and
d. organisational learning and growth.

Note: state any assumptions that you make

(CAT Paper T7 December 2007)

Question 2

The use of the balanced scorecard rather than a profit-based measure is likely
to help solve the following problems:

(1) Subjectivity
(2) Short-termism

Which is/are true?

A. 1 only
B. 2 only
C. Both1 and 2
D. Neither 1 nor 2

Question 379

The following statements have been made about the balanced scorecard:

(1) It focuses solely on non-financial performance measures


(2) It looks at both internal and external matters concerning the organisation

Which of the above statements is/are true?

A. 1 only
B. 2 only
C. Neither 1 nor 2
D. Both 1 and 2

79 Specimen Exam Applicable from December 2014

248
Question 4

HH plc monitors the % of total sales that derives from products developed in
the last year. Which part of the balanced scorecard would this measure be
classified under?

A. Financial perspective
B. Customer perspective
C. Internal perspective
D. Learning perspective

Question 5

A company wants to encourage an investment centre to make new


investments. Performance measurement using which of the following KPIs
would achieve this?

A. ROI
B. ROCE
C. RI
D. IRR

Question 6

Why would a company want to encourage the use of non-financial


performance indicators?

A. To encourage short termism


B. To look at the fuller picture of the business
C. To enable results to be easily manipulated to the benefit of the
manager
D. To prevent goal congruence

Question 7

A company has current assets of $1.8m, including inventory of $0.5m, and


current liabilities of $1.0m.

What would be the effect on the value of the current and acid test ratios if the
company bought more raw material inventory on three months credit?

Current ratio Acid test


A. Increase Decrease
B. Decrease Increase
C. Increase Decrease
D. Decrease Decrease

249
CHAPTER 19
Divisional Performance
Measurement
Decentralisation is the delegation of decision-making to lower levels of
management.

One disadvantage of decentralisation is that managers may make decisions


that are not in the best interests of the overall company (dysfunctional
decisions). Hence, senior managers need to introduce systems of
performance measurement to ensure that decisions made by junior managers
are in the best interests of the company as a whole.

19.1 ACCA SYLLABUS GUIDE OUTCOME 1:


Explain the meaning of, and calculate from supplied data, return on
investment (ROI) and residual income (RI) in the context of divisional
performance appraisal80

In an investment centre, managers have the responsibilities of a profit centre


plus responsibility for capital investment. Two measures of divisional
performance are commonly used:

1. Return on investment (ROI) =


controllable (traceable) profit %
controllable (traceable) investment

2. Residual income = controllable (traceable) profit - an imputed


interest charge on controllable (traceable) investment.

What is the difference between controllable or traceable profit?

The main problem with measuring performance is in deciding which costs are
controllable and which costs are traceable. The performance of a manager is
indicated by the controllable profit and the success of the division as a whole
is judged on the traceable profit.

80 Examined Sept/Dec 2015 Qs 5

250
Controllable costs and revenues are those costs and revenues which result
from decisions within the authority of a particular manager within the
organization. These should be used to assess the performance of the
managers.
For example, depreciation on machinery in Division A is a traceable fixed cost
because profit centre managers do not have control over the investment in
non-current assets.

Most variable costs are controllable in the short term because managers can
influence the efficiency with which resources are used.

Some costs are non-controllable, such as increases in expenditure items due


to inflation. Other costs are controllable in the long term rather than the short
term. For example, production costs might be lower by the introduction of
new machinery. However, its results will be seen in the long term

Illustration 1

In 2011 a divisions controllable return on investment was 25% and its


controllable profit was $80,000. The cost of capital was 18% per annum.

What was the divisions controllable residual income in the last year?

$80,000____ = 25%
Capital Employed

Capital Employed = 80,000 = $320,000


25%

Profit 80,000
Imputed Interest (320 x 18%) 57,600
Residual Income 22,400

Lecture Example 1

Division A is a division of Abco Co. It earns a profit of $2.5m and its net assets
are presently $10m.

The cost of capital for Division A is 10% p.a.

Division A is considering the following investment:

Capital required for investment $1m

Profit to be generated by investment $0.3m

251
Required:

a) Calculate the present return on investment and residual income


for the division.
b) Calculate the return on investment and residual income of the new
investment. Would the manager of Division A accept this
investment, taking into consideration both ROI and RI.

19.2 ACCA SYLLABUS GUIDE OUTCOME 2:


Discuss the shortcomings and benefits of using ROI and RI for
divisional performance appraisal

19.2.1 Relative merits of ROI and Residual Income81

Return on investment is a relative measure and hence suffers accordingly. For


example, assume you could borrow unlimited amounts of money from the
bank at a cost of 10% per annum. Would you rather borrow $100 and invest it
at a 25% rate of return or borrow $1m and invest it at a rate of return of 15%?

Although the smaller investment has the higher percentage rate of return, it
would only give you an absolute net return (residual income) of $15 per
annum after borrowing costs. The bigger investment would give a net return of
$50,000. Residual income, being an absolute measure, would lead you to
select the project that maximises your wealth.

Residual income also ties in with net present value, theoretically the best way
to make investment decisions. In the long run, companies that maximise
residual income will also maximise net present value and in turn shareholder
wealth. Residual income does, however, experience problems in comparing
managerial performance in divisions of different sizes. The manager of the
larger division will generally show a higher residual income because of the
size of the division rather than superior managerial performance.

Lecture Example 2

Is each of the following an advantage of residual income as a measure


of divisional performance over return on investment?
. Yes No
It helps in comparing performance of the managers of divisions of
different sizes

81 Performance Measurement by S. Jay, Student Accountant, April 2004,


http://www.accaglobal.com/en/student/qualification-resources/acca-qualification/acca-exams/f5-
exams/exams-f52/performance-measurement.html

252
It ensures that managers will select projects with positive net
present values (NPV)
It is directly related to net present value (NPV)

It makes divisional managers aware of the cost of financing their


divisions
It is more easily understood by divisional managers

It avoids short term dysfunctional decision making

It gives an absolute measure of performance

It relates the size of the division's income to the size of the


investment

19.2.2 Problems common to both ROI and Residual Income82

The following problems are common to both measures:

1. Identifying controllable (traceable) profits and investment can be


difficult.
2. If used in a short-term way they can both over-emphasise short-term
performance at the expense of long-term performance. Investment
projects with positive net present value can show poor ROI and
residual income figures in early years leading to rejection of projects by
managers.
3. If assets are valued at net book value, ROI and residual income figures
generally improve as assets get older. This can encourage managers
to retain outdated plant and machinery.
4. Both techniques attempt to measure divisional performance in a single
figure. Given the complex nature of modern businesses, multi-faceted
measures of performance are necessary.
5. Both measures require an estimate of the cost of capital, a figure which
can be difficult to calculate.

Worked out Example83 (similarly examined in June 2012)

PQR plc is considering opening a new division to manage a new investment


project. Forecast cashflows of the new project are as follows:

82 Performance Measurement by S. Jay, Student Accountant, April 2004,


http://www.accaglobal.com/en/student/qualification-resources/acca-qualification/acca-exams/f5-
exams/exams-f52/performance-measurement.html
83 Performance Measurement, by S. Jay, Student Accountant, April 2004,

http://www.accaglobal.com/en/student/qualification-resources/acca-qualification/acca-exams/f5-
exams/exams-f52/performance-measurement.html

253
Year 0 1 2 3 4 5
Forecast net cash flow $m (5.0) 1.4 1.4 1.4 1.4 1.4

PQR's cost of capital is 10% pa. Straight line depreciation is used.

Required:

Calculate the project's projected ROI and residual income over its five-
year life.

ROI

Year 1 2 3 4 5
1 Opening investment at net book value 5.0 4.0 3.0 2.0 1.0
2 Forecast net cash flow $m 1.4 1.4 1.4 1.4 1.4
3 Straight line depreciation (1.0) (1.0) (1.0) (1.0) (1.0)
4 Profit 0.4 0.4 0.4 0.4 0.4

ROI (4 1 x 100) 8% 10% 13% 20% 40%

Residual income

Year 1 2 3 4 5
Profit (as above) 0.4 0.4 0.4 0.4 0.4
Imputed capital charge (opening investment x 10%) 0.5 0.4 0.3 0.2 0.1
Residual income (0.1) 0.0 0.1 0.2 0.3

Comment: the poor ROI and residual income figures in the first year could
lead managers to reject the project. However, it shows the tendency for both
ROI and residual income to improve over time. Despite constant annual
cashflows, both measures improve over time as the net book value of assets
falls. This could encourage managers to retain outdated assets.

Lecture Example 3

In the last year a divisions controllable return on investment was 25% and its
controllable profit was $80,000. The cost of finance appropriate to the division
was 18% per annum.

254
What was the divisions controllable residual income in the last year?
A. $5,600 $80,000 (0.25 0.18)

B. $22,400 $80,000 ($80,000 0.25 0.18)

C. $74,400 $80,000 ($80,000 (0.25 0.18)

D. $76,400 $80,000 ($80,000 0.25 0.18)

Lecture Example 4

An investment centre has net assets of $1,000,000, and made profits before
interest of $200,000. The notional cost of capital is 10%.

The investment centre has an opportunity to invest in a new product costing


$100,000. The project would have a four-year life, and would make cash
profits of $50,000 each year.

Required:

(a) What would be the average ROI with and without the investment?
Would the investment centre manager wish to undertake the
investment if performance is judged on ROI in Year 1?

(b) What would be the average annual RI with and without the
investment? Would the investment centre manager wish to
undertake the investment if performance is judged on RI in Year 1?

To calculate ROI and RI, use the value for capital employed as at the
start of Year 1.

Further Questions

Question 1

The manager of a division is considering a new project. The project is


expected to increase the divisions annual net profit by $153,900, but it will
cause net current assets to rise by $810,000.

The managers performance is evaluated against a target Return on


Investment. The target is 18%. However the use of Residual Income is being
considered.

The cost of capital for the division is 16%.

255
Would the manager adopt the project if the performance measure was either
(i) Return on Investment (ROI) or (ii) Residual Income (RI)?

(i) ROI (ii) RI


A. Yes Yes
B. Yes No
C. No Yes
D. No No

Question 2

The Northern Division of Gemas Co. currently earns a return on investment of


15.5% based on capital employed of $2,680,000. The divisional management
team have decided to implement a project which will require an investment of
$320,000. The project is expected to generate a profit of $53,000 per annum.
The divisional cost of capital is 13%.

What will be the residual income of the division after the project is
implemented? _____________

Question 3

A division has a residual income of $240,000 and a net profit before imputed
interest of $640,000.

If it uses a rate of 10% for computing imputed interest on its invested capital,
what is its return on investment (ROI) to the nearest whole number?

A. 4%
B. 10%
C. 16%
D. 27%

Question 4

A division currently earns a return on investment (ROI) of 20%. It is


considering investing in a project which has a residual income (RI) of $1,000
at an imputed interest charge of 20%.

What is the effect on the divisions ROI if the project is undertaken?

A. Increase
B. Decrease
C. Remain the same
D. Not possible to tell from this information

256
CHAPTER 20
Transfer Pricing
20.1 ACCA SYLLABUS GUIDE OUTCOME 1:
Explain the basis for setting a transfer price using variable cost, full
cost and the principles behind allowing for intermediate market
Explain how transfer prices can distort the performance assessment of
divisions and decisions made

Division
A

Planks of wood

External External
Division
Suppliers of Market for
B wooden chairs
wood planks
and tables

Transfer pricing is used when divisions of an organisation need to charge


other divisions of the same organisation for goods and services they provide
to them. Usually, each division will report its performance separately. Hence,
some monetary value must be allocated to record the transfer of these goods
or services. For example, division A might make a component that is used as
part of a product made by division B of the same company, but that can also
be sold to the external market, including makers of rival products to division
B's product.

There will therefore be two sources of revenue for A.

(a) External sales revenue from sales made to other organisations,valued at


the selling price.
(b) Internal sales revenue from sales made to other responsibility centres
within the same organisation, valued at the transfer price.

257
A good transfer price should have the following characteristics: -

1. Preserve divisional autonomy: almost inevitably, divisionalisation is


accompanied by a degree of decentralization in decision making so
that specific managers and teams are put in charge of each division
and must run it to the best of their ability. Divisional managers are
therefore likely to resent being told by head office which products they
should make and sell. Ideally, divisions should be given a simple,
understandable objective such as maximizing divisional profit.

2. Be perceived as being fair for the purposes of performance


evaluation and investment decisions.

3. Permit each division to make a profit: profits are motivating and


allow divisional performance to be measured using positive ROI or
positive RI
.
4. Encourage divisions to make decisions which maximize group
profits: the transfer price will achieve this if the decisions which
maximize divisional profit also happen to maximize group profit this is
known as goal congruence. Furthermore, all divisions must want to do
the same thing. Theres no point transferring out if the next division
doesnt want to transfer in.84

In practice it is difficult to achieve all four aims.

Illustration 1

Division A makes components at a cost of $20/unit. These are transferred to


division B for $60. Division B buys the components in at $60, incurs own costs
of $30 and then sells them to the external customers at $130 per unit.

Division A Division B

Transfer Price --- 60


Own costs 20 30
Divisional profit 40 40
Transfer price / selling price 60 130

Each division is making a profit of $40/unit. The group is making a profit of


$80/unit.

84 K. Garrett, Transfer Pricing, Student Accountant, 10/2009

(http://www.accaglobal.com/content/dam/acca/global/pdf/sa_oct09_garrett3.pdf )

258
Division A $40 + Division B $40 = $80

OR

Final revenue $130 own costs (20+30) $50 = $80

Note that for every $1 increase in the transfer price, Division A will make $1
more profit; Division B will make $1 less profit. Although the group will make
the same profit, changes in profits can result in divisions taking a different
decision. Hence, overall profits might be affected.

20.1.1 Potential benefits of operating a transfer pricing system within a


divisionalised company

Potential benefits of operating a transfer pricing system within a divisionalised


company include the following: -

a. Leads to goal congruence - motivates divisional managers to


make decisions, which improve divisional profit and improve
profit of the organisation as a whole. Hence it prevents
dysfunctional decision making.

b. Transfer prices set at a level that enables divisional performance


to be measured 'commercially'.

c. Divisional autonomy is not undermined. A transfer pricing


system helps to ensure that a balance is kept between divisional
autonomy to provide incentives and motivation, and centralised
authority to ensure that the divisions are all working towards the
same goals of the organisations.

20.1.2 Practical Transfer Pricing

Transfer prices are set using the following techniques: -

a. Market prices
b. Production cost this can be based on variable or full cost
including a mark-up
c. Negotiation

20.1.3 The Use of Market Prices as a basis of Transfer Pricing

If an external market price exists for transferred goods, profit centre managers
will be aware of the price they could obtain or the price they would have to pay

259
for their goods on the external market, and they would inevitably compare this
price with the transfer price.

20.1.3.1 The Merits of Market Value Transfer Prices

1. Divisional autonomy

A transferor division should be given the freedom to sell output on the open
market, rather than to transfer it within the company.

'Arm's length' transfer prices, which give profit centre managers the freedom
to negotiate prices with other profit centres as though they were independent
companies, will tend to result in a market-based transfer price.

2. Corporate profit maximization

In most cases where the transfer price is at market price, internal transfers
should be expected, because the buying division is likely to benefit from a
better quality of service, greater flexibility, and dependability of supply. Both
divisions may benefit from cheaper costs of administration, selling and
transport. A market price as the transfer price would therefore result in
decisions which would be in the best interests of the company or group as a
whole.

3. Divisional performance measurement

Where a market price exists, but the transfer price is a different amount (say,
at standard cost plus), divisional managers will argue about the volume of
internal transfers.

For example, if division X is expected to sell output to division Y at a transfer


price of $8 per unit when the open market price is $10, its manager will decide
to sell all output on the open market. The manager of division Y would resent
the loss of his cheap supply from X, and would be reluctant to buy on the
open market. A wasteful situation would arise where X sells on the open
market at $10, where Y buys at $10, so that administration, selling and
distribution costs would have been saved if X had sold directly to Y at $10, the
market price.

20.1.3.2 The disadvantages of market value transfer prices

1. The market price may be a temporary one, induced by adverse


economic conditions, or dumping, or the market price might depend on
the volume of output supplied to the external market by the profit
centre.

260
2. A transfer price at market value might, under some circumstances, act
as a disincentive to use up any spare capacity in the divisions. A price
based on incremental cost, in contrast, might provide an incentive to
use up the spare resources in order to provide a marginal contribution
to profit.

3. Many products do not have an equivalent market price so that the price
of a similar, but not identical, product might have to be chosen. In such
circumstances, the option to sell or buy on the open market does not
really exist.

4. There might be an imperfect external market for the transferred item,


so that if the transferring division tried to sell more externally, it would
have to reduce its selling price.

20.1.4 Cost-Based Transfer Prices

Transfer prices based on variable/marginal cost

A variable cost approach entails charging the variable cost (equal to


marginal cost) that has been incurred by the supplying division to the
receiving division.

The problem is that with a transfer price at marginal cost, the supplying
division does not cover its fixed costs.

Drawbacks when transfer prices are based on variable/marginal


cost

Although good economic decisions are likely to result, a transfer price


equal to marginal cost has certain drawbacks:

Selling division will make a loss as its fixed costs cannot be covered.
This is demotivating.
Performance measurement is distorted. Selling division is condemned
to making losses while buying division gets an easy ride as it is not
charged enough to cover all costs of manufacture. This effect can also
distort investment decisions made in each division. For example,
buying division will enjoy inflated cash inflows.
There is little incentive for selling division to be efficient if all marginal
costs are covered by the transfer price. Inefficiencies in selling division
will be passed up to buying division. Therefore, if marginal cost is going
to be used as a transfer price, at least make it standard marginal cost,
so that efficiencies and inefficiencies stay within the divisions
responsible for them.

261
There are two approaches to transfer pricing which try to preserve the
economic information inherent in variable costs while permitting the
transferring division to make profits, and allowing better performance
valuation85.

1. Variable cost plus lump sum: transfers are made at variable cost but,
periodically, a transfer is made between the two divisions to account for
fixed costs and profit.

2. Dual pricing: In this approach, the supplying division transfers out at


cost plus a mark up and the receiving division transfers in at variable
cost. Obviously, the divisional current accounts wont agree, and some
period-end adjustments will be needed to reconcile those and eliminate
fictitious interdivisional profits.

20.1.4.1 Transfer prices based on full cost

Under this approach, the full cost (including fixed overheads absorbed)
incurred by the supplying division in making the 'intermediate' product is
charged to the receiving division.

The drawback to this is that the division supplying the product makes no profit
on its work so is not motivated to supply internally.

If a full cost plus approach is used, a profit margin is also included in this
transfer price. The supplying division will therefore gain some profit at the
expense of the buying division.

20.1.4.2 Actual cost versus standard cost

When a transfer price is based on cost, standard cost should be used, not
actual cost. A transfer at actual cost would give the supplying division no
incentive to control costs because all of the costs could be passed on to the
receiving division. Actual cost-plus transfer prices might even encourage the
manager of the supplying division to overspend, because this would increase
divisional profit, even though the organisation as a whole suffers.

Standard cost-based transfer prices should encourage the supplying division


to become more efficient. The problem with this approach is that it penalizes
the supplying division if the standard cost is unattainable, while it penalizes
the receiving division if it is too easily attainable.

85 K. Garrett, Transfer Pricing, Student Accountant, 10/2009,

http://www.accaglobal.com/gb/en/student/acca-qual-student-journey/qual -resource/ acca-


qualification/f5/technical-articles/transfer-pricing.html

262
20.1.5 Negotiated Transfer Prices

In some cases, the divisions of a company are free to negotiate the transfer
price between themselves and then to decide whether to buy and sell
internally or deal with outside parties. Negotiated transfer prices are often
employed when market prices are volatile and change occurs constantly. The
negotiated transfer price is the outcome of a bargaining process between the
supplying and receiving division.

20.1.6 Which is the optimal transfer price?

Minimum (fixed by the supplying division):

Transfer price variable cost of supplying division + any lost contribution

Maximum (fixed by receiving division):

Transfer price the lower of net marginal revenue of the receiving division
and the external purchase price

When unit variable costs and/or unit selling prices are not constant, there will
be a profit-maximising level of output and the ideal transfer price will only be
found by negotiation and careful analysis: -

1. Establish the output and sales quantities that will optimise the
profits of the company or group as a whole.
2. Establish the transfer price at which both profit centres would
maximise their profits at this company-optimising output level.
There may be a range of prices within which both profit centres
can agree on the output level that would maximise their
individual profits and the profits of the company as a whole. Any
price within the range would then be 'ideal'.

Illustration 2

263
Division A sells goods to Division B. Division Bs only source of getting
components is from Division A.

Division A Division B

$ $
Own costs: Variable 10 15
Fixed 14 18

Division B can sell goods to external customers for $80/unit.

For the selling division, Division A, the transfer price should be greater than
(or equal to) the variable cost of production. Hence, it cannot be lower than
$10.

For the buying division, Division B, the transfer price plus its own variable
costs must not be greater than the marginal revenue earned from external
sales. Hence, the transfer price must not be higher than $65.

$80 - $15 = $65

Hence the transfer price should be higher than $10 but less than $65.

This is known as the economic transfer price rule as this rule gives the
correct economic decision. If the final selling price is too low, for example $23,
no workable transfer price is available. Why? Division A will only accept to
transfer goods if the transfer price as at least $10. Division B will only accept
to purchase goods from Division A if the transfer price is $8 ($23 - $15).

Lecture Example 1

Division A has costs of $15 p.u., and transfer goods to Division B which has
additional costs of $10 p.u.. Division B sells externally at $35 p.u.. There is no
external market for Division As units.

Determine a sensible range for the transfer price in order to achieve goal
congruence.

Lecture Example 2

Bright Homes Ltd has two divisions, Bright and Homes. Bright produces two
products, X and Y. Product X is sold to external customers for $50 per unit.
There is no external market for Product Y. It is only sold to Homes Division.

Homes sells its goods externally. It can obtain its supplies (Product Y) from
either Bright or an external supplier for $40 per unit.

264
X Y
Variable costs per unit $34 $37
Fixed costs per unit $5 $5

Total cost per unit $39 $42

Required:-

Determine an appropriate transfer price for the sale of product Y from


division Bright to division Homes:-

a. When division Bright has spare capacity and limited external


demand for product X
b. When division Bright is operating at full capacity with unsatisfied
external demand for product X

Lecture Example 3

Division P has costs of $15 p.u., and transfer its goods to Division Q which
has additional costs of $5 p.u.. Division Q sells their goods to external
customer at $30 p.u..

The company has a policy of setting transfer prices at cost + 20%.

Calculate:

(a) the transfer price


(b) the profit made by the company overall and by each division
separately.

265
CHAPTER 21
Performance Analysis in Not for
Profit Organisations and the
Public Sector
A not-for-profit organisation is an organisation whose attainment of its
prime goal is not assessed by economic measures. However, in pursuit of that
goal it may undertake profit-making activities. (Bois)

Therefore one possible definition of a not-for-profit seeking organisation is that


its first objective is to be involved in non-loss operations to cover its costs,
profits only being made as a means to an end for example charities, statutory
bodies offering public transport or the provision of services such as leisure,
health or public utilities such as water or road maintenance.

21.1 ACCA SYLLABUS GUIDE OUTCOME 1:


Comment on the problems of having non-quantifiable objectives in
performance management
FAST FORWARD
A major problem with many not-for-profit organisations, particularly
government bodies, is that it is extremely difficult to define their objectives at
all. In addition they tend to have multiple objectives, so that even if they could
all be clearly identified it is impossible to say which is the overriding objective.

For example, objectives for not-for-profit organisations include:


Surplus maximisation (equivalent to profit maximisation)
Revenue maximisation (as for a commercial business)
Producer satisfaction maximisation (satisfying the wants of staff and
volunteers)
Client satisfaction maximisation (the police generating the support of
the public)

It is also difficult to measure the performance of not-for-profit organisations.


As already said, it is sometimes very difficult to define their objective at all, let
alone try to measure their performance.

266
Example

One of the objectives of the local council could be to provide adequate street
lighting throughout the area. Its other objective could be to improve road
safety.

1. How could the adequacy of street lighting be measured?


2. What is the amount of money to pay for adequately lit streets and
improved road safety?

Without information about what is being achieved (outputs) and what it is


costing (inputs), it is impossible to make efficient resource allocations. These
allocation decisions rely on a range of performance measures which, if
unavailable, may lead managers to allocate resources based on subjective
judgment.

Without performance measures, managers will not know the extent to which
operations are contributing to effectiveness and efficiency; when diagnostic
interventions are necessary; how the performance of their organisation
compares with similar units elsewhere; and how their performance has
changed over time.

Government may require performance information to decide how much to


spend in the public sector and where, within the sector, it should be allocated.
In particular they will be interested to know what results may be achieved as a
consequence of a particular level of funding, or to decide whether or not a
service could be delivered more effectively and efficiently in the private sector.
Likewise people who provide funds for other kinds of not-for-profit
organisations are entitled to know whether their money is being put to good
use.

21.2 ACCA SYLLABUS GUIDE OUTCOME 2:


Comment on the problems of having multiple objectives in this sector

Not-for-profit organisations have multiple stakeholders. This gives rise to


multiple objectives. Organisations will need to prioritise/compromise as, very
often, it is impossible to say which is the overriding objective.

Unlike commercial organisations, public sector organisations are subject to


strong political influences. For example, a decision to close a local hospital in
an effort to save costs is likely to be less acceptable to the public than the
closure of a factory for the same reason.

267
21.3 ACCA SYLLABUS GUIDE OUTCOME 3:
Explain how performance could be measured in this sector
Outline Value for Money (VFM) as a public sector objective

Performance is judged in terms of inputs and outputs and hence the value for
money criteria of economy, efficiency and effectiveness.86

Effectiveness is the relationship between an organisations outputs and


its objectives. It ensures that the outputs of a service or programme
succeed in achieving objectives.
Efficiency is the relationship between inputs and outputs. It is
concerned with maximizing output for a given input.
Economy is attaining the appropriate quantity and quality of inputs at
lowest cost. Therefore, economy is concerned with the cost of inputs.

Public sector organisations are now under considerable pressure to prove that
they operate economically, efficiently and effectively, and are encouraged
from many sources to draw up action plans to achieve value for money as part
of the continuing process of good management.

21.3.1 Indicators to assess overall performance

1. Effectiveness

Financial indicators

a. Quality of service/output measures; e.g. exam results; pupil/teacher ratio


b. Utilisation of resources; e.g. hospital bed occupancy; are trained teacher
fully used to teach the subjects they have been trained for?
c. Flexibility; e.g. average waiting time

Non-financial indicators

a. Workplace morale
b. Staff attitude to dealing with the public
c. Client satisfaction in the service being provided

2. Efficiency

Financial indicators to measure efficiency

a. Cost per unit of activity (e.g. cost per student)


b. Variance analysis
c. Comparisons with benchmark information
d. Cost component as a proportion of total costs

86 Examined Sept/Dec 2015 Qs 2

268
e. Costs recovered as a proportion of costs incurred (eg payment received
from householders requesting collection of bulky/unusual items of refuse)

3. Economy

A-value-for-money (VFM) audit will look also at the economy of the use of
resources, for e.g. in the case of state education, it will look into the cost
wages of school teachers, the cost of books, equipment.

Lecture Example 1

A government body uses measures based upon the three Es to the measure
for money generated by a publicly funded hospital. It considers the most
important performance measure to be cost per successfully treated patient.

Which of the three Es best describes the above measure?

A. Economy (A measure of cost related to input)

B. Effectiveness (A measure of output related to objectives)

C. Efficiency (A measure of output related to input)

21.4 ACCA SYLLABUS GUIDE OUTCOME 4:


Explain the need to allow for external considerations in performance
management, including stakeholders, market conditions and allowance
for competitors

Suggest ways in which external considerations could be allowed for in


performance management.

Performance management needs to allow for external considerations


including stakeholders, market conditions and allowance for competitiors.

21.4.1 Stakeholders

Stakeholders are groups of people or individuals who have a legitimate


interest in the activities of an organisation. They include customers,
employees, the community, shareholders, suppliers and lenders.

There are three broad types of stakeholder in an organisation.


Internal stakeholders (employees, management)
Connected stakeholders (shareholders, customers, suppliers,
financiers)
External stakeholders (the community, government, pressure groups)

269
The stakeholder approach suggests that corporate objectives are, or should
be, shaped and influenced by those who have sufficient involvement or
interest in the organisations operational activities.
FAST FORWARD
The objectives of the employees and management are likely to have a strong
influence on how an organisation is run. They are interested in the
organisations continued existence. They have individual interests and goals
which can be harnessed to the goals of the orgainisation.

Connected stakeholders are interested in increasing shareholder value. If


management performance is measured and rewarded by reference to
changes in shareholder value then shareholders will be happy, because
managers are likely to encourage long-term share price growth.

External stakeholder groups the government, local authorities, pressure


groups, the community at large, professional bodies are likely to have quite
diverse objectives. The government is interested in creating jobs, providing
training, and collecting tax. External stakeholders in particular induce social
and ethical obligations.

21.4.2 Market conditions

This would consider such factors as economic growth, inflation, interest rates,
exchange rates, government fiscal policy.

21.4.3 Competition

Performance management must consider information on competitors prices


and cost structures. It must identify which features of an organisations
products add value. Hence, management can react quickly and effectively to
changing market conditions.

21.5 ACCA SYLLABUS GUIDE OUTCOME 5:


Interpret performance in the light of external considerations

Organisations may need to develop performance measures to ensure that the


needs of stakeholders are met.

Examples of performance measures would be: -


Stakeholder Measure
Employees: - Employees Morale index, Retail price index
Management: - Salary, Share options
Shareholders: -Share price, dividend yield, Earnings per share
Customer: - Price, Quality, Service
Government: - Taxation, Inflation, Exports, Employment,
Community: - Salary, Employment, Pollution, Congestion

270
Lecture Example 2

Lewisville is a town with a population of 100,000 people. The town council of


Lewisville operates a bus service which links all parts of the town with the
town centre. The service is non-profit seeking and its mission statement is to
provide efficient, reliable and affordable public transport to all the citizens of
Lewisville. Attempting to achieve this mission often involves operating
services that would be considered uneconomic by private sector bus
companies, due either to the small number of passengers travelling on some
routes or the low fares charged. The majority of the town council members are
happy with this situation as they wish to reduce traffic congestion and air
pollution on Lewisvilles roads by encouraging people to travel by bus rather
than by car.

However, one member of the council has recently criticised the performance
of the Lewisville bus service as compared to those operated by private sector
bus companies in other towns. She has produced the following information:

Lewisville Bus Service


Summarised Income and Expenditure Account
Year ending 31 March 2006
$000 $000
Passenger fares 1,200
Staff wages 600
Fuel 300
Depreciation 280
1,180
Surplus 20

Summarised Balance Sheet as at 31 March 2006.

$000 $000
Fixed assets (net) 2,000
Current assets
Stock 240
Cash 30
270
Less creditors due within one year 60
Net current assets 210
Total assets less liabilities 2,210
Ordinary share capital (1 shares) 2,000
Reserves 210
2,210

Operating Statistics for the year ended 31 March 2006

271
Total passengers carried 2,400,000 passengers
Total passenger miles travelled 4,320,000 passenger miles

Private sector bus companies Industry average ratios


Year ended 31 March 2006.

Return on capital employed 10%


Return on sales (net margin) 30%
Asset turnover 033 times
Average cost per passenger mile 374p

Required:

(a) Calculate the following ratios for the Lewisville bus service
(i) Return on capital employed (based upon opening investment);
(ii) Return on sales (net margin);
(iii) Asset turnover;
(iv) Average cost per passenger mile.

(b) Explain the meaning of each ratio you have calculated. Discuss the
performance of the Lewisville bus service using the four ratios.

(c) Another council member suggests that the performance of the bus service
should be assessed on the basis of economy, effectiveness and efficiency.

Required:

Explain the meaning of the following terms in the context of


performance measurement and suggest a measure of each one
appropriate to a bus service.

(i) Economy;
(ii) Effectiveness;
(iii) Efficiency

(CAT June 2006 Qs 3)

272
Further questions

Question 1

A government is looking at assessing state schools by reference to a range of


both financial and non-financial factors, one of which is average class sizes.

Which of the three Es best describes the above measure?

A. Economy
B. Effectiveness
C. Efficiency
D. Externally

Question 2

A government is looking at assessing hospitals by reference to a range of


both financial and non-financial factors, one of which is survival rates for heart
by-pass operation.

Which of the three Es best describes the above measure?

A. Economy
B. Effectiveness
C. Efficiency
D. Externally

273

You might also like